You are on page 1of 312

This textbook was developed by TVTI faculties through KOICA

project “PMC Service for Capacity Development for TVET


Leaders and Trainers in Ethiopia”

Construction Planning and Resource


management

August 2022

Technical and Vocational Training Institute (TVTI)


Construction Planning and Resource
management

Teddy Abadi (M.Sc.)

Department Of Road Construction Technology


April 2022

TECHNICAL VOCATIONAL AND TRAINING


INSTITUTE (TVTI)

Yeka Subcity, Woreda 9, Addis Ababa, Ethiopia


Phone: 011-646-4455, Fax: 011-646-5675/5678
E-mail: info@etu.edu.et, Website: http://www.etu.edu.et
Contents
Course Profile.......................................................................................................I
Detail course outline...........................................................................................III
3.7.1. Resource leveling steps..................................................................VII
3.7.2. Heuristic Procedure for Resource Smoothing.................................VII
3.7.3. General producer for human resource leveling...............................VII
Chapter 1: Construction Planning........................................................................2
Lesson Plan (Construction Planning)...............................................................2
Chapter One: Construction planning...................................................................4
1.1 Introduction................................................................................................4
1.2 Basic concepts in the development of construction plans and Types of
project plans.....................................................................................................6
1.2.1 Time Plan.............................................................................................7
1.2.2 Plan.....................................................................................................7
1.3.3 Material Plan........................................................................................8
1.2.4 Construction Equipment Plan...............................................................8
1.2.5 Finance Plan.........................................................................................8
1.3 Project Planning Steps................................................................................9
1.4 Choice of technology and construction method..........................................9
1.5 Estimating resource requirements for work activities...............................12
1.6 Defining precedence relationships among activities.................................14
1.6.1. Overlap or lag......................................................................................2
1.6.2. Types of activities relationships..........................................................3
1.6.3. Drawing Project Network...................................................................5
1.6.3.2. Activity on node network (AON).......................................................7
1.6.3.3. Comparison between AOA and AON..............................................8
1.7 Estimating activity durations and Direct Cost.........................................13
Assessments......................................................................................................16
Practical..........................................................................................................16
Week 1: Practice.........................................................................................16
Week 2: Practice.........................................................................................16
Theory............................................................................................................17
Objective Type Question................................................................................17
Subjective type of question............................................................................18
Evaluation Sheet:...............................................................................................19
Reference...........................................................................................................20
Chapter 2: Construction Schedules....................................................................22
Lesson Plan (Construction Schedules)...........................................................22
Chapter 2: Construction Schedules....................................................................24
2.1 Introduction..............................................................................................24
2.2. Importance of Project Scheduling..........................................................24
2.2. Project schedule preparation steps...........................................................26
2.4. Other schedules derived from project schedules.....................................27
2.4.1. Preparing Invoice Schedule...............................................................27
2.4.2. Schedule of Milestone Events...........................................................28
2.4.3. Schedule of Plant and Equipment......................................................29
2.4.4. Schedule of Project Staff...................................................................29
2.4.5. Schedule of Labour Requirement......................................................30
2.4.6 Schedule of Materials Requirement...................................................31
2.4.7. Schedule of Specialized Agencies.....................................................32
2.5. Work breakdown structure......................................................................34
2.5.1. WBS coding......................................................................................36
2.6. Bar chart/ Gantt chart..............................................................................39
2.7. Network scheduling.................................................................................43
2.7.1. Rules for Preparing the Network Diagram.......................................43
2.7.2. Dummy Activity................................................................................44
2.7.3. Presentation of Networks and Precedence Diagram Method (PDM).44
2.7.4. Common Errors in Drawing Networks..............................................45
2.7.5 Numbering the events of a network activities....................................47
2.8. The Critical Path Method........................................................................47
2.8.1. Calculations for the Critical Path Method.........................................48
2.8.2. Critical Path Scheduling Algorithms................................................49
Precedence Network Calculations...............................................................50
2.8.3. Identifying the Critical Activities.....................................................55
2.9. Float Calculations....................................................................................56
2.10. Program Evaluation and Review Technique..........................................57
Assessments......................................................................................................66
1. Practical......................................................................................................66
Week 3: Practice.........................................................................................66
Week 4: Practice.........................................................................................67
Week 5 : Practice:.......................................................................................69
Week 6 : Practice........................................................................................69
Week 7: Practice:.......................................................................................70
Week 9: Practice........................................................................................71
2. Theory........................................................................................................72
Objective type of question...........................................................................72
Subjective Type Question...........................................................................73
Evaluation Sheet:...............................................................................................76
References.........................................................................................................77
Chapter 3: Resource Management.....................................................................78
Lesson Plan (Resource Management)............................................................78
Chapter 3: Resource Management.....................................................................82
3.1. Resource Management Process...............................................................82
3.2. Resource Management Techniques.........................................................82
3.2.1. Resource Forecasting........................................................................82
3.2.2. Resource Allocation..........................................................................83
3.2.3. Resource Leveling.............................................................................84
3.2.4. Resource Utilization..........................................................................84
3.3 Resource Management Tools...................................................................84
3.3.1. Resource Management Plan..............................................................84
3.3.2. Resources Breakdown Structure (RBS)............................................84
3.3.3. Responsibility Assignment Matrix....................................................85
3.4, Resource Management Key Terms..........................................................85
3.4.1. Resource Over allocation..................................................................85
3.4.2. Resource Dependency.......................................................................86
3.5. Human Resource Management................................................................86
3.5.1. Definitions:.......................................................................................86
3.5.2. Introduction......................................................................................87
3.5.3.Human resources planning.................................................................89
3.5.3.2. Construction Personnel (Field Supervision & Labour)...................91
3.5.3.3. Construction Sub-Contracting........................................................91
3.5.4. Construction Project Costs...............................................................92
3.5.5. Schedule of Labour Requirement.....................................................95
3.5.6. Labor and Labor Costs.....................................................................96
3.6. Resource Allocation and Aggregation...................................................100
3.7. Resource Leveling.................................................................................102
3.7.1. Resource leveling steps...................................................................106
3.7.2. Heuristic Procedure for Resource Smoothing.................................109
3.7.3. General producer for human resource leveling................................121
Assessment......................................................................................................129
Practical........................................................................................................129
Week 10: Practice.....................................................................................129
2. Theory......................................................................................................131
Objective type of questions.......................................................................131
Subjective type of questions......................................................................132
Evaluation Sheet:.............................................................................................135
References.......................................................................................................136
Chapter 4: Materials and Equipment Management..........................................138
Lesson Plan..................................................................................................138
Chapter 4: Materials and Equipment Management..........................................142
4.1. Materials Management..........................................................................142
4.1.1. Definition and Introduction............................................................142
4.1.2. Material Procurement Process in Construction Organization..........145
4.1.3. Materials Management Functions...................................................147
4.1.4. Inventory Management.......................................................................152
4.2. Equipment Management........................................................................173
4.2.1. Introduction.....................................................................................173
4.2.2. Advantages of using construction equipment..................................174
4.2.3. Classification of Construction Equipment.......................................174
4.2.4. Factors Behind the Selection of Construction Equipment...............181
4.2.5. Plant and Equipment Acquisition....................................................185
4.2.6. Equipment Costs, Life and Replacement Analysis..........................186
4.2.7. Equipment Utilization, Equipment Life Downtime and cost of
downtime...................................................................................................208
4.2.8. Factors for Equipment’s Downtime...............................................210
Assessment......................................................................................................212
Practical........................................................................................................212
Week 11: Practice.....................................................................................212
Week 12: Practice.....................................................................................215
Week 13 :Practice.....................................................................................216
Week 14 practice.......................................................................................216
Objective type questions..............................................................................217
Subjective type questions.............................................................................219
Evaluation sheet..............................................................................................222
References.......................................................................................................223
Chapter 5: Time Management.........................................................................224
Lesson Plan (Time Management).................................................................224
Chapter 5: Time Management.........................................................................226
5.1. Introduction...........................................................................................226
5.2. Importance of Time Management in Construction................................227
5.3. Network Crashing and Cost-Time Trade-Off........................................228
5.4. Activity Time-Cost Relationship...........................................................231
5.5. Project Time-Cost Relationship.............................................................235
5.6. Shortening Project Duration..................................................................236
Assessment......................................................................................................259
Practicae.......................................................................................................259
Week 15 :Practicae....................................................................................259
Theory..........................................................................................................264
Objective type question.............................................................................264
Subjective type of questions......................................................................272
Evaluation sheet..............................................................................................276
References.......................................................................................................277
Course Profile
Department Road Construction Technology

Course title Course Title: Construction Planning and Resource Management

Construction planning and project management is a relatively new field,


especially in the Ethiopian construction industry. However, its impact has
been quite remarkable. It has become an important practice for improving
the efficiency of construction operations. This course deals with some
Course topics and tools in the large field of project management. This content will
description
be dedicated mainly to undergraduate engineering students, especially
Civil Engineering students where most of the applications are presented in
the civil engineering field and integrated with the construction industry. It
provides the reader with the main practical knowledge to manage a
construction project from preliminary stages to handover.
Upon successful completion of the course, students will be able to:
 Identify the general management and project management;
 Prepare preplanning and contract strategy;
 Prepare project scheduling and planning;
Course  Prepare Scheduling of Linear Projects;
 Identify resource management;
objective  Allocate human resource management in a construction project;
 Control construction project;
 Define principles of planning, scheduling, and management;
 Practice MS project and Primavera;
 Compare manual and computerized construction planning and
scheduling and management techniques.
Knowledge:- After the completion of this course the student will be able
to:
1.1. Understand the project life cycle
1.2. Acquiring knowledge regarding different planning phase
1.3. understand the difference between planning and scheduling
1.4. Understand the concept of PERT and CPM
1.5. Understand the concept and the purpose of scheduling
Learning 1.6. Acquire knowledge resource management
outcomes
1.7. To allocate different types of resources for a particular project
1.8. Understand resources loading, resource allocation, and resource
leveling
Practical skills: After the completion of this course the student will be
able to:
1.1. plan a particular project
1.2. Prepare schedule using CPM
1.3. Difference different types of resources (consumable and non-
I
consumable resources)
1.4. Allocate human resources for construction project
1.5. Allocate machinery and equipment for construction project
1.6. Allocate different consumable materials
1.7. Prepare resource loading, to allocate available resources and level
resources
Pre- None (even if there are no pre-requisite for this course, others should have
requisite background students are required to have priority to start the course can be
started)
Target
group • Road students and experts

II
Detail course outline

ETHIOPIAN TECHNICAL UNIVERSITY


FACULTY OF FACULTY OF CIVIL
DEPARTMENT OF ROAD
TECHNOLOGY CONSTRUCTION TECHNOLOGY
Program Bachelor of Technology in Road Construction Technology
1. Instructor/s (teaching team members) Construction
Instructor Information Other teaching team members' information
Name Role Choose an item.
Office Name
Location
Phone Number Office Location
E-mail Phone Number & Email
Office Hours Office Hours
2. Course Information
Course Name/Title Construction Planning and Resource Management
Course Code RCTE 3024
Credit hours 3 Cr. Hrs /5 ECTS
Workload Lecture Tutorial Lab. Home study Assessment Total
32 48 ----- 49 6 135
Pre-requisite, and/or None even if there are no pre-requisites for this course others should
other restrictions have a background and students are required to have priority to starting
the course be stated
Target group 3rd year Road Construction Technology
Semester Semester II
Course of delivery Semester based
Status of the course Core Compulsory/Supportive/ General/ Vocational Pedagogy
3. Course Description and Objectives
Course Description: Construction planning and project management is a relatively new field,
especially in the Ethiopian construction industry. However, its impact
has been quite remarkable. It has become an important practice for
improving the efficiency of construction operations. This course deals
with some topics and tools in the large field of project management.
This content will be dedicated mainly to undergraduate engineering
students, especially Civil Engineering students where most of the
applications are presented in the civil engineering field and integrated
with the construction industry. It provides the reader with the main
practical knowledge to manage a construction project from preliminary
stages to handover.
Course objective: Upon successful completion of the course, students will be able to:
 Identify the general management and project management;
 Prepare preplanning and contract strategy;
 Prepare project scheduling and planning;
 Prepare Scheduling of Linear Projects;

III
 Identify resource management;
 Allocate human resource management in a construction project;
 Control construction project;
 Define principles of planning, scheduling, and management;
 Practice MS project and Primavera;
 Compare manual and computerized construction planning and
scheduling and management techniques.
4. Method of Instruction
Class lectures 3 lecture hours every week
Active learning (involves the full participation of students)
• Conducting the lecture using deductive and inductive methods
according to the nature of the topic provided.
• Use multi-media, animated models, written materials, and physical
objects
•Active learning (involves the full participation of students)
• Conducting the lecture using deductive and inductive methods
according to the nature of the topic provided.
• Use multi-media, animated models, written materials, and physical
objects
Practical 3 practical hours every alternative week.
The student shall perform the given Laboratory works, gather
information from Laboratory results, and make necessary calculations.

•Preparing laboratory report and smiting to the technical assistance


Study of lecture This is fully the responsibility of the student
notes.
In every lab activity, different questions shall be given which the
Lab assignments
students shall include in their lab reports.
Demonstrating how to conduct construction planning and resource
Demonstrations management i.e. planning, resource calculation, resource allocation,
resource leveling, project scheduling, project leveling, etc.
Individual The student …
assignment
Group Assignment The student …
5. Learning Outcomes
5.1. Knowledge: - After the completion of this course the student will be able to:
5.1 Understand the project life cycle
.1
5.1 Acquiring knowledge regarding different planning phase
.2
5.1 understand the difference between planning and scheduling
.3
5.1 Understand the concept of PERT and CPM
.4
5.1 Understand the concept and the purpose of scheduling
.5
5.1 Acquire knowledge resource management

IV
.6
5.1 To allocate different types of resources for a particular project
.7
5.1 Understand resources loading, resource allocation, and resource leveling
.8
5.2. Practical skills:- After the completion of this course the student will be able to:
5.2 plan a particular project
.1
5.2 Prepare schedule using CPM
.2
5.2 Difference different types of resources (consumable and non-consumable resources)
.3
5.2 Allocate human resources for construction project
.4
5.2 Allocate machinery and equipment for construction project
.5
5.2 Allocate different consumable materials
.6
5.2 Prepare resource loading, to allocate available resources and level resources
.7
5.3. Attitude:- Develop professional ethics for teaching /Training requirements
6. Detail Course Outline
Required
Topics to be covered Topics to be covered Outcom materials
Week
(2 lecture hours) (3 lab hours) es /equipment/d
evices
Chapter One: Construction Planning

1.1. Introduction  Introducing to 5.1.1, Internet,


1.2. Basic concepts in the development construction 5.1.2 Textbooks,
of construction plans and Types of planning and 5.1.3, MS Project,
choice of 5.2.1 or
1 project plans
technology and primavera
1.3. Project Planning Steps construction
1.4. Choice of technology and method
construction method
 Determination 5.1.1, Internet, PC,
1.5. Estimating resource requirements
of activity 5.1.2 Textbooks,
for work activities relationship, 5.1.3, charts,
1.6. Defining precedence relationships activity 5.2.1 MS Project,
2
among activities duration, and or
1.7. Estimating activity durations and estimated primavera
Direct Cost resource for
each task
Chapter Two: Scheduling procedures and techniques

3 2.1. Construction Schedules  Scheduling 5.1.2, Internet,


2.1.1 Importance of Project Scheduling and resource 5.1.5, Textbooks,

V
2.1.2 Project schedule preparation steps assigning 5.2.1 PC, charts,
2.1.3 Other schedules derived from project  Coding to MS Project,
schedules tasks or
2.2. Work breakdown structure primavera
2.2.1. WBS coding
2.3. Bar chart/ Gantt chart
4 2.4.Network scheduling  Developing 5.1.2, Internet,
2.4.1. Rules for Preparing the network 5.1.5, Textbooks,
Network Diagram scheduling 5.2.1 PC, charts,
2.4.2. Dummy using Ms MS Project,
2.4.3. Presentation of Networks and project or
Precedence Diagram Method (PDM) primavera
2.4.4. Common Errors in Drawing Textbooks,
Networks PC, charts
2.4.5. Numbering the events of a
network activities
5 2.5.Critical Path Method  Scheduling 5.2.1 Internet,
2.5.1. Calculations for the Critical Path  Solve problems 5.1.1 Textbooks,
Method associated with 5.1.2. PC, charts,
2.5.2. Critical Path Scheduling Algorithms scheduling, and MS Project,
5.1.3.
2.5.3. Identifying the Critical Activities critical path using or
2.6.Float Calculations AOA and PDM 5.1.4 primavera
2.7. Program Evaluation and Review and determining 5.2.5 Textbooks,
Technique the float PC, charts
Chapter Three: Labour management

6 3.1. Resource Management Process  Resource 5.1.3 PC, charts,


3.1.1. Resource Planning utilization 5.2.4 MS
3.1.2.Resource Scheduling  Resource 5.3 Project, or
measuring primavera
3.1.3.Resource Allocation
 Resource
3.2. Resource Management Techniques Allocation
3.2.1.Resource Forecasting  Resource
3.2.2.Resource Allocation Scheduling
3.2.3.Resource Leveling  Resource over-
3.2.4.Resource Utilization allocation and
3.3. Resource Management Tools dependency
3.3.1. Resource Management Plan
3.3.2.Resources Breakdown Structure
(RBS)
3.3.3.Responsibility Assignment Matrix
Resource Management Key Terms
3.3.4. Resource Over allocation
3.3.5. Resource Dependency

7 3.4. Human Resource Management  Human resource 5.1.3 PC, charts,


3.5.1. Definitions: management 5.2.4 MS
 Labour 5.3 Project, or

VI
3.5.2. Introduction scheduling primavera
3.5.3.Human resources planning  Calculating
3.5.3.1. Home office personnel labour cost
3.5.3.2. Construction Personnel (Field
Supervision & Labour)
3.5.3.3. Construction Sub-Contracting
3.5.4. Construction Project Costs
3.5.5.Schedule of Labour Requirement
3.5.6. Labor and Labor Costs
3.5.6.1. Indirect labour cost
3.5.6.2. Labour Unit Costs
3.5. Resource Allocation and
Aggregation
8 3.6. Resource leveling  Leveling or 5.1.8, Software
3.6.1. Resource leveling steps Smoothing a 5.2.4, (primavera
3.6.2. Heuristic Procedure for Resource given resource 5.2.7 or MS
fluctuation project)
Smoothing
3.6.3. General producer for human
resource leveling
9 Mid-term exam

Chapter Four: Materials and Equipment

10 4.1 Materials Management  Determination of 5.1.6,


4.1.1. Definition and Introduction Time of material 5.1.7,
Internet
4.1.2.Material Procurement Process purchasing, 5.1.8,
and road
in Construction Organization delivery, quantity, 5.2.6
text Books
4.1.3.Materials Management transportation, and
-
Functions distribution
4.1.4.Inventory Management
11 4.2 Equipment Management  Determination of 5.1.6 Internet,
4.2.1.Introduction the construction 5.1.7 Different
4.2.2.Advantages of using equipment 5.1.8 road
construction equipment 5.2.6 constructio
4.2.3.Classification of Construction n
Equipment equipment
4.2.4.Factors Behind the Selection of
Construction Equipment
4.2.5.Plant and Equipment
Acquisition
12 4.2.6.Equipment Costs, Life and  Determination of the 5.1.6 Internet,
Replacement Analysis construction 5.1.7 charts, and
equipment cost, 5.1.8 Road
5.2.6 Equipment
depreciation, salvage
value

VII
13 4.2.7.Equipment Utilization,  Obtaining the 5.1.6 Internet,
Equipment Life Downtime and equipment 5.1.7 road text
cost of downtime utilization 5.1.8
4.2.8.Factors for Equipment’s 5.2.6
mechanism and cost
Downtime
of downtime

Chapter Five: Time management

14 5.1. Introduction  Project time 5.2.6, MS project


5.2. Importance of Time Management management 5.3.7 or
in Construction  Measuring the primavera
5.3. Network Crashing and Cost- critical path software
Time Trade-Off
5.4. Activity Time-Cost Relationship
15 5.5. Project Time-Cost Relationship  Project crashing 5.1.2, Internet,
5.6. Shortening Project Duration / 5.1.7, charts,
project crashing 5.2.1 software
16 Final Exam

7. Suggested texts and reference materials

Text Book 1. Fundamentals of Construction Management

Reference 1. Principles & Practices of Commercial Construction, by Cameron K. Andres P.E.


Books Retired, Ronald C. Smith & W. Ronald Woods
2. Microsoft Project 2013 Step by Step by Carl Chatfield &Timothy Johnson
3. Building Construction: Project Management, Construction Administration,
Drawings, Specs, Detailing Tips, Schedules, Checklists, and Secrets Others
Don’t Tell You: Architectural Practice Simplified, by Gang Chen
4. Construction Project Management, by Frederick Gould & Nancy Joyce
5. Principles & Practices of Commercial Construction, by Cameron K. Andres P.E.
Retired, Ronald C. Smith & W. Ronald Woods
6. Microsoft Project 2013 Step by Step by Carl Chatfield &Timothy Johnson
8. Assessment methods

Type Weight Behavior and Criteria/ Assessment for Learning

Mid Semester Exam 20%


Continuous 10% Question paper which includes information such as due date, to
Assessment and whom to submit and place along with questions shall be given.
assignment (1)
Continuous 10% Question paper which includes information such as due date, to
Assessment and whom to submit and place along with questions shall be given.
assignment (2)

VIII
Lab work and Different final course projects shall be given earlier
Group assignment immediately in the second week and students will work on and
20%
or mini-project present before the final exam and evaluate within a group.
applicable
Final Exam 40%

Total 100%
9. Academic Honesty

In all cases i.e. In all cases i.e. in performing assignments, laboratory works, project works, and
examinations, copying from others and using others’ work as own is considered to be cheating,
and cheating is forbidden by the law of the academic principles and regulations of the institute.
Failure to do so will lead to taking disciplinary action starting from canceling the results of the
assignments; project works, laboratory activities, and examination up to the dismissal of the
institute in consultation with the concerned body.
10. Submitting Date

All assignments, project works and laboratory reports should be submitted to the instructor or
laboratory assistant according to the timetable provided. All assignments, project works and
laboratory results will be invalid if they are not submitted on time and will be reported to the
concerned body as miss conduct of the student.
11. Classroom Behavior

Classroom discipline is primary for a healthy teaching-learning process. Therefore, it is the


responsibility of the class to avoid disturbing behaviors and activities that compete for the
attention of the class and the instructor. Switching off the cell phone is vital in class. Failure to
do so will lead to taking disciplinary measures.
12. Approval

Name Signature Date

Instructor:

Section Head:

Department Head:

IX
X
1
Chapter 1: Construction Planning

Lesson Plan (Construction Planning)

1. Learning objectives
 Be able to prepare project plans in terms of defining: activities, logical relations,
durations, and activities' direct cost.
 Be able to identify the types of project plans in the development of construction plans
 Be able to know the project planning steps
 Be able to Drawing Project Network
 Be able to prepare Project network representation using different graphical methods
including activity on arrow and activity on node.

2. Motivation
 Outputs through Question and Answer; repetitive discussion and instruction of the
project planning, types of project plans, activities, logical relations, durations, activities
direct cost, and Project network representation.

3. Expectations or Outcomes
 Define the terms of defining: activities, logical relations, durations, and activities' direct
cost
 Can identify the types of project plans in the development of construction plans
 knowing the project planning steps
 Can prepare Drawing Project Network and Project network representation using
different graphical methods.

4. Equipment
 Desktop or Laptop computer with internet connection.
 MS software or primavera

2
5. Practice contents/Activities/Safety

6. Clean-up
 After finishing practice, all tools and equipment utilized in the practice shall be
returned to the proper storage place.
 Clean up the practice shop.

7. Independent practice/Follow-up activities


Learning through assignment

8. Review/Reflection
Review the outcome of the practice, improvement measures, and previously reflected

opinions.

3
Chapter One: Construction planning

1.1 Introduction

Planning is the process of identifying the methods, resources, and activities necessary to
accomplish the project’s objectives. It achieves this by drawing on the expertise, experience,
and knowledge of organizations and individuals to:
 Understand the need, problem, or opportunity that the project will address and the
benefits that it will deliver;
 Define what has to be accomplished and delivered, typically stated in terms of scope,
time, budgets, and quality;
 Develop a plan to deliver the project.
Planning is a general term that sets a clear road map that should be followed to reach a
destination. The term, therefore, has been used at different levels to mean different things.
Planning involves the breakdown of the project into definable, measurable, and identifiable
tasks/activities, and then establishes the logical interdependences among them. Generally,
planning answers the questions:
 What is to be done?
 How to do it?
 Who does it? (In construction)
For example, plans may exist at several levels: corporate strategic plans, pre-tender plans,
pre-contract plans, short-term construction plans, and long-term construction plans.
Generally, these plans are different from each other; however, all these plans involve four
main steps:
 Performing breakdown of work items involved in the project into activities.
 Identifying the proper sequence by which the activities should be executed.
 Activities representation.

 Estimating the resources, time, and cost of individual activities.

4
Detailed planning for tendering purposes and the preparation of construction needs to be
conducted through brainstorming sessions among the planning team. The inputs and outputs
of the planning process are shown in Figure 1.1

 Activities
 Contract information
 Relationships
 Drawings
among activities
Inputs  Specifications Output
 Method statement
 Available resources Planning  Responsibility
 Bills of quantities
 Project network
 Organizational data
diagram
 Construction methods
 Activities duration

Figure 1.1: Planning inputs and outputs

Some of the activities involved in construction planning are briefly discussed below:
Define the scope of work: Since all activities involve the consumption of different resources
to different extents, the scope of work involved must be properly and, to the extent possible,
completely defined. Any modification in the scope could have serious repercussions in terms
of the time of completion and cost, and even be the root of litigation, besides souring the
relationships between different agencies. For example, if felling trees and getting
environmental clearances are added to the scope of a contractor who has been awarded a job
for the construction of roads, it would cause difficulties.
Identify activities involved: This part of planning is very closely linked to defining the scope
and involves identifying activities in a particular job. Since different activities involved
consume different physical resources to varying extents, these activities must be exhaustively
listed, along with the resources required. For example, through different agencies that may be
concerned with ‘environmental impact assessment’, it is important for them to identify the
tools or parameters each will be using to plan effectively.
Establish project duration: This can be done only with a clear knowledge of the required
resources, productivities, and interrelationships. This information is used to prepare a
network and other forms of representations outlining the schedules. It may be remembered

5
that the duration required for any activity is related to the resources committed, and it may be
possible to reduce the project duration by increasing the resource commitment, even at an
additional cost. Thus, a balance between time and project cost is required to arrive at an
optimum level of resource commitment.
Define procedures for controlling and assigning resources: It is important that the
planning document prepared is followed by others involved in the execution of the project, or
its phases. Thus, the procedures to be followed for procurement and control of resources for
different activities, manpower, machines, material, and money are also laid down.
Develop appropriate interfaces: The planner needs to devise an appropriate system for
management information system (MIS) reporting. Tools such as computers and formats for
reporting are widely used, and it may be noted that software is readily available to aid the
planner.
Update and revise plans: Although a construction plan needs to be continuously updated,
and revised during monitoring, some basic issues should be borne in mind before drawing up
a full-fledged plan. For example, the planner should clearly understand the product to be
produced in terms of scope and expected performance, the input required and the process
involved, including the issues in quality control and tolerances at different steps. At the same
time, the time and productivity aspects involved in the different activities should also be
understood, besides the interdependence of activities. The planning should also identify
milestones and targets for the different agencies to facilitate proper monitoring during
execution. The inclusion of features identifying risks associated with a project, and the
appropriate responses for mitigation enhance the quality of the project plan.

1.2 Basic concepts in the development of construction plans and


Types of project plans

Schedule, cost, quality, and safety can be identified as specific items on which the success of
any construction project is evaluated. Although there is a complex interrelationship between
these, it is possible to discuss them independently a statement such as a project being
completed with very high quality but with different levels of cost and time overruns can at
least be technically understood. Thus, at times it makes sense to have different plans for each
of these criteria and draw up a time plan or schedule, cost plan, quality plan, and safety plan.

6
Specifically, depending upon the nature and stage of the project, one may also need to deal
with a plant and equipment plan, a maintenance plan, and a staff deployment plan. It should
be emphasized that all ‘independent’ plans should always be in line with an overall ‘master
plan’ that lays down the overall plan of the project. A brief description of some of the
commonly used plans in the construction industry is:

1.2.1 Time Plan

Time is the essence of all construction projects, and contracts often have clauses outlining
awards (bonus payments) or penalties (as liquidated damages) for completing work ahead or
later than a scheduled date. While effort is made to ensure timely completion of work, it
should be noted that some of the common reasons for delays could be a sluggish approach
during planning, delay in award of contract, changes during execution, and alterations in the
scope of work, delay in payments, slow decision-making, and delay in supply of drawings,
materials, and labor trouble. Several reasonably well-established techniques are available and
commonly used for time planning (or ‘scheduling’) activities. For instance, critical path
method (CPM), program evaluation and review technique (PERT), precedence network
analysis (PNA), line of balance technique (LOB), linear program chart (LPC), and time scale
network (TSN). The choice of the method to be used in a particular case depends on the
intended objective, the nature of the project, the target audience, etc.

1.2.2 Plan

This plan focuses on estimating the size of the workforce, division into functional teams, and
scheduling the deployment of manpower. It may be noted that manpower planning also
involves establishing labor productivity standards, providing a suitable environment and
financial incentives for optimum productivity, and grouping the manpower in suitable
functional teams to get optimum utilization.

7
1.3.3 Material Plan

The material plan involves the identification of required materials, estimation of required
quantities, defining specifications, and forecasting material requirement, besides the
identification of the appropriate source(s), inventory control, procurement plans, and
monitoring of the usage of materials.

1.2.4 Construction Equipment Plan

Modern construction is highly mechanized and the role of heavy equipment in ensuring
timely completion of projects cannot be over-emphasized. Machines are used in modern
construction for mass excavation, trenching, compacting, grading, hoisting, concreting,
drilling, material handling, etc. The induction of modern equipment could improve
productivity and quality, besides reducing costs. At the same time, it should be borne in mind
that heavy equipment is very costly and should be optimally utilized to be productive also the
characteristics of equipment must be kept in mind when drawing up an equipment plan.

1.2.5 Finance Plan

Large construction projects require huge investments, and a long time to complete, it is
obvious that all the money is not required at any one point in time. Contractors fund their
projects from their working capital, a part of which is raised by the contractors using their
sources (e.g., bank loans secured against assets, deployment of resources from their
inventory), whereas the rest comes from a combination of avenues such as mobilization
advance for the project, running-account bills paid by the client, secured advances against
materials brought at the site, advance payments, and credits from suppliers against work
done. So careful analysis needs to be carried out to determine how the requirement of funds
varies with time. It is little wonder that capital inflow can be looked upon as the lifeline of
any large project. Careful planning for funds and finances has achieved added significance in
cases when projects are funded by the private sector or financial institutions that view the
project as a financial investment and seek returns in monetary terms also.

8
1.3 Project Planning Steps

The following steps may be used as a guideline, or checklist to develop a project plan:

 Define the scope of work, method statement, and sequence of work.


 Generate the work breakdown structure (WBS) to produce a complete list of activities.
 Develop the organization breakdown structure (OBS) and link it with the work
breakdown structure o identify responsibilities.
 Determine the relationship between activities.
 Estimate activity's time duration, cost expenditure, and resource requirement.
 Develop the project network.

1.4 Choice of technology and construction method

Along with the development of civilization, the development of technologies for carrying out
construction projects and the capabilities of civil engineering have taken place. The
expectations of decision-makers to build faster, more, and construct taller construction have
increased as well. The rising population numbers and technological progress have led to the
emergence of urban agglomerations that concentrate on a significant and constantly
increasing number of construction projects. The carrying out of a construction project located
in an urban agglomeration is currently associated with various technological and
organizational problems that evolve along with it.
A construction project is a development effort undertaken to meet the needs of a developer
and its future users. A construction project includes work meant to prepare, organize and
perform construction work, settle its accounts, and hand over a finished product ready for
operation. The carrying out of a construction project is a highly complicated process that is
dependent on non-repeatable external factors and their characteristics. The basic
characteristics of a construction project include:
 The individual character of a construction project,

9
 The natural environment and direct interference within it
 The space in which construction is being performed, the need to manage it,
 The duration of carrying out a project,
 The set budget,
 The proper quality of construction work (special technical, functional, and
economic requirements),
 local legal regulations,
 The active cooperation of stakeholders: the developer, designer, and contractor,
 Availability of a large number of contractors performing work on a structure, the
cooperation of numerous specialists and companies (designers, construction
engineers, workers, suppliers, material and machinery manufacturers),
 High costs of machines and manpower,
 The diversity of construction technologies,
 The seasonal nature of the work being performed (the capacity to perform
construction work varies throughout the year: it is higher in summer and lower in
winter),
 The development of new technologies and technological progress, enabling the
dynamism of construction services,
 The random factors such as atmospheric conditions, logistics problems, changes in
human resources, equipment failure
As in the development of appropriate alternatives for facility design, choices of appropriate
technology and methods for construction are often yet critical ingredients in the success of
the project.
For example, a decision on whether to pump or transport concrete in buckets will directly
affect the cost and duration of tasks involved in building construction. A decision between
these two alternatives should consider the relative costs, reliabilities, and availability of
equipment for the two transport methods. Unfortunately, the exact implications of different
methods depend upon numerous considerations for which information may be sketchy during
the planning phase, such as the experience and expertise of workers or the particular
underground condition at a site. In selecting alternative methods and technologies, it may be
necessary to formulate several construction plans based on alternative methods or

10
assumptions. Once the full plan is available, then the cost, time, and reliability impacts of the
alternative approaches can be reviewed.
This examination of several alternatives is often made explicit in bidding competitions in
which several alternative designs may be proposed for alternative construction methods may
be permitted. In this case, potential constructors may wish to prepare plans for each
alternative design using the suggested construction method as well as to prepare plans for
alternative construction methods which would be proposed as part of the value engineering
process. In informing a construction plan, a useful approach is to simulate the construction
process either in the imagination of the planner or with a formal computer-based simulation
technique. By observing the result, comparisons among different plans or problems with the
existing plan can be identified. For example, a decision to use a particular piece of equipment
for an operation immediately leads to the question of whether or not there is sufficient access
space for the equipment. Three-dimensional geometric models in a computer-aided design
(CAD) system may help simulate space requirements for operations and for identifying any
interference. Similarly, problems in resource availability identified during the simulation of
the construction process might be effectively forestalled by providing additional resources as
part of the construction plan.

Example- Laser Leveling An example of technology choice is the use of laser leveling
equipment to improve the productivity of excavation and grading. In these systems, laser
surveying equipment is erected on a site so that the relative height of mobile equipment is
known exactly. This height measurement is accomplished by flashing a rotating laser light on
a level plane across the construction site and observing exactly where the light shines on
receptors on mobile equipment such as graders. Since laser light does not disperse
appreciably, the height at which the laser shines anywhere on the construction site gives an
accurate indication of the height of a receptor on a piece of mobile equipment.

In turn, the receptor height can be used to measure the height of a blade, excavator bucket, or
another piece of equipment. Combined with electro-hydraulic control systems mounted on
mobile equipment such as bulldozers, graders, and scrapers, the height of excavation and
grading blades can be precisely and automatically controlled in these systems. This
automation of blade heights has reduced costs in some cases by over 80% and improved the

11
quality of the finished product, as measured by the desired amount of excavation or the extent
to which a final grade achieves the desired angle.
These systems also permit the use of smaller machines and less skilled operators. However,
the use of these semi-automated systems requires investments in laser surveying equipment
as well as modification to equipment to permit electronic feedback control units. Still, laser
leveling appears to be an excellent technological choice in many instances.

1.5 Estimating resource requirements for work activities

Estimating activity is the process of estimating the type and quantities of materials, human
resources, equipment, or supplies require performing each project activity. The accuracy of
the duration and cost estimation is built upon the understanding of the resources required for
each activity to achieve an accurate estimate of duration and cost for the project as a whole.
Strictly speaking if, you understand the resource details at the activity level then figuring out
the total cost and duration will be a breath. But, if you fail to understand the required
resources then expect the cost overrun, schedule slippage, and possibly total project filial.

Figure 1.2: Estimating activity resource requirements

12
Many factors affect the requirements; mainly there are four factors for estimating resource
requirements:
 Complexity of task
 Skill level of resource
 Organizational experience in doing similar activities
 Availability of the required resources.
If properly way these factors should be able to provide an accurate estimate for the
resource required. When choosing resources you must give adequate consideration to
what resource is best to choose to do a particular job. For instance digging a hole: 40 x 40
x 40cm:
 An 8-year-old child will complete the task within 1 to 2 hour
 A 22-year-old adult will complete the task within 10 to 20 minutes
 Excavating machine will complete the task within 10 seconds. So, hunting the
Excavating machine is the best choice to do the digging task associated with the
cost perspective.
The key benefit of estimating the resource requirements process is to identify the type,
quantity, and characteristic resource required to complete the activity which allows more
accurate cost and duration estimates.

Inputs Tools Outputs


 Resource management plan  Expert judgment  Activity resource requirement
 How to identify resources  Helps to choose the right, alterative,
 Basis of estimation
 How to quantify resources technique and determine the number of
Method used
 Schedule baseline(when) resources. Resource used
 Project documents  Published estimating data Constraints & new risk introduced
 Activity list  Rate of production
 Activity attributes
 Resource calendars
 Risk registers

Figure 1.3: Input, Tool, and Outputs requirements for activity resource
estimating

13
1.6 Defining precedence relationships among activities

This is the logical relationship implying that an activity needs one activity or more activities
to be completed before this activity can start. For example, to be able to start plastering, the
brickwork needs to have been completed, i.e., logically, brickwork precedes plastering. It is a
common practice in most construction projects to represent the precedence of activities in the
form of a table, called the precedence table. For preparing the precedence table, a list of
activities that should precede a given activity is given. It should also be mentioned that this
concept (of precedence) is sometimes referred to as ‘dependence’.
Strictly speaking, this implies that if it is identified that activities A, C, and D must precede
activity X (in other words, X depends on A, C, and D), in the parlance of network analysis,
precedence can be stated as follows: to initiate an activity (i, j), all activities having (i) as the
end event should have been completed. A variation of this concept could be for an activity
that can be started so long as another activity (which should logically proceed) has at least
started. For example, painting a wall should, indeed, be preceded by plastering, the latter
doesn't need to be completed before the former can be taken up. Painting can be taken up
even as plastering is being carried out, provided, of course, enough work fronts are available.
To identify the relationships among activities, the planning team needs to answer the
following questions for each activity in the project:
 Which activities must be finished before the current one can start?
 What activity may be constructed concurrently with the current one?
 What activity must follow the current one?
A circle of activity precedence will result in an impossible plan. For example, if activity A
precedes activity B, activity B precedes activity C, and activity C precedes activity A, then
the project can never be started or completed. Figure 1.3 illustrates the resulting activity
network.

14
Figure 1.4: Example of a circle of activity precedence

Example 1.1
Suppose a site preparation and concrete slab foundation construction project consists of nine
different activities:

A. Site clearing (of brush and minor debris),


B. Removal of trees,
C. General excavation,
D. Grading general area,
E. Excavation for utility trenches,
F. Placing formwork and reinforcement for concrete,
G. Installing sewer lines,
H. Installing other utilities,
I. Pouring concrete.
Activities A (site clearing) and B (tree removal) do not have preceding activities since they
depend on none of the other activities. We assume that activities C (general excavation) and
D (general grading) are preceded by activity A (site clearing). It might also be the case that
the planner wished to delay any excavation until trees were removed so that B (tree removal)
would be a precedent activity to C (general excavation) and D (general grading). Activities E
(trench excavation) and F (concrete preparation) cannot begin until the completion of general
excavation and grading, since they involve subsequent excavation and trench preparation.
Activities G (install lines) and H (install utilities) represent installation in the utility trenches
and cannot be attempted until the trenches are prepared so activity E (trench excavation) is a

15
preceding activity. We also assume that the utilities should not be installed until grading is
completed to avoid equipment conflicts, so activity D (general grading) is also preceding
activities G (install sewers) and H (install utilities). Finally, an activity I (pour concrete)
cannot begin until the sewer line is installed and formwork and reinforcement are ready, so
activities F and G are proceeding. Other utilities may be routed over the slab foundation, so
activity H (install utilities) is not necessarily a preceding activity for activity I (pour
concrete). The result of our planning is the immediate precedence shown in Table 1.1.

Table 1.1: Precedence relations for Example 1.1

Activity Description Predecessors


A Site clearing -
B Removal of trees -
C General excavation A
D Grading general area A
E Excavation for utility trenches B, C
F Placing formwork and reinforcement for a concrete B, C
G Installing sewer lines Installing D, E
H Installing other utilities D, E
I Pouring concrete F, G

Example 1.2

Determine the relationships between activities of the project studied below in fig 1.5 and the
list of activities in Table 1.2 which shows a double-span bridge. Break the construction works
of the bridge into activities. The plan will be used for bidding purposes.

16
Figure 1.5: Double span bridge

A list of the double-span bridge activities is shown in Table 1.2

Table 1.2: Activities of the double-span bridge

No Activity Description
1 10 Set-up site
2 14 Procure reinforcement
3 16 Procure precast beams
4 20 Excavate left abutment
5 30 Excavate right abutment
6 40 Excavate central pier
7 50 Foundation left abutment
8 60 Foundation right abutment
9 70 Foundation central pier
10 80 Construct left abutment
11 90 Construct right abutment
No Activity Description
12 100 Construct central pier
13 110 Erect left precast beams
14 120 Erect right precast beams
15 140 Fill left embankment
16 150 Fill right embankment
17 155 Construct deck slab
18 160 Left road base
19 170 Right road base
20 180 Road surface
21 190 Bridge railing
22 200 Clear site

17
Table 1.3: Solution of Example 1.3

Activity Description Predecessor


10 Set-up site -
14 Procure reinforcement -
16 Procure precast beams -
20 Excavate left abutment 10
30 Excavate right abutment 10
40 Excavate central pier 10
50 Foundation left abutment 14, 20
60 Foundation right abutment 14, 30
70 Foundation central pier 14, 40
80 Construct left abutment 50
90 Construct right abutment 60
100 Construct central pier 70
110 Erect left precast beams 16, 80, 100
120 Erect right precast beams 16, 90, 100
140 Fill left embankment 80
150 Fill right embankment 90
155 Construct deck slab 110, 120
160 Left road base 140
170 Right road base 150
180 Road surface 155, 160, 170
190 Bridge railing 155
200 Clear site 180, 190

1.6.1. Overlap or lag

Overlap between activities (negative lag) is defined as how much a particular activity must be
completed before a succeeding activity may start. The absence of overlap means that the first
activity must finish before the second may start. A negative overlap (lag) means a delay is
required between the two activities (Figure 1.6.)

18
Figure1.6: Overlap among activities

1.6.2. Types of activities relationships

Four types of relationships among activities can be defined as described and illustrated below
(Figure 1.7). Typically, relationships are defined from the predecessor to the successor activity.
a) Finish-to-start (FS). The successor activity can begin only when the current activity
completes.
b) Finish-to-finish (FF). The finish of the successor activity depends on the finish of the
current activity.
c) Start-to- start (SS). The start of the successor activity depends on the start of the current
activity.
d) Start-to-finish (SF). The successor activity cannot finish until the current activity starts.

Figure 1.7: Types of relationships

19
A. Finish to Start

A n B
Lag

Activity B can start after n days (units) from the finish of activity A. “n” is called the Lag
after the finish of “A”

B. Finish to Finish

A B

Activity “B” must finish n days (units) after the finish of activity “A”

C. Start to Start

A B

Activity “B” starts n days (units) after the start of activity “A”

D. Start to Finish
n

A B

Activity b must finish n days (units) after the start of the activity “A”

Figure 1.7: Types of relationships (continues)

20
1.6.3. Drawing Project Network

A network is a graphical representation of the project activities and their relationships. A project
network is a set of arrows and nodes. Before drawing the network, it is necessary to ensure that
the project has a unified starting and ending point. The need for this start activity arises when
there is more than one activity in the project that has no predecessors and the end activity is
needed when there is more than one activity that has no successors. Also, networks should be
continuous (i.e., each activity except the first and the last has both preceding and succeeding
activities). Two ways are commonly used to draw a network diagram for a project:

1. Activity on Arrow (AOA) representation.


2. Activity on Node (AON) representation

1.6.3.1. Activity on arrow network (AOA)

In this method, the arrows represent activities while the nodes represent the start and the end of
an activity (usually named events) (Figure 1.8). The length of the arrow connecting the nodes has
no significance and may be straight, curved, or bent. When one activity depends upon another,
both appear on the diagram as two arrows having a common node.
The following are some rules that need to be followed when constructing an AOA network
diagram:
 Each activity must have unique i - j numbers, where i (the number at the tail of the arrow)
is smaller than j (the number at the head of the arrow).
 Avoid back arrows.
In some situations, when more than one arrow leaves the same node and arrives at another node,
dummy activities must be used. The dummy activity is an activity with zero duration, consumes
no resources, is drawn as dashed lines, and is used to adjust the network diagram. A dummy
activity is also used when one activity depends upon two preceding activities and another activity
depends only upon one of these two preceding activities

21
Finish
Node

A D F
1 4 5 6

Starting
Node
B
C E
2 3

Figure: 1.8. AOA diagrams

22
1.6.3.2. Activity on node network (AON)

This method is also called the precedence diagram method. In this method, the nodes represent
activities and the arrows represent logical relationships among the activities. If the arrow starts
from the end side of an activity (activity A) and ends at the start side of another activity (activity
B), then A is a predecessor of B (Figure 1.10). AON representation allows the overlap or lag
representation on the relationship arrows connecting activities.

Figure 1.9: Use of dummy activity

23
Figure: 1.10. AON diagrams
Rules of Activity on node network
 There should be only one start node and only one finish node.
 At the starting node, no activity enters but many can start.
 At the finish node many activities can merge but no activity start.
 At the intermediate node, many activities can merge and many activities can start
 Two nodes should not be connected with two different activities

1.6.3.3. Comparison between AOA and AON

While both networks can be used to represent a project network, there are some differences
between them:
 There is no need for the use of dummy activities in AON representation.
 AON is more easily to draw and read.
 In AOA, an activity can only start when all its predecessors have finished.
 AON allows for overlap/lag representation.
 AON allows for the representation of the four types of relationships while AOA allows
only for the finish-to-start relationship.

Example 1.3:
Construct an AOA and AON network for the activities listed in Table 1.4.
Table 1.4: Data for Example 1.3.

No Activity Predecessors
1 A -
2 B -
3 C A, B
4 D C
5 E C
6 F D

24
7 G D, E

Forming an AOA network for this set of activities might begin by drawing activities A, B, and C
as shown in Figure 1.11 (a). At this point, we note that two activities (A and B) lay between the
same two events nodes; for clarity, we insert a dummy activity X and continue to place other
activities as in Figure 1.11 (b). Placing activity G in the figure presents a problem, however,
since we wish both activity D and activity E to be predecessors. Inserting an additional dummy
activity Y along with activity G completes the activity network, as shown in Figure 1.11 (c).

Figure 1.11: AOA Network for Example 1.1

To understand the drawing of the AON, some ordering for the activities may be necessary. This
is done by placing the activities in a sequence step order. A sequence step may be defined as the
earliest logical position in the network that activity can occupy while maintaining the logical
relationships. In this example, as there are two activities (activities A and B) has no predecessor,
then a start activity is added to have one unified start activity (Start) for the project. Also, a finish
activity (Finish) is added as there are two activities without successors (activities F and G).

25
Considering the data given in Table 1.5, sequence step 1 is assigned to the Start activity. Then,
we take all activities on the list one by one and look at their immediate predecessors and then
assign a sequence step that equals the highest sequence step of all immediate predecessors plus
one as given in Table 1.5 After all sequence step numbers have been assigned, the AON diagram
can be drawn.

Example 1.4

Figure 1.12: AOA Network

26
Example 1.5

Figure 1.13: AOA Network

Table 1.5: Determining the sequence steps

Activity Predecessors Sequence step (SS)


Start - SS(Start)=1
A Start 2=SS(Start)+1
B Start 2=SS(Start)+1
C A, B 3=Highest of [SS(B), SS(A)]
D C 4=SS(C)+1
E C 4=SS(C)+1
F D 5=SS(D)+1
G D, E 5=Highest of [SS(D), SS(E)]
Finish F, G 6= Highest of [SS(F), SS(G)]

27
AON representation is shown in Figure 1.14, including project start and finish nodes. Note that
dummy activities are not required for expressing precedence relationships in activity-on-node
networks.

Figure 1.14: An AON Network

Example: 1.6
Draw the AOA and AON networks for the following project:
This case study is for a small 3 houses project. The main segments of a single house, the
responsibilities, and the logical relationship are identified as follows: 11 work packages are
involved: A and B (civil work, substructure), C, D, E, and F (civil work, superstructure), G
(electrical, interior), H (electrical, exterior), I (mechanical, HVAC), J (mechanical, elevator), and
K (mechanical, plumbing) where the substructure is supervised by Teddy (activity A), and
Dagnew (activity B) and the superstructure is supervised by Luel (activities C and F) and Teame
(activities D and E).
All electrical work is supervised by Rahel.
HVAC and plumbing are supervised by Yordanos, elevator work is supervised by Afomya.
Activities E and F follow activity B.
Activity C precedes activity G.
An activity I follows the completion of activity E.
The predecessors to activity K are activities H and I.
Activity D follows activity A and precedes activity H.
28
Activity J is preceded by activities F and G.
Solution
The AOA is given in Figure 1.15 and the AON is given in Figure 1.16 as shown below.

Figure 1.15: AOA network

Figure 1.16: AOA network

1.7 Estimating activity durations and Direct Cost

Having defined the work activities, each activity has an associated time duration. These
durations are used in preparing a schedule. For example, suppose that the durations shown in
Table 1.6 were estimated for a project. The entire set of activities would then require at least 3

29
days since the activities follow one another directly and require a total of 1.0 + 0.5 + 0.5 + 1.0 =
3 days.

Table 1.6: Durations and predecessors for a four-activity project

Activity Predecessors Duration (Days)


Excavate trench - 1
Place formwork Excavate trench 0.5
Place reinforcing Place formwork 0.5
Pour concrete Place reinforcing 1

All scheduling procedures rely upon estimates of the duration of the various project activities as
well as the definitions of the predecessor relationships among activities. A straightforward
approach to the estimation of activity durations is to keep historical records of particular
activities and rely on the average durations from this experience in making new duration
estimates. Since the scope of activities is unlikely to be identical between different projects, unit
productivity rates are typically employed for this purpose. The duration of an activity may be
estimated as:

Activity duration = Quantity of work


Number of crews x Resource output

Typically, the quantity of work is determined from engineering drawings of a specific project.
The number of crews working is decided by the planner. In many cases, the number or amount of
resources applied to particular activities may be modified in light of the resulting project plan
and schedule. An estimate of the expected work productivity must be provided. Historical
records in a firm can also provide data for the estimation of productivity.
Having defined an activity duration means that the planner has already defined the number of
resources that will be employed in a particular activity. Knowing activity duration and resources
employed, it is simple to estimate the activity's direct cost. Then, the three elements of an
activity: duration, cost, and resources form what is called the construction method. Some

30
activities can be performed using different construction methods. Where, its method will have its
resources, cost, and duration.

Example 1.7:
If the daily production rate for a crew that works inactivity is 175 units/day and the total crew
cost per day is LE 1800. The material needed for daily work is 4.5 units at LE 100/unit.

a. Calculate the time and cost it takes the crew to finish 1400 units
b. Calculate the total unit cost. Consider an eight-hour workday
Solution
a. Duration (units of time) = Quantity / Production per unit of time x number of crews
= 1400 / 175 x 1 = 8 days
Cost (labor cost) = Duration (units of time) x crew cost per unit of time
= 8 days x LE 1800 / day
= LE 14400 Total direct cost
= Le 14400 + 4.5 units of material x LE 100 / day x 8 days
= LE 18000
b. Unit cost = total cost / quantity
= LE 18000 / 1400
= LE 12.86 / unit
Sometimes the productivity of a specific crew is expressed in man-hours/unit, not units/day. For
example, if the productivity is said to be 0.5 Man-hour/cubic meters, this means how long it will
take one labor to construct one unit. This way applied to any crew formation and work hours.
Example 1.8:
What is the duration in days to install 6000 square feet of walls shuttering if:
A. The crew of 2 carpenters is used with an output of 200 square feet/day
B. Productivity is measured as 0.008 man-hour/square feet. Number of carpenters =3, and
number of working hours/day = 8 hours
Solution
A. Duration = 6000 / 200 = 3 days
B. Total man-hours needed = 6000 x 0.008 = 48 man-hours (if one man used)

31
Duration = 48 / 8 = 6 days (if one man used)
Duration using 3 men = 6 / 3 = 2 days

Assessments

Practical

Week 1: Practice

Design a G + 3 building or 5-kilometer road with a bridge (Auto CAD) First identify the building
blocks of a project Tasks, what resources you will use, Second identify the three pillars of task
scheduling work, duration, and resource, and precedence of the activities and prepare that for
How does Ms. Project schedule a task:
 The link b/n the task and its predecessors
 The duration you enter for the task
 The resources working and nonworking time
 The resource units you assign to an activity

Week 2: Practice

Design a G + 3 building or 5-kilometer road with a bridge (Auto CAD) First identify the building
blocks of a project Tasks, what resources you will use, Second identify the three pillars of task
scheduling work, duration, and resource, and precedence of the activities and prepare that for
How does Ms. Project schedule a task:

 The link b/n the task and its predecessors


 The duration you enter for the task
 The resources working and nonworking time
 The resource units you assign to an activity
32
Theory

Objective Type Question

I. The elements of construction project planning are:


A. Time
B. Resources
C. Cost
D. All
II. Which of the following is not a typical activity category?
A. Production
B. Procurement
C. Administrative
D. None of the above

33
Subjective type of question

1. What does project planning?


2. What do the activity relationship and its precedence?
3. Deferential the activity on arrow and activity on node
4. What is the duration in days to install 6000 square feet of walls shuttering if: a. Crew of 2
carpenters is used with an output of 200 square feet/day b. Productivity is measured as 0.008
man-hour/square feet. Number of carpenters =3, and number of working hours/day = 8 hours
5. The construction of a reinforced concrete wall involves placing 660 m3 of concrete, fixing 50
tons of steel, and 790 m2 of formwork. The following information belongs to the jobs involved
in this activity: A 6-man concrete crew can place 16 m3 of concrete/day. A steel-fixer and
assistant can fix 0.5 tons of reinforcement/day. A carpenter and assistant can fix and remove 16
m2 of shuttering/day. So, calculate the duration of the activity considering the steel-fixer as the
critical resource.

34
Evaluation Sheet:

Evaluation Out of 6 Score Remark

Working
Principle
Road or Building
Design sample
Result
Evaluatio Discussion
n
Design output

Applications

Tool Setup

Usage and
Work storage of the
Evaluatio tools and
n materials
Accuracy of
Measurement and
Standards
Time used
Time
(180 Min.)

Total /54

35
Reference

1. Kumar Neeraj Jha (2015), (Associate Professor Department of Civil Engineering), IIT,
Delhi Construction Project Management, Theory, and Practice. SECOND EDITION

2. Emad Elbeltagi, Ph.D., P.Eng., (2009), Professor of Construction Management Structural


Engineering Department, Faculty of Engineering, Mansoura University, Lecture notes on
construction Project management

3. Cameron K. Andres P.E. Retired, Ronald C. Smith & W. Ronald Woods, Principles &
Practices of Commercial Construction.

4. Frederick Gould & Nancy Joyce, Construction Project Management.

5. Fundamentals of Construction ManagementCormican, David. (1985). “Construction


Management: Planning and Finance.” Construction Press, London.

6. Pilcher, Roy. (1992). “Principles of Construction Management.” Mc-Graw Hill Book


Company, 3rd ed.

7. Elgare Allah, Mohamed Ibrahim & Nawara, Jamal Mohamed. (1984). “Edarat Almsharee’
Alhandaseah.” John Wiley & Sons Inc., NY. (This book is available in Arabic).

36
37
38
Chapter 2: Construction Schedules

Lesson Plan (Construction Schedules)

1. Learning objectives
At the end of this chapter trainees will be able to:

 Prepare project schedules;


 Identify the Importance of Project Scheduling;
 Identify and know the steps of preparing scheduling and other schedule derived from
schedules;
 Develop the level of understanding of the Work breakdown structure and Work
breakdown structure coding;
 Know the scheduling techniques(Bar chart/Gantt chart, CPM, PERT);
 Know the relationships existing among different activities in the form of a network
for a project, Network Scheduling, Presentation of networks, and Precedence
Diagram Method.
 Understand how to determine the Calculations of Critical Path Method, Backward
(Earliest Time), Forward path (Latest Time), float, determination;
 Identify the method of determining the probability beta distribution of Mean (Te),
Standard deviation (σ), variance (ν), and the probability of activity completion (Z) of
the project.
2. Motivation
Outputs through Question and Answer; repetitive discussion and instruction of the project
schedule, steps of project scheduling, activities precedence and work break down structure,
Scheduling Techniques, Network Scheduling, presentation of Network, forward and
backward path determination, critical path determination and probability beta distribution of
Mean (Te), Standard deviation (σ), and variance (ν) and the probability of activity completion

39
(Z) of project determination.
3. Expectations or Outcomes
 Define the terms of project schedule, steps of project scheduling, activities precedence and
work break down structure, Scheduling Techniques, Network Scheduling, presentation of
Network, forward and backward path determination, critical path determination, and
probability beta distribution of Mean (Te), Standard deviation (σ), and variance (ν) and the
probability of activity completion (Z) of project determination;
 Able to identify the types of project scheduling;
 knowing the project scheduling steps;
 Can prepare Project scheduling, Network, and Project network representation using different
graphical methods;
 Able to determine the Network, forward and backward path and critical path determination,
and probability beta distribution of Mean (Te), Standard deviation (σ), variance (ν), and the
probability of activity completion (Z) of activity and project.
4. Equipment
 Desktop or Laptop computer with internet connection.
 MS software textbook
5. Practice contents/Activities/Safety

6. Clean-up
 After finishing practice, all tools and equipment utilized in the practice shall be returned to
the proper storage place.
 Clean up the practice shop.

7. Independent practice/Follow-up activities


Learning through assignment

8. Review/Reflection
Review the outcome of the practice, improvement measures, and previously reflected opinions.

40
Chapter 2: Construction Schedules

2.1 Introduction

In chapter one, the AOA and AON networks were presented, also the time and cost of individual
activities based were calculated. However, we do not know how long the total project duration is.
Also, we need to evaluate the early and late times at which activities start and finish. In addition,
since real-life projects involve hundreds of activities, it is important to identify the group of
critical activities so that special care is taken to make sure they are not delayed. All these
statements are the basic objectives of the scheduling process, which adds a time dimension to the
planning process. In other words, we can briefly state that: Scheduling = Planning + Time.
Scheduling is the determination of the timing of the activities comprising the project to enable
managers to execute the project on time. The project scheduling id sued for:
 Knowing the activity's timing and the project completion time.
 Having resources available on-site at the correct time.
 Making correction actions if the schedule shows that the plan will result in late completion.
 Assessing the value of penalties on project late completion.
 Determining the project cash flow.
 Evaluating the effect of change orders on the project completion time.
 Determining the value of project delay and the responsible parties.

2.2. Importance of Project Scheduling

The success of a project heavily depends on how effective the scheduling is and how tightly the
project can be controlled. Poor scheduling can easily result in completion delays and cost
overruns. These, in turn, result in claims and counter-claims, disagreements, and disputes.

41
The appropriate method should be used in planning and scheduling a project; management
decisions should not depend on experience and intuition alone. Preparing a project schedule is
extremely important due to several reasons.
The project schedule is the basis of extracting several other schedules and information. Even at
the tendering stage, project schedules need to be prepared although there may not be required
information for preparing project schedules. Project schedules are helpful not only for a
contractor but for the owners as well.
For the contractors, project schedules help in the timely mobilization of the required resources
and help in the identification of bottlenecks in the early stages of the project duration. They also
act as a reference for comparison with actual progress, cost of construction, and profit margin at
any given time during monitoring.
For the owner also, the project schedule helps in understanding the requirement of finance to be
mobilized. Also, with the project schedule in place, it is easy for the owner/client to monitor the
project accordingly. The detailed steps carried out to prepare a project schedule by a contractor
are listed below:
 Identify and study the scope of work from the contract document (tender drawings, bill of
quantities, terms and conditions, etc.). Contractors normally do not have access to all the
details needed to prepare an exhaustive schedule at the beginning, but that should not be an
excuse for the non-preparation of the schedule for the project. Contractors should try to
extract as much information as possible, even by unofficially meeting the owner’s
representative and architects for the project. Owners try not to give information based on
incomplete documentation on their part, for the fear of getting penalized at a later date.
 Decide on the construction methodology for completing the project within the given
schedule. The schedule specified in the tender document mostly governs the construction
methodology adopted by the contractor.
 The total project scope is finally brought down into work packages and activities. This
breakdown into smaller activities helps the planner in estimating resource requirements and
duration for completing the activities. The productivity norms, as well as the experience of
the planner, are utilized to arrive at the estimates of duration and resource requirements for
these activities.
 Using the relationships existing among different activities in the form of a network for the

42
project is prepared. The network can be prepared by using either PERT/CPM or the
precedence network.
 From the network, a week-wise or month-wise project schedule is derived.
 In large projects involving several subprojects, a master project schedule is prepared in
addition to preparing schedules for each of the subprojects.
 Finally, it should be kept in mind that these schedules need to be revised as and when any
new information is available or at a fixed interval such as fortnightly or monthly.

2.2. Project schedule preparation steps

To draw up a comprehensive schedule, the steps mentioned below should be followed:

Identification and classification of tasks: This is the first critical step wherein all individual
tasks or activities required to complete the project are listed.

Defining duration of tasks: This is essentially the first iteration to estimate the time durations
involved in completing the different tasks. All available information should be used to make an
accurate estimate of the duration of each task and depending upon the nature of the project and
the activity, the estimated time may be given in hours, days, or months.

Task sequence: Activities in a construction project may appear to be independent, but often
depend upon each other. For example, the painting of a house is done by an independent agency,
but the work can commence only after the plastering of the walls is completed. Continuing with
the example of house construction, there is obvious interdependence in electrical, plumbing, and
finishing works. Although it appears trivial, such issues of ‘inherent’ logic and ‘predecessor’ and
‘successor’ tasks should be sorted out at the outset to reduce the possibility of difficulties later
on. In larger projects, it also helps to identify tasks that can be completed or carried out at the
same time. Such tasks, or others that require careful handling, maybe ‘flagged’ at the outset and
special attention is paid to their progress and any possible slippage. To that extent, it may be
noted that an easy-to-understand depiction of relationships between different activities is an

43
important function of the project planner. Different methods of such representations, such as a
network diagram, are available and discussed in greater detail elsewhere.

Create a calendar of events: Once the list of activities, their interdependence, and the time
involved in completing them is known from the steps above, a calendar of events can be easily
created by highlighting the milestone events.

2.4. Other schedules derived from project schedules

From a project schedule, the following schedules can be prepared:

1. Prepare invoice schedule


2. Prepare cash inflow and cash outflow schedule
3. Prepare staff schedule
4. Prepare labor schedule
5. Prepare the material schedule
6. Prepare a specialized subcontractor schedule
7. Plant and equipment schedule
8. Prepare a working capital schedule
9. Estimation of direct and indirect costs

2.4.1. Preparing Invoice Schedule

An invoice is the amount of money realized by executing a particular activity or work.


Accordingly, invoice value is the product of the quantities of work done each month for a
particular activity and the corresponding rates. The rates are nothing but the quoted rates by the
contractor. It must be kept in mind that the invoice schedule is the estimated billing amount on a
monthly or weekly basis. The actual invoice may be exactly as per the planned one or it may be
ahead or behind the estimated figure. This schedule also acts as the basis for monitoring the
progress of invoices. There is a limitation, however, in using this schedule to monitor the

44
physical progress of a project, as completion dates of individual activities/milestones cannot be
indicated herein.

Table 2.1 Distribution of quantity planned to be executed activity-wise every


month

Item description M1 M2 M3 M4 M5 M6 M7 M8 M9 M10


Earthwork all soil 250 750 1,250 1,250 500
Concrete works 200 400 500 300 200
Formwork 1,400 2,800 3,500 2,100 1,400
Reinforcement works 25 50 62.5 37.5 25
Road works 2,000 3,000 4,000 4,000 4,000 2,000 1,000

2.4.2. Schedule of Milestone Events

Milestone events are very important in the proposed schedule of a project. Top management is
more concerned with the scheduled completion of a milestone event. Some examples of
milestone events are the completion of the foundation, completion of the superstructure, and
completion of sanitary and plumbing works. Thus, from the schedule of the project, the details of
the milestone

Table 2.2 Estimated bill value or invoice value every week for the case project

Item description M1 M2 M3 M4 M5 M6 M7 M8 M9 M10


Earthwork all soil 0.25 0.75 1.00 1.25 1.25 0.50
Concrete works 8.00 16.00 16.00 20.00 12.00 8.00
Formwork 4.20 8.40 8.40 10.50 6.30 4.20
Reinforcement works 11.25 22.50 22.50 28.125 16.87511.25

45
Road works 11.00 16.50 22.00 22.00 11.00 5.50
Total 0.25 24.2 47.9 59.15 76.34 57.64 45.45 11 5.5

2.4.3. Schedule of Plant and Equipment

In this schedule, the date of a particular plant and equipment is also mentioned. The release date
of a plant and equipment that is surplus at the project site is mentioned as well. Finally, the time
required for doing pre-dispatch maintenance is mentioned (see table 2.3).

Table 2.3 Schedule of plant and equipment for the case project

No Requirements
NO. Plant and equipment description Remarks
required from up To
1 Earth rammer of capacity 8 tones 2 M6- M7 Required
2 Concrete mixer 500 l/5 m3 capacity 1 M2- M7
3 Builders hoist 2 M3- M7
4 Front-end dumper 1 M1- M10
5 Theodolite with optical plummet 1 M1- M10
6 Tilting/dumpy level 2 M1 -M10
7 Compression testing machine 100 t 1 M2- M7
8 Bar-bending machine 1 M2- M7
9 Bar-bending machine 1 M2- M7
10 Wood surface planner—horizontal 1 M2- M7
11 Water pump centrifugal multistage 1 M2-M10
12 Mobile diesel compressor 1 M2-M10

46
2.4.4. Schedule of Project Staff

Depending on the invoicing to be done per month, a month-wise schedule for project staff is
maintained (see Table 2.4). In this schedule, the date by which a particular staff is required is
also mentioned. The release date of a particular category of staff that is surplus at the project site
is to be mentioned as well. The number of staff is dependent on the nature of the activity, the
quantum of work, the working hours adopted by the organization, the proportion of work that the
main contractor is planning to subcontract, etc.

Table 2.4 Schedule of project staff

NO Plant and equipment Nos Requirements Remarks


description required From UP To
1 Project manager 1 M1- M10
2 Resident engineer 1 M1- M10
3 Civil engineer 1 M2- M10
4 Jr engineer-electrical 1 M6- M10
5 Jr engineer-CIVIL 1 M2- M7 For concreting activities
6 Jr engineer-CIVIL 1 M6 –M7 For road works
7 Forman 1 M2- M7 For formwork activities
8 Forman 1 M2- M7 For concreting activities
9 Accountant 1 M1- M10
10 Timekeeper 1 M1- M10

2.4.5. Schedule of Labour Requirement

From the split-up of quantity, labor requirement is calculated. The requirement of labor is
estimated for each activity. The number of laborers is dependent on the quantity involved in an
activity and the productivity of laborers. While the productivity aspect has been dealt with

47
separately elsewhere in the book, it has been referred to for illustration here. Thus, in our
example, the quantity of earthwork involved in month 1 is 833.33 m3. If the productivity of
laborers for this work is assumed to be 3 man-hours per m3, the total man-hours required for this
activity alone would be 2,500 man-hours.

Table 2.5 Schedule of Labour

NO. Description (labor Total man- M M M M M M


category) Total man months 1 2 3 4 5 6

1 Unskilled

2 Semi-skilled

3 Skilled

2.4.6 Schedule of Materials Requirement

Based on the quantity of a particular item, the type and quantity of materials to be purchased are
established. The schedule helps the project manager to assess the timing at which a particular
material is desired for the project. The material requirement as a function of time shown month-
wise in the schedule (could be any other measurement of duration such as a week) is
predetermined based on the bar chart of the project. The sample schedule of materials
requirements is provided in Table 7.10 for the case project.

For example, if the material for the item brickwork is required to be calculated, we find that the
number of bricks required in months 3, 4, 5, 6, 7, 8, and 9 is 85,000, 127,500, 170,000, 170,000,
170,000, 85,000 and 42,500, respectively. This is on an assumption that 500 bricks are needed
for 1 m3 of brickwork. Such predetermined schedules help a project manager in taking the

48
procurement action well in time, thus realizing advantages such as discounts from the material
supplier. If, on the other hand, the material is procured on an emergency basis, chances are that
one may compromise on quality besides getting the material at higher prices.

Table 2.6 Schedule of materials required for the case project

NO. Description Unit Qty M1 M2 M3 M4 M5 M6 Remark


1 Cement Bags 17,8251 10 20 20 25 15 10
2 Sand- coarse, fine, and M3 3010
filling
3 Aggregates- 40, 20, M3 1854
12/10, 6 mm, and
above 40 mm for road
4 Bricks 75 class Each 850000
designation
5 Reinforcement bar T 258

2.4.7. Schedule of Specialized Agencies

In a project, a contractor may not be in a position to do all the activities economically. Further,
for some of the activities in a project, he may not have the requisite expertise. In such cases, the
contractor enters into a contractual arrangement with some specialized agencies called
subcontractors or specialty contractors. As a first step, a list of activities for which the contractor
wants to engage subcontractors is prepared. The scheduled dates of start and completion of such
activities are noted. Let us assume that the contractor wants to engage subcontractors for
activities such as ‘road works’. Looking at the schedule, the contractor would know the latest
time to start the process of finalization of a contractor for a particular activity.

Table 2.7 Schedule of specialized agencies


49
NO. Description Unit Qty Date of Date of Remark
start completion
1 Plastering work M6 M9
2 Road works M5 M8

Other types of schedules can be prepared based on the project schedule. These are the schedule
of direct costs, a schedule of overheads, a schedule of cash inflows and outflows, etc. Typical
formats for all these schedules have been given in the figures below. These have been described
in appropriate places in detail.

Table 2.7 Schedule of Direct Costs

NO. Description Unit Qty Unit Total M1 M2 … M6 Remark


cost cost
1
2
Total

Table 2.8 Schedule of Overheads

NO. Description Total Amount M1 M2 … M6 Remark


1 Salaries for staff
2 Conveyance at site
3 Total of overheads
3 Overheads cumulative total
4 Direct costs (cumulative)
5 Invoicing (cumulative)
6 Site contribution

50
Total

Table 2.9 Schedule of Cash Inflow

NO. Description Total M1 M2 … M6 Remark


Amount .
1 Invoicing
2 Progressive payments (A)
3 Advance cumulative (B)
4 Gross amount due (C) 5 (A 1 B)

Table 2.10 Schedule of Cash Outflow

NO. Description Total M1 M2 … M6 Remark


Amount
1 Departmental labour
2 Subcontractor labor (b)
3 Specialized contractor (c)
4 Material purchase (d)

51
2.5. Work breakdown structure

The WBS is described as a hierarchical structure that is designed to logically subdivide all the
work elements of the project into a graphical presentation. The full scope of work for the project
is placed at the top of the diagram, and then sub-divided into smaller elements of work at each
lower level of the breakdown. At the lowest level of the WBS, the elements of work are called a
work package. A list of the project’s activities is developed from the work packages.
Effective use of the WBS will outline the scope of the project and the responsibility for each
work package. There is not necessarily a right or wrong structure because what may be an
excellent fit for one discipline may be an awkward burden for another.
A WBS is a deliverable-oriented grouping of project components that organizes and defines the
total scope of the project. Any work not in the WBS is outside the scope of the project. It is used
to develop or confirm a common understanding of the project scope. Each descending level
within the WBS shows an increasingly detailed description of the project deliverables.
A WBS is normally presented in the form of a chart. The WBS should not, however, be confused
with the method of presentation. Simply drawing an unstructured activity list in chart form does
not make it a WBS.
Each item in the WBS is usually assigned a unique identifier, which is used to provide a structure
for a hierarchical summation of costs and resources. The items at the lowest level of the WBS
may be referred to as work packages, particularly in organizations that follow earned value
management practices. These packages can be decomposed further, if necessary, into activities.
The activities can then be sequenced and estimated to create the project schedule.
Effective use of the WBS will outline the scope of the project and the responsibility for each
work package. There is not necessarily a right or wrong structure because what may be an
excellent fit for one discipline may be an awkward burden for another. To visualize the WBS,
consider Figure 2.1 which shows a house construction project.
A WBS dictionary will typically include
 Work package descriptions
 other planning information
 schedule dates
 cost budgets

52
 staff assignments

House

Civil Electrical
Plumbing

Foundation Wall/Roof Wiring Fitting


Piping H/C water

Figure 2.1: WBS and their description

As shown in Figure 2.1, level 1 represents the full scope of work for the house. In level 2, the
project is sub-divided into its three main trades, and in level 3 each trade is sub-divided into
specific work packages. Figure 2.2 shows another example of a more detailed.

Level 1
Gas development project

Level 2 Recovery unit 300 Process unit 400

Separation and
Level 3 Train 2 Gas treating
Train 1 stabilization

Level 4 Civil Piping


Instrumentati Structural steel
on
Pipe fabrication
Level 5

Figure 2.2: Five levels of WBS

53
Level 1: The entire project.
Level 2: Independent areas.
Level 3: Physically identified sections contained in level 2, reflect construction strategy.
Level 4: Disciplines set up a schedule.
Level 5: Master schedule activities, quantity, and duration.

2.5.1. WBS coding

A project code system provides the framework for project planning and control in which each
work package in a WBS is given a unique code that is used in project planning and control. The
coding system provides a comprehensive checklist of all items of work that can be found in a
specific type of construction. Also, it provides uniformity, transfer & comparison of information
among projects. An example of this coding system is the Master Format (Figure 2.3.) which was
developed through a joint effort of 8 industry & professional associations including the
Construction Specifications Institute (CSI); and Construction Specifications Canada (CSC).
Figure 2.4 shows an example of the coding system using a standardized system such as the
Master Format. The Master format is divided into 16 divisions as follows:
1. General Requirements.
2. Site work.
3. Concrete.
4. Masonry.
5. Metals.
6. Woods & Plastics.
7. Thermal & Moisture Protection.
8. Doors & Windows.
9. Finishes.
10. Specialties.
11. Equipment
12. Furnishings.
13. Special Construction.
14. Conveying Systems.

54
15. Mechanical.
16. Electrical.

Figure 2.3.: Master Format coding system

55
Figure 2.4.: An example of an activity coding system

Figure 2.5.: An example of an activity coding system

2.6. Bar chart/ Gantt chart

56
Bar chart/Gantt chart Developed by Henry Gantt where each activity is represented by a bar and
the Bar shows start/finish times, as well as Progress, which means that it is the simplest form
and named after the person who first used it. A bar chart is a time versus activity chart in which
activities are plotted using their early or late times.
Advantages of Bar chart:
 Useful to report information to Top Management;
 A simple format and readily understood at all levels of management; and
 It can provide a quick, visual overview of a project in a convenient way to monitor job
progress, schedule equipment, and crews, and record project advancement.
Disadvantages of Bar chart:
 Interdependencies among activities are difficult to show. The bar chart itself doesn’t
provide a basis for ascertaining which activities are critical and which are floaters.
 It is not an adequate planning and scheduling tool because it doesn’t portray a detailed,
integrated, and complete plan of operations.
 Can't tell what will be effect the delay today will have on the timing of future activities.

Figure 2.6.: Bar chart with activity and Time

57
An early bar chart is drawn using the ES times of activities, while the late bar chart is drawn
using the LS times. The bar chart representation shows various details. Float times of activities
and critical activities can be shown in different colors, or bold borders, as shown in Figures 2.7
and 2.8. The bar chart can also be used for accumulating total daily resources and/or costs, as
shown in the bottom part of Figures 2.9 and 2.10. In this figure, the numbers on each activity
represent the number of laborers needed.

Figure 2.7: Early bar chat

58
Figure 2.8: Late bar chart

Figure 2.9: Using a bar chart to accumulate resources

Figure 2.10: Using a bar chart to accumulate resources

59
2.7. Network scheduling

A network is a model consisting of nodes and links connecting between these nodes. And is a
method of scheduling activities by joining them in a series of interconnected links, which reflect
the relationships of activities by the planner. There are many basic Assumptions in network
scheduling. Among them:
 The project can be broken down into a group of activities.
 Each activity can be assigned a duration.
 The logical relationship among activities is known and fixed in the network chains.

2.7.1. Rules for Preparing the Network Diagram

 No activity can start before the preceding activity is finalized.


 There is only one start and finish for an activity.
 No activity leads back and forms a loop.
 The logical precedence, concurrent and subsequent activities must be developed.
 Each activity is represented by one and only one arrow in the network:
 This implies that no single activity can be represented twice in the network.
 This is to be distinguished from the case where one activity is broken into segments.
Apart from the rules listed in the above lists, a few important suggestions for drawing good
networks are:
 Try to avoid arrows that cross each other.
 Use straight arrows.
 Do not attempt to represent the duration of activity by arrow length.
 Use arrows from left to right (or right to left). Avoid mixing two directions, vertical and
standing arrows may be used if necessary.

60
2.7.2. Dummy Activity

A Dummy activity is established only to show a relationship among activities. A dummy


activity is assumed to have a duration of ZERO time units or resources and it is introduced on
the network when it is necessary and is denoted by a broken arrow as shown in Figure 2.11
below.

Figure 2.11: Using a bar chart to accumulate resources

2.7.3. Presentation of Networks and Precedence Diagram Method (PDM)

As seen in the Drawing Project Network (chapter one), and network schedule (chapter two), the
network is a graphical display of the proposed plan which Shows the job activities and their
order of sequence (logic) in pictorial form respectively. Generally, there are two methods of
network presentation; they are Arrow or Activity on Arrow and Precedence Diagram.

Table 2.11 Presentation of Networks consists

Scheme Representation

61
Activity Relation

Arrow diag. Link Arrow Node (Circle)

Precedence diag. Node (Circle) Link arrow

Figure 2.12: Arrow Vs. Precedence diagram

2.7.4. Common Errors in Drawing Networks

Three types of errors are most commonly observed while drawing network diagrams:
 Dangling
 Looping (or Cycling)
 Redundancy

Dangling: To disconnect an activity before the completion of all activities in a network diagram
is known as dangling.
As shown in the figure below, activities (b - c) and (d - e) is not the last activities in the network.
So the diagram is wrong and indicates the error of dangling.

62
Figure 2.13: Dangling network error

Looping (or cycling): Looping errors are also known as cycling errors in a network diagram.
Drawing an endless loop in a network is known as an error of looping as shown in the following
figure.

Figure 2.14: Looping network error

Redundancy: Unnecessarily inserting the dummy activity in a network diagram is known as


the error of redundancy as shown in the following diagram.

Figure 2.15: Dangling network error

63
2.7.5 Numbering the events of a network activities

As per Fulkerson's rule, the Procedure for numbering the events is:
Step1: Number the start or initial event as 0 or 1.
Step2: From event 0 or 1, strike off all outgoing activities. This would have made one or more
events initial events (an event that does not have incoming activities). Number that event
as 2.
Step3: Repeat step 2 for event 2, event 3, and till the end event. The end event must have the
highest number.

2.8. The Critical Path Method

The most widely used scheduling technique is the critical path method (CPM) for scheduling.
This method calculates the minimum completion time for a project along with the possible start
and finish times for the project activities. Many texts and managers regard critical path
scheduling as the only usable and practical scheduling procedure. Computer programs and
algorithms for critical path scheduling are widely available and can efficiently handle projects
with thousands of activities.
The critical path itself represents the set or sequence of activities that will take the longest time
to complete. The duration of the critical path is the sum of the activities' durations along the
path. Thus, the critical path can be defined as the longest possible path through the "network" of
project activities. The duration of the critical path represents the minimum time required to
complete a project. Any delays along the critical path would imply that additional time would
be required to complete the project.
There may be more than one critical path among all the project activities, so completion of the
entire project could be delayed by delaying activities along with any one of the critical paths.
For example, a project consisting of two activities performed in parallel that each requires three
days would have each activity critical for completion in three days. Formally, critical path

64
scheduling assumes that a project has been divided into activities of fixed duration and well-
defined predecessor relationships. A predecessor relationship implies that one activity must
come before another in the schedule.
The CPM is a systematic scheduling method for a project network and involves four main steps:
 A forward path to determine activities early-start times;
 A backward path to determine activities late-finish times;
 Float calculations; and
 Identifying critical activities.

2.8.1. Calculations for the Critical Path Method

The inputs to network scheduling of any project are simply the AOA or the AON networks with
the individual activity duration defined. The network scheduling process for AOA and AON
networks, however, is different. To demonstrate these two techniques, let’s consider a simple 5-
activity project, with activity A at the start, followed by three parallel activities B, C, and D;
which are then succeeded by activity E. The AOA or the AON networks of this example are
presented in Figure 2.16. Detailed analyses of these AOA or the AON networks are presented in
the following subsections. It is noted that the example at hand involves only simple finish-to-
start relationships among activities.

65
Figure 2.16: AOA or the AON networks

2.8.2. Critical Path Scheduling Algorithms

A. Earliest Event Time Algorithm


Step 1: Let E (0)=0.
Step 2: For j= 1, 2, 3, …n (where n is the last event)
let E(j)= maximum{E(i)+Dij}

B. Latest Event Time Algorithm


Step 1: Let L(n)=the required time of project completion.
Note: L(n) must equal or exceed E(n).
Step 2: For i= n-1, n-2, …0 (where n is the last event)
let, L(i)= minimum{L(j)-Dij} where: the minimum is computed over all activities
(i,j) that have i as the starting event.
C. Earliest Time Calculations/ Forward Path determination
The forward-path determines the early-start times of activities. The forward-path proceeds from
the most left node in the network and moves to the right, putting the calculations inside the
shaded boxes to the left. The earliest event time algorithm computes the earliest possible time,
E(i), at which each event, i, in the network can occur.
The earliest event times are computed as the maximum of the earliest start times plus activity
durations of each of the activities immediately preceding an event. The earliest start time of
each activity (i,j) is equal to the earliest possible time for the preceding event E(i):

ES(i,j) = E(i)

The earliest finish time of each activity (i,j) is calculated as:

EF(i,j) = E(i) + Dij

That node is also a point at which some activities start (tail arrows of successor activities).
Certainly, all successor activities can start only after the latest predecessor is finished. Therefore,

66
for the forward path to determine the early-start (ES) time of an activity, we have to look at the
head arrows coming into the start node of the activity. We then have to set the activity ES time as
the latest finish time of all predecessors.

Figure 2.17: Forward Path preparation

Example 2.1

Precedence Network Calculations

E Duration E ES: Earliest Start


S . F EF: Earliest Finish
Activity ID LS: Latest Start
L TF L LF: Latest Finish
S F

Finish to Start:

Late Finish A 10 5 15 28 5 33 EST B


= 45-13 13 = 15 + 13
A B
= 32 27 32 45 50 = 28

67
Start to Start:

10
LS of A 10 5 15 20 5 25 EST B
= 45-10 A B = 10 + 10
=35 35 40 = 20
45 50

Finish to Finish:

8
Late Finish A 10 5 15 18 5 23 EFT of B
= 50-8 B = 15 + 8
A
=42 = 23
37 42 45 50

Start to Finish:

16
LST of A 10 5 15 21 5 26 EFT of B
= 50-16 A B = 10 + 16
=34 34 39 45 50 = 26

Figure 2.18: Forward path calculations in AON networks

Example 2.2
From the given data above (figure 2.17) and its activity duration, we can determine the earliest
time using the forward path.
Solution
Now, move forward to successor nodes 5, 7, and 9. However, since node 9 is linked to nodes 5
and 9 by dummy activities, we begin with nodes 5 and 7. Node 5 receives one head arrow from
its predecessor activity B, we evaluate the EF time of B as 6 (ES (3) + d (3)). Successor
activities to node 5, therefore, can have an ES time of 6. Similarly, the ES time at node 7 is
calculated as time 9.

68
Figure 2.19: Forward path calculations in AOA networks

Moving to node 9, the EF times of its 3 predecessors (d1, C, and d2) are times 6, 7, and 9,
respectively. Accordingly, the ES time of successor activities is the largest value 9. Notice that
only the largest EF value of predecessor activities is used to calculate the ES of successor
activities and all other values are not used. As such, only ES values can be directly read from
the calculations in Figure 2.18. EF values, on the other hand, can be calculated as EF = ES + d.
The last node (11) receives one head arrow, activity E which has an ES value of 9. The EF time
of activity E, therefore =9 + 5 = time 14. Since node 11 is the last node, the EF of this node
becomes the end of the project, reaching a total project duration of 14 days.

Example 2.3
From the given data below (figure 2.20) and its activity duration, we can determine the earliest
time using the forward path.

69
Figure 2.20: Forward Path determination

D. Latest Time Calculations/backward path determination

The latest event time algorithm computes the latest possible time, L(j), at which each event, j, in
the network can occur, given the desired completion time of the project, L(n) for the last event n.
The latest finish time consistent with the completion of the project in the desired time frame of
L(n) for each activity (i,j) is equal to the latest possible time L(j) for the succeeding event:
LF(i,j) = L(j)

The latest start time of each activity (i,j) can be calculated as:
LS(i,j) = L(j) - Dij

The backward path determines the late-finish (LF) times of activities by proceeding backward
from the end node to the starting node of the AOA network. We put the LF values in the right
side boxes adjacent to the nodes, as shown in Figure 2.20 above. For example 2.3 at hand:

70
Solution

Figure 2.21: Backward Path determination in AOA networks

Start from the last node of the network (node 11) and we transfer the early-finish value from the
left box to the late-finish (LF) value at the right side box.
Then, move backward to node 9 which has only one tail arrow of activity E. With the LF time of
E being time 14, its LS time becomes LS = LF - d = 14 – 5 = time 9. At node 9, therefore, time 9
becomes the LF time of the predecessor activities of this node.

Moving backward to predecessor nodes 5, and 7. Node 5 has one tail arrow of the dummy
activity d1, and as such, the LF time value to be used at node 5 becomes 9. Similarly, the LF
time value of node 7 becomes 9.
Moving to node 3, we evaluate the LS time of its 3 successor activities B, C, and D as 6, 5, and
3, respectively. The LF time at node 3, therefore, becomes the smallest value 3. With other LS
values not used, the values in the calculation boxes, as such, directly show the LF times of
activities. LS times can be calculated as LS = LF – d.
Now, proceed to the first node in the network (node 1). It connects to one tail arrow of activity
A. The LS time of A, therefore, is LS = LF – d = 3 – 3 = 0, a necessary check to ensure the
correctness of the calculation.

71
Example 2.4
From the given data below and its activity duration, can determine the latest time using the
backward path method.

Figure 2.22: Backward Path determination

2.8.3. Identifying the Critical Activities

Activities with zero total floats mean that they have to be constructed right at their scheduled
times, without delays. These activities are considered to be critical. They deserve the special
attention of the project manager because any delay in critical activities causes a delay in the
project duration.
One interesting observation in the results of CPM analysis is that critical activities form a
continuous path of the critical activities that span from the beginning to the end of the network.
In our example, activities A, D, and E (excluding dummy activities) are critical and the critical
path is indicated by bold lines in Figure 2.21 and Critical activities can also be easily determined
as the ones having zero float times, activities A, D, and E. Notice that among the 3 paths in this
example (A-B-E; A-C-E; and A-D-E), the critical path is the longest one, an important

72
characteristic of the critical path. In real-life projects with many activities, more than one critical
path may be formed. By definition, the length of these critical paths is the same.
In addition, the earliest start and latest finish times of each event are useful pieces of information
for developing a project schedule. Events that have equal earliest and latest times, E(i)=L(i), lie
on the critical path(s) and activity is a critical activity if it satisfies all of the following
conditions:
E(i)=L(i)
E(j)=L(j)
E(i) + Dij =L(j)

2.9. Float Calculations

Once forward path and backward path calculations are complete, it is possible to analyze the
activity times. First, let's tabulate the information we have shown in Table 2.12. One important
aspect is Total-Float (TF) calculations, which determine the flexibility of an activity to be
delayed. Notice in Table 2.12 that some activities such as activity A have ES time = LS time, and
its EF time = LF time, indicating no slack time for the activity. Other activities such as B can
start early at time 3 and late at time 6, indicating a 3-day of total float. Float calculations can be
illustrated as shown in Figure 2.23 for any activity.

Table 2.12 Float and critical activity determination

Activity Durat Early Late Late Early Total Critical


ion Start (ES) Finish Start Finish Float Activity
(LF) (LS) (EF) (TF)
A 3 0 3 6 3 0 Yes
B 3 3 9 5 6 3 No
C 4 3 9 7 7 2 No
D 6 3 9 9 9 0 Yes
E 5 9 14 14 14 0 yes

73
The two ways of schedule of each activity using its activity times.
Total Float (TF) is determined by:
a). TF = LF – EF
b). TF= LS – ES and

Free Float (FF) is determined by:


Free Float (FF) = ETj – ETi – d or
FF = smallest ES (of succeeding activities) – EF (of current activity)

The free float defines the amount of time that an activity can be delayed without affecting any
succeeding activity. With free float available for an activity, a project manager knows that the
float can be used without changing the status of any non-critical activity to become critical.

Figure 2.23: Float calculations

2.10. Program Evaluation and Review Technique

Both CPM and PERT were introduced at approximately the same time and, despite their separate
origins. The PERT method shares many similarities with CPM. Both require that a project be
broken down into activities that could be presented in the form of a network diagram with their

74
sequential relationships of activities. Both require time estimates for each activity, which are
used in routine calculations to determine project duration and scheduling data for each activity.
CPM requires reasonably accurate knowledge of time and cost for each activity. However, the
duration of an activity cannot be accurately forecasted, and a degree of uncertainty exists.
Contrary to CPM, PERT introduces uncertainty into the estimates for activity and project
durations. It is well suited for those situations where there is either insufficient background
information to specify accurately time and cost or where project activities require research and
development.
In the original development of the PERT approach, AOA notations are used. However, AON
diagramming can be easily used alternatively. The method is based on the well-known “central
limit theorem”. The theorem states that: “Where a series of sequential independent activities lie
on the critical path of a network, the sum of the individual activity durations will be distributed
in an approximately normal fashion, regardless of the distribution of the individual activities
themselves. The mean of the distribution of the sum of the activity durations will be the sum of
the means of the individual activities and its variance will be the sum of the activities’
variances”. The primary assumptions of PERT can be summarized as follows:
 Any PERT path must have enough activities to make the central limit theorem valid.
 Any PERT path must have enough activities to make the central limit theorem valid.
 The mean of the distribution of the path with the greatest duration, from the initial node
to a given node, is given by the maximum mean of the duration distribution of the paths
entering the node.
 PERT critical path is long enough than any other path in the network.
PERT, unlike CPM, uses three-time estimates for each activity. These estimates of the activity
duration enable the expected meantime, as well as the standard deviation and variance, to be
derived mathematically. These duration estimates are:
Optimistic duration (a); an estimate of the minimum time required for an activity if
exceptionally good luck is experienced.
Most likely or modal time (m); is the time required if the activity is repeated several times
under essentially the same conditions.
Pessimistic duration (b); an estimate of the maximum time required if unusually bad luck is
experienced.

75
The use of these optimistic, most likely, and pessimistic estimates stems from the fact that these
are thought to be easier for managers to estimate subjectively. The formulas for calculating the
mean and variance are derived by assuming that the activity durations follow a probabilistic beta
distribution under a restrictive condition. The probability density function of beta distributions
for a random variable x is given by:

f(x) = k(x - a) α (x - b) β , a ≤ x ≤ b, α, β > −1

Where k is a constant which can be expressed in terms of α and β. Several beta distributions for
different sets of values of α and β are shown in figure 2.24.

Figure 2.24: Illustration of several Beta distributions

Using beta distribution, simple approximations are made for the activities’ meantime and their
standard deviation. Using the three times estimates, the expected mean time (te) is derived using
Eq. Te = a+ 4m +b per 6 Then, Te is used as the best available time approximation for the
activity in question. The standard deviation (σ) is given by Eq. σ = b-a per 6, and hence the
variance (ν) can be determined as ν = σ 2.

76
a+4 m+ b
t e=
To cote, 6
b−a
σ te=
6
ν=( σ te )2

The critical path calculations proceed as CPM. Associated with each duration in PERT,
however, is its standard deviation or its variance. The project duration is determined by
summing up the activity expected mean time along the critical path and thus will be an expected
mean duration. Since the activities on the critical path are independent of each other, central
limit theory gives the variance of the project duration as the sum of the individual variances of
these critical path activities.
Once the expected mean time for project duration (Te) and its standard deviation (σ) is
determined, it is possible to calculate the chance of meeting a specific project duration (TS).
Then normal probability tables are used to determine such chance using the Equation:

Z = Ts -Ta and then we can determine, that Ts = Z*σ+ Ta


σ
Where, Ts = the chance of meeting a specific project duration
The procedure for hand probability computations using PERT can be summarized in the
following steps:

 Make the usual forward and backward pass computations based on a single estimate
(mean) for each activity.
 Obtain estimates for a, m, and b for only critical activities. If necessary, adjust the length
of the critical path as dictated by the new te values based on a, m, and b.
 Compute the variance for event x (νX) by summing the variances for the critical activities
leading to event x.
 Compute Z using the above Equation and find the corresponding normal probability.

77
Example 2.5

A project having nine activities is shown in Table 2.13 Suppose that the project has very
uncertain activity time durations. As a result, project scheduling considering this uncertainty is
desired. So, determine the mean, standard division, variance, and probability of project
completion.

Table 2.13: Precedence relations and durations, and activity duration


estimation

Duration
Activity Description Predecessors Duration
a m b
A Site clearing - 4 3 4 5
B Removal of trees - 3 2 3 5
C General excavation A 8 6 8 10
D Grading general area A 7 5 7 8
E Excavation for trenches B, C 9 6 9 14
F Placing formwork and RFT for concrete B, C 12 10 12 14
G Installing sewer lines D, E 2 2 2 4
H Installing other utilities D, E 5 4 5 8
I Pouring concrete F, G 6 4 6 8

Solution

Table 2.14: Precedence relations and durations, and activity duration


estimation

Predeces Dura Duration Mean Standard


Tasks Description
sors tion a m b Deviation
A Site clearing - 4 3 4 5 4.0 0.33
B Removal of trees - 3 2 3 5 3.2 0.50
C General excavation A 8 6 8 10 8.0 0.67

78
D Grading general area A 7 5 7 8 6.8 0.50
E Excavation for trenches B, C 9 6 9 14 9.3 1.33
F Placing formwork and B, C 12 10 12 14 12.0 0.67
RFT for concrete
G Installing sewer lines D, E 2 2 2 4 2.3 0.33
H Installing other utilities D, E 5 4 5 8 5.3 0.67
I Pouring concrete F, G 6 4 6 8 6.0 0.67

Figure 2.25: PERT calculation

The critical path for this project ignoring uncertainty in activity durations consists of activities A,
C, F, and I. Applying the PERT analysis procedure suggests that the duration of the project
would be approximately normally distributed. The sum of the means for the critical activities is
4.0 + 8.0 + 12.0 + 6.0 = 30.0 days, and the sum of the variances is (0.33)2 + (0.67)2 + (0.67)2 +
(0.67)2 = 1.44 leading to a standard deviation of 1.2 days. And With normally distributed project
duration, the probability of meeting a project deadline can be computed using Equation Z= Ts-Ta
per σ For example, the probability of project completion within 35 days is:

Z = Ts -Ta = 35-30 = 4.167


σ 1.2

79
Where z is the standard normal distribution tabulated value of the cumulative standard
distribution, which can be determined from standard tables of the normal distribution that the
probability of completing the project in 35 days is 100%

Example 2.6
A road project has a list of tasks to be performed whose time estimates are given in Table 2.15,
as follows:

Table 2.15: Precedence relations and durations and activity duration


estimation

Activity Name T0 Tm Tp
1-2 A 4 6 8
1-3 B 2 3 10
1-4 C 6 8 16
2-4 D 1 2 3
3-4 E 6 7 8
3-5 F 6 7 4
4-6 G 3 5 7
4-7 H 4 11 12
5-7 I 2 4 6
6-7 J 2 9 0

A. Draw the project network


B. Construct a network diagram and calculate the time earliest, TE, and time Latest TL for
each activity.
C. Find the critical path.
D. Find the probability that the project is completed in 19 days. If the probability is less than
20%, find the probability of completing it in 24 days

80
Solution
A. Project network diagram

Figure 2.26: PERT calculation

B. The time expected for each activity is calculated using the formula :
Ta = t0 + 4tm + tp
6
= 4+4(6) +8 =36 = 6 to activity A, and the expected time is calculated for all tasks
6 6

C. The variance of activity time is calculated using the formula


σ2 = (tp – t0 ) 2
(6)2
σ2 = (8 – 4 ) 2 = 16 = 0.444 for A And compute the rest as per this formula
(6)2 36

Table 2.16: Precedence relations, durations mean, and standard division

Activity Name T0 T Tp Ta σ2
m
1-2 A 4 6 8 6 0.444
1-3 B 2 3 10 4 1.777
1-4 C 6 8 16 9 2.777
2-4 D 1 2 3 2 0.111
3-4 E 6 7 8 5 0.333

81
3-5 F 6 7 4 8 0.111
4-6 G 3 5 7 5 0.444
4-7 H 4 11 12 10 1.777
5-7 I 2 4 6 4 0.444
6-7 J 2 9 0 8 1.777

D. From the network diagram, the critical path identified is 1-4, 4-6, and 6-7, with a project
duration of 22 days.
E. The probability of completing the project within 19 days is given by, P (Z< Z0)
To find Z0 = Ts−TE /√ ∈ σ 2
Where Ts is the probability of completing the project within the scheduled time and, TE is the
expected time of project completion:

Z = 24 - 22
√ 2.777+0.444 +1.777 ¿
= -3/5 = -1.3416 days
It is known, P (Z < Z0) = Y (-1.3416) and,
From the normal negative tables, Y (-1.3416) = 0.0901
= 0.0901 and then times 100%
= 9.01%
Thus, the probability of completing the R & D project in 19 days is 9.01%. Since the probability
of completing the project in 19 days is less than 20%, we find the probability of completing it in
24 days.
Z = 24 - 22
√ 2.777+0.444 +1.777
= 2
√5
= 0.8944 days, and
P (Z < Z0) = Y (0.8944)
From the normal positive Table, Y (0.8944) = 0.8133
= (0.8133) X100%
= 81.33%

82
Assessments

1. Practical

Week 3: Practice

NO Task Name Duration Predecessors


1 Concrete Work 59 days
2 C-5 Lean Concrete 3 days
3 C-5 under footing 1 day
4 C-5 under the grade beam 1 day 3
5 C-5 under masonry 1 day 4
6 C-25 Concrete 12 days
7 C-25 footing 1 day 3
8 C-25 grade beam 3 days 4
9 C-25 10 cm thick ground floor slab 4 days 4
10 C-25 entrance steps 2 days 5
11 C-25 ramp 2 days 5
12 C-30 Concrete 1 day
13 C-30 foundation column 1 day 7
14 Form Work 16 days
15 FW for footing 4 days 20
16 FW for foundation column 4 days 21
17 FW for grade beam 6 days 22
18 FW for entrance steps 2 days 24
19 Rebar Work 18 days
20 Rebar for footing 3 days 3

83
21 Rebar for foundation column 3 days 20
22 Rebar for grade beam 3 days 21
23 Rebar for ground floor slab 3 days 22
24 Rebar for entrance steps 3 days 23
25 Rebar for ramp 3 days 23
26 Expansion Joint 9days
27 30mm thick and 15 cm high expansion joint 6 days 8

Having the task given from the above table creating a project schedule using MS Project
Specifically:
A. Open the MS project new sheet and save it by your Name
B. Make the auto-schedule and fix the fixed duration
C. Make project starting date 9/10/2022
D. Change working time and make half of Saturday a working day schedule the
resources working and nonworking time
E. Make concrete a summary task for all tasks
F. Make all bold task summary task
G. Create a resource list for the above tasks

Week 4: Practice

Revision of practice 3

NO Task Name Duration Predecessors


1 Concrete Work 59 days
2 C-5 Lean Concrete 3 days
3 C-5 under footing 1 day
4 C-5 under the grade beam 1 day 3
5 C-5 under masonry 1 day 4
6 C-25 Concrete 12 days
7 C-25 footing 1 day 3
8 C-25 grade beam 3 days 4
9 C-25 10 cm thick ground floor slab 4 days 4

84
10 C-25 entrance steps 2 days 5
11 C-25 ramp 2 days 5
12 C-30 Concrete 1 day
13 C-30 foundation column 1 day 7
14 Form Work 16 days
15 FW for footing 4 days 20
16 FW for foundation column 4 days 21
17 FW for grade beam 6 days 22
18 FW for entrance steps 2 days 24
19 Rebar Work 18 days
20 Rebar for footing 3 days 3
21 Rebar for foundation column 3 days 20
22 Rebar for grade beam 3 days 21
23 Rebar for ground floor slab 3 days 22
24 Rebar for entrance steps 3 days 23
25 Rebar for ramp 3 days 23
26 Expansion Joint 9days
27 30mm thick and 15 cm high expansion joint 7 days 8

Having the task given from the above table creating a project scheduling using Ms. Project.
Specifically:
H. Open the MS project new sheet and save it by your Name
I. Make the auto-schedule and fix the fixed duration
J. Make project starting date 9/10/2022
K. Change working time and make half of Saturday a working day schedule the
resources working and nonworking time
L. Make concrete a summary task for all tasks
M. Make all bold task summary task
N. Create a resource list for the above tasks
O. Assign the list of resources for each task
P. In addition, Assign the duration you enter for the task and Take your
assumptions and locally used standards to determine the duration

85
Week 5 : Practice:

Repeat of practice 4 and in addition to, Link b/n the task and its predecessors

Week 6 : Practice

NO Task Name Duration Predecessors


1 Concrete Work 49 days
2 C-5 Lean Concrete 30 days
3 C-5 under footing 1 day
4 C-5 under the grade beam 1 day 3
5 C-5 under masonry 1 day 4
6 C-25 Concrete 41 days
7 C-25 footing 1 day 3
8 C-25 grade beam 3 days 4
9 C-25 10 cm thick ground floor slab 4 days 4
10 C-25 entrance steps 2 days 5
11 C-25 ramp 2 days 5
12 C-30 Concrete 1 day
13 C-30 foundation column 1 day 7
14 Form Work 40 days
15 FW for footing 4 days 20
16 FW for foundation column 4 days 21
17 FW for grade beam 6 days 22
18 FW for entrance steps 2 days 24
19 Rebar Work 46 days
20 Rebar for footing 3 days 3
21 Rebar for foundation column 3 days 20
22 Rebar for grade beam 3 days 21
23 Rebar for ground floor slab 3 days 22
24 Rebar for entrance steps 3 days 23
25 Rebar for ramp 3 days 23
26 Expansion Joint 6 days
27 30mm thick and 15 cm high expansion joint 8 days 8

Having the task given from the above table and you did some tasks in the previous creating a
project scheduling using Ms. Project. Specifically:
A. Open the MS project new sheet and save it by your Name

86
B. Make the auto-schedule and fix the fixed duration
C. Make project starting date 9/10/2022
D. Change working time and make half of Saturday a working day schedule the resources
working and nonworking time
E. Make concrete a summary task for all tasks
F. Make all bold task summary task
G. Create a resource list for the above tasks
H. Assign the list of resources for each task
I. Link b/n the task and its predecessors
J. Assign the duration you enter for the task and Take your assumptions and locally used
standards to determine the duration
K. Include recurring tasks, milestones, and notes.

Week 7: Practice:

Repeat week 6 practice

Having the task given from the above table creating a project scheduling using Ms. Project.
Specifically:
a. Open the MS project new sheet and save it by your Name
b. Make the auto-schedule and fix the fixed duration
c. Make project starting date 9/10/2022
d. Change working time and make half of Saturday a working day schedule the
resources working and nonworking time
e. Make concrete a summary task for all tasks
f. Make all bold task summary task
g. Create a resource list for the above tasks
h. Assign the list of resources for each task
i. Link b/n the task and its predecessors
j. assign the duration you enter for the task

87
Week 9: Practice

Having the data on week 7 done on the project properties:

 Give your project title- MS Project

 Author-Your name

 Set the project start date to be -August 12, 2023. Project ‘scheduled from start
date’.

 Adjust the calendar to: ‘Nigth shift’.

 Set September 28, 2023, to be a ‘Holiday’.

 Give the name of your calendar to be -- ‘Mid calendar’.

88
2. Theory

Objective type of question

Type I: State True (T) or False (F):


1. The critical activities can be determined easily when using the bar chart.
2. The network must have definite points of beginning and end.
3. The network must be continuous from start to end.
4. There are no dummy activities in the arrow networks,
5. A forward pass is used to determine the late start and late finish times.
6. The time for completing a project is equal to the sum of the individual activity times.

Type II: Multiple choice


1. The free float is defined as:
A. The amount of time an activity can be delayed without affecting the following activity.
B. The amount of time an activity can be delayed without affecting the total project
duration.

2. Total float equals:


A. The amount of time an activity can be delayed without affecting the following activity.
B. The amount of time an activity can be delayed without affecting the total project duration.

89
Subjective Type Question

1. Derive the rate from the contractor’s perspective for one m3 of excavation of earth (a) in
normal soil where the depth of excavation varies from 0 m to 0.5 m, and (b) in a trench
excavation where the depth of excavation varies from 0 m to 1.5 m. analyze the rates for
manual excavation.
2. The total concrete quantity to be poured in a project is specified to be 100,000 m3 of M15
grade, and it is to be completed in 16 months, although the total project duration is 20
months. The contractor needs to estimate the cost of 1 m3 of M15 grade concrete.

3. For a project, the schedule of quantities to be executed month-wise along with the total
quantities is given in the Figure below. Compute the average monthly labor requirement and
the associated cost toward labor benefits.

4. For the project data in the Table answer the following questions:
a). Draw an AOA network of the project?
b). Perform forward path and backward path calculations?
c). what is the effect of delaying activity D by 3 days?

Activity Duration Immediate predecessor


A 2 -
B 6 A
C 3 A
D 1 B

90
E 6 B
F 3 C, D
G 2 E, F

5. Perform PDM calculations for the small project below and determine activity times.
Durations are shown in the activities.

6. For the activities listed in


the table below, draw the time-scaled diagram and mark the critical path. Determine the

Activity Duration Immediate predecessor


A 4 -
B 4 A
C 8 B
D 3 C
E 5 A
F 2 B, E
G 8 C, F
H 5 D, G
I 17 -
J 10 G, I

7. Recently, you were assigned to manage a project for your company. You have constructed a
network diagram depicting various activities in the project. In addition, you asked various
managers and subordinates to estimate the amount of time they would expect each activity to
take. Their responses (in days) were as follows:

Activity Duration
Optimistic (a) Most likely (m) Pessimistic (b)
91
A 4 7 10
B 8 10 14
C 3 6 7
D 2 4 6
E 7 8 9

a. Compute the mean and variance in time for each activity.


b. Determine the critical path and the expected length of the critical path.
c. Assume that the time required to complete a path is normally distributed. What is the
probability of completing the critical path in less than 17 days?
d. If you wanted to be at least 95% sure of completing the project on time, what schedule
durations would you quote?

92
Evaluation Sheet:

Evaluation Out of 5 Score Remark

Working Principle

Install MS. Project


Result Software
Evaluatio
Software
n
Program output

Applications
Tool Setup
Work Usage and storage
Evaluatio of the software
n Working of the
software
Time used (180
Time Min.) (1 per 10
min.)
Total /45

93
References

[1]. Kumar Neeraj Jha (2015), (Associate Professor Department of Civil Engineering), IIT,
Delhi Construction Project Management, Theory, and Practice. SECOND EDITION
[2]. Emad Elbeltagi, Ph.D., P.Eng., (2009), Professor of Construction Management
Structural Engineering Department, Faculty of Engineering, Mansoura University,
Lecture notes on construction Project management
[3]. Eldosouky, Adel I. (1996). “Principles of Construction Project Management.”
Mansoura University Press, Mansoura, Egypt.
[4]. Gould, Frederick E. (1997). “Managing the Construction Process: Estimating,
Scheduling, and Project Control.” Prentice-Hall Inc., New Gersy.
[5].Harris, Frank & McCaffer, Ronald. (1983). “Modern Construction Management.”
Granada Publishing, Great Britain.
[6]. Elgare Allah, Mohamed Ibrahim & Nawara, Jamal Mohamed. (1984). “Edarat
Almsharee’ Alhandaseah.” John Wiley & Sons Inc., NY. (This book is available in
Arabic).

94
Chapter 3: Resource Management

Lesson Plan (Resource Management)

1. Learning objectives
At the end of this chapter students/ trainees will be able to:
 Know the resource Management Process;
 Develop a better understanding of the Resource Management Techniques;
 Know the resource Management tools;
 Improve the level of understanding of how resource allocation, over-allocation, and
resource dependency;
 Inform about the importance of Human Resource Management;
 Develop the level of understanding of Human resources planning and the function of
human resource management in construction projects;
 Identify and know Construction Project Costs, labor requirements, and labor costs;
 Improve the level of understanding on how to Resource, Loading, Allocation, and
Aggregation;
 Develop the level of understanding towards Resource Levelling/smoothing, Heuristic
Procedure for Resource Smoothing.

2. Motivation
Outputs through Question and Answer; repetitive discussion and instruction of Resource
management, resource management process, resource management technique, resource
Management tools, resource allocation, over-allocation, resource dependency, the importance
of Human Resource Management, Human resources planning, and the function of human
resource management in construction projects, Construction Project Costs, labor requirement,
and labor cost, Resource, Loading, Aggregation, and Resource Leveling/smoothing,

95
3. Expectations or Outcomes
 Define the terms of project resource management, resource management process, resource
management technique, resource Management tools, resource allocation, over-allocation,
resource dependency, the importance of Human Resource Management, Human resources
planning, and the function of human resource management in construction projects,
Construction Project Costs, labor requirement, and labor cost, Resource, Loading,
Aggregation, and Resource Leveling/smoothing,
 Able to identify the project Resource management, resource management process, resource
management technique, resource Management tools, resource allocation, over-allocation,
resource dependency, the importance of Human Resource Management, Human resources
planning, and the function of human resource management in construction projects,
Construction Project Costs, labor requirement, and labor cost, Resource, Loading,
Aggregation, and Resource Leveling/smoothing;
 Knowing and understanding the project Resource management, resource management
process, resource management technique, resource Management tools, resource allocation,
over-allocation, resource dependency, the importance of Human Resource Management,
Human resources planning, and the function of human resource management in construction
projects, Construction Project Costs, labor requirement, and labor cost, Resource, Loading,
Aggregation, and Resource Leveling/smoothing;
 Can prepare Construction Project Costs, labor requirements, labor cost, Resource, Loading,
Aggregation, and Resource Leveling/smoothing.

4. Equipment
 Desktop or Laptop computer with internet connection.
 MS software textbook

5. Practice contents/Activities/Safety

96
6. Clean-up
 After finishing practice, all tools and equipment utilized in the practice shall be returned to
the proper storage place.
 Clean up the practice shop.

7. Independent practice/Follow-up activities


Learning through assignment

8. Review/Reflection
Review the outcome of the practice, improvement measures, and previously reflected

opinions.

97
98
99
Chapter 3: Resource Management

3.1. Resource Management Process


As stated in the above chapter, resource management is an ongoing process that starts during
the project planning phase and continues until project closure. This is known as the resource
management life cycle, which is made up of four stages that project managers should understand
to properly manage project resources.
Resource Planning: Once you’ve defined your project scope, you need to estimate what
resources are needed for each task. Don’t forget to count resources that might be needed to
manage changes and implement risk management strategies.
Resource Scheduling: Now that you have project resources at your disposal, you need to ensure
those resources are readily available. To do so, you’ll need to simply align your resource
schedule with your overall project schedule and have a solid supply chain in place.
Resource Allocation: Resource allocation is an ongoing process that is simply defined as
picking the right resources at the right time to achieve project tasks. For example, there are
critical tasks that need to be prioritized when creating the resource schedule.

3.2. Resource Management Techniques

Project managers implement the following resource management techniques to forecast, plan,
allocate, level, and optimize resources during the execution of a project.

3.2.1. Resource Forecasting

Project managers must do their best at estimating what resources are needed for a project and
how those resource requirements fit with the organization’s current plans. To do this, you must
define your project scope to identify all project tasks and their required resources.

100
3.2.2. Resource Allocation

Resource allocation consists in evaluating available resources, capacity, resource scheduling, and
the tasks that need to be completed to find the team members with the most relevant skills and
make sure they have all the project resources they need when they need them.
One of the problems or dangers inherent in the scheduling of the project is that they often do not
address the issues of resource utilization and availability .schedules tend to focus primarily on
time rather than on both time and resources, which includes people. An important measure of a
project manager's success is how well he or she balances the trade-offs among performance,
time, and cost. During a period of crisis, it is occasionally possible to add additional resources
such as additional staff to a project at little or no cost. Most of the time, however resolving
performance, time, and cost trade-offs entail additional costs to the organization. The project
manager’s goal must be to achieve project success without increasing the cost or time required
to complete the project. The key to accomplishing this goal is effectively managing human
resources on the project.

Once people are assigned to projects, there are two techniques available to the project managers
that help them to use project staff most effectively: resource loading and resource-leveling.
Resource loading refers to the number of individual resources an existing schedule requires
during specific periods. Resource loading helps project managers develop a general
understanding of the demand a project will make on the organization's resources, as well as on
individual people’s schedules. The project manager often uses histograms, to depict period–by–
period variations in resource loading. A histogram can be very helpful in determining staffing
needs or in identifying staffing problems.
A resource histogram can also show when work is being over-allocated to a certain person or
group. Over-allocation means more resources than the area available are assigned to perform
work at a given time.

101
3.2.3. Resource Leveling

The purpose of resource leveling is to assess your team members’ skills and find opportunities
for better resource allocation. By thoroughly understanding what your team members can offer,
you can assign tasks based on their abilities to maximize resource efficiency.

3.2.4. Resource Utilization

Careful resource planning is equally as important as resource tracking. Project managers need to
keep track of resource utilization to spot any resources that aren’t being used efficiently. Then
they can simply reallocate those resources or make changes to the resource management plan.

3.3 Resource Management Tools

These are the tools that project managers use to apply resource management techniques to plan
and monitor project resources.

3.3.1. Resource Management Plan

Just like every other aspect of project management, when you’re scheduling your resources, you
want to first start with a plan. This is the bedrock on which you’ll build your management
process. Make a detailed list of all resources you think are needed to complete the project. The
more detailed the better. As this is a plan, you’ll want to involve others in this process, as certain
team members might require additional resources than you expect. Be sure to include essential
and non-essential personnel and equipment. It’s better to plan for items and not ultimately
require them than have to scramble to, say, acquire an expensive piece of equipment at the last
minute, and pay a premium price. Resource planning, in short, is financial planning.

3.3.2. Resources Breakdown Structure (RBS)

Now that you have a completed list of the resources needed to complete your project, you want
to order them in some way. You can create hierarchies of resources, also known as Resources

102
Breakdown Structure (or RBS), according to hiring organization (like a reporting structure or
team hierarchy) or by geography (such as all the teams or equipment required in Asia or Africa).
These basic hierarchies should include personnel and preferably all resources on which the
project funds will be spent, but it’s up to you to define which type of hierarchies are relevant to
your project. Note that an RBS differs from a work breakdown structure in a typical project plan,
which defines the tasks associated with core parts of a project.

3.3.3. Responsibility Assignment Matrix

With your resources listed and organized, you can begin to start defining resources that have
various levels of responsibility for completing project tasks or the overall project. This is called
a Responsibility Assignment Matrix (RAM). Here’s where you can clarify the roles and
responsibilities of the entire project. You can break this down into who is responsible for doing
what tasks, who owns that task, which can help if there are questions about the task, and who
needs to stay informed about the progress of this task. Many RAMs are simple charts with coded
letters that define different functions of accountability, based on the RACI method. R stands for
“Responsible,” A is “Accountable,” C is “Consulted” and I is “Informed.” You can do this by
individual or role depending on the needs of the organization.

3.4, Resource Management Key Terms

To better understand managing resources for projects and to learn how to develop an effective
resource plan, we’ll first want to break down the different terms and processes that are often
associated with resource management. Remember, resources can be human or non-human, like
equipment or office space. It’s everything that has a cost required to complete the project.

3.4.1. Resource Over allocation

Over-allocation simply means when a person is given too much work. They can’t finish the work
in the time you’ve scheduled or allocated for it to be completed. This can lead to overtime, which

103
impacts the budget, or can block and even derail a project. Resources must be balanced so you’re
going to need a way to stay on top of your team’s workload throughout the life cycle of the
project. To see exactly how project management software can help you manage your resources,
watch the short video below. Features like Gantt charts, workload calendars, timesheets, and
project reports combine to simplify your resource management experience. And when those
features are online, then all of your data is in real-time, so your resource updates are always
fresh.

3.4.2. Resource Dependency

Resource dependency refers to a theory that an organization should avoid putting all of its eggs
in one team basket. That is, over-reliance on one team to accomplish core work, especially if it’s
an external team, can lead to workload blocks and resource shortages. You want to plan against
an unhealthy dependency on one team or one resource to accomplish the work. You can mitigate
this by distributing the work across multiple resources or having backup plans if planned
equipment resources become unavailable. You also want to make sure your resources aren’t
blocked by an over-dependence or over-reliance on one or more resources. If one team member
has too many tasks and not enough time to get them done, they could be blocking other people’s
work.

3.5. Human Resource Management

3.5.1. Definitions:

Is the process of obtaining, training, appraising, and compensating employees, and of attending
to their labor relations, health and safety, and fairness concerns? And is a strategic, integrated,
and coherent approach to the employment, development, and well-being of the people working
in organizations.
It involves all management decisions and actions that affect the nature of the relationship
between the organization and its employees which includes a set of policies designed to

104
maximize organizational integration, employee commitment, flexibility, and quality of work.
And it is concerned with how organizations manage their workforce.

3.5.2. Introduction

Human resource management in the 21st century impacts an organization at all phases of the
construction business. From the strategic planning phase of knowing the current and available
skill capacity of its labor force to knowing the operational effectiveness of each department and
abilities to carry out new applications and processes, human resource management impacts the
organization. At every stage of the business cycle, leaders need and want to know: “Do we have
the talent to get to achieve our goals?” Human resource departments have evolved into the
linchpin for implementing annual construction project goals, but before a human Resource
department can do that well, they must first be able to provide daily operations.

HR forms are tools that allow organizations to process daily HR transactions. Human Resource
forms include everything from job posting requests, job applications, and salary adjustments to
organizational performance assessments, project completion reviews, and position terminations
and promotions. Effective Human Resource departments ensure the forms they use are necessary
and serve a purpose that adds value somewhere along the line. This volume will provide Human
Resource professionals with a tool to use in building Human Resource forms. These forms can
be used to process Human Resource transactions and measure HR effectiveness. The volume
presents forms that can be downloaded, modified, and used by construction organizations today.
Advantages of Human Resource Management:

 Avoid Personnel Mistakes. First, having a command of this knowledge will help you avoid the
sorts of personnel mistakes you don’t want to make while managing. For example, no
construction project manager wants to:
 Hire the wrong person for the job
 Experience high turnover
 Have your people not doing their best
 Waste time with useless interviews

105
 Have your company taken to court because of your discriminatory actions
 Have your company cited under federal occupational safety laws for unsafe practices
 Have employees think their salaries are unfair relative to others in the organization etc.
….

 Improve Profits and Performance.


 Effective human resource management can help ensure that you get results through
people.
 Remember that you can do everything else right as a manager lay brilliant plans, draw
clear organization charts, set up world-class assembly lines, and use sophisticated
accounting controls but still fail, by hiring the wrong people or by not motivating
subordinates.
 On the other hand, many managers’ presidents, generals, governors, and supervisors have
been successful even with inadequate plans organizations, or controls. They were
successful because they had the knack for hiring the right people for the right jobs and
motivating, appraising, and developing them.
 Improve job site efficiency
 Increased field productivity
 Fewer costly interruptions
 Improved employee safety
 Improved financial performance
 Minimized unnecessary tool, equipment, and material purchases
 Lower insurance premiums
 Improved asset history that enables more informed purchasing decisions
 Greater purchasing power through the ability to combine orders
 Improved regulatory compliance
 Improved financial controls and regulatory compliance

106
3.5.3.Human resources planning

Human resource for construction planning breaks down into three major categories as follows:
1. Home office personnel
2. Construction personnel(field supervision & labour)
3. Construction sub-contractors

3.5.3.1. Home office personnel

This is direct to the owner, consultant, contractor, or management personnel. Normally,


personnel cannot be phased off the job quickly while the project budget will overrun therefore
construction manager must remember all necessary start-up requirements of design documents,
facilities, personnel, materials, and equipment must be available to support an early labor pool.

Management sets direction

Plannin Performanc
Resources e • Attain
g
• Human goals
• Financial • Products
• Raw Materials • Services
• Technological • Efficienc
Controlling Organizi
• Information ng

Outputs

Leading
Inputs

Figure 3.1.: Management Function inputs and outputs

107
Planning: inventing a systematic approach for attaining the goals of the organization.
Management starts with planning. Without a plan, organizations will never succeed plan. Due to
this management should assess all possible scenarios including the best and worst and what
actions to take. Planning will help us in:

 What to accomplish (goals);


 When to accomplish the goals;
 What resources to use;
 Who should accomplish what;
 Where to accomplish; and
 What methods to use.

Organizing: refers to the process of designing jobs and departments and determining authority
relationships in organizations.

 Permits people to work together to achieve goals;


 Helps to achieve synergy (interaction);
 Avoid duplication of resources;
 Establish authority; and
 Facilitate communication.

Leading: Influencing, inspiring, empowering employees to work towards the leaders’ vision,
and guiding, leading, and overseeing employees achieve organizational goals. Leading involves:

 Building successful groups and teams in organizations;


 Motivating people;
 Communication; and
 Developing organizational change.

108
Controlling: Establishing performance standards and comparing results and expectations to
make appropriate changes, and is the process of measuring performance, comparing it with the
objectives, and making any necessary adjustments. The purpose of control includes:

 Adapting to changes;
 Minimizing consequences of errors;
 Helping the organization cope with complexities; and
 Improving efficiency.

3.5.3.2. Construction Personnel (Field Supervision & Labour)

It is known that the field personnel breaks down into two groups of people namely: Field
supervision and craft labor. Percentage-wise, the field supervision working hours are very low
compared to craft labor hours, so the personnel loading curve is not usually required for
supervision staff. But, the number and quality of field supervision staff are the most vital
activities contributing to the success of any construction project. And the number & type of field
supervision varies depending on the Size, contracting plan, and type of the project involved.

3.5.3.3. Construction Sub-Contracting

Here such trades as painting, electrical, sheet metal work, roofing, etc. namely are subcontracted
on m0………………o1st construction jobs. Since the sub-contractor has to come to the job site
to perform the work. Having sub-contracting the problem of communication cost control, and
quality is relatively easy to handle.
Most sub-contractors are major players on the construction team, so be sure the fastest mutual
respect and project goal participation among the field staff. And if any sub-contractor fails to
perform properly, do not hesitate to discuss the problem with their management and make
personnel changes if necessary.

109
Figure 3.2: Construction Sub-Contracting

3.5.4. Construction Project Costs

In preparing the cash flow for a project, it is necessary to compute the costs that must be
expended in executing the works using activity durations and their direct and indirect costs. The
principal components of a contractor's costs and expenses resulting from the use of labor,
materials, equipment, and subcontractors. Additional general overhead cost components include
taxes, premiums on bonds and insurance, and interest on loans. The sum of a project's direct
costs and its allocated indirect costs is termed the project cost.
Generally, the costs that are spent on a specific activity or project can be classified as;
Fixed cost: costs that are spent once at a specific point of time (e.g., the cost of purchasing
equipment, etc.),
Time-related cost: costs spent along the activity duration (e.g.: labor wages, equipment rental
costs, etc.),
Quantity-proportional cost: costs change with the quantities (e.g., material cost)

110
3.5.4.1. Project direct costs

The costs and expenses that are incurred for a specific activity are termed direct costs. These
costs are estimates based on a detailed analysis of contract activities, the site conditions, resource
productivity data, and the method of construction used for each activity. A breakdown of direct
costs includes labor costs, material costs, equipment costs, and subcontractor costs. Activities’
direct costs are estimated as presented previously in chapter 2.

3.5.4.2. Project indirect costs

Other costs such as overhead costs are termed indirect costs. Part of the company’s indirect costs
is allocated to each of the company's projects. The indirect costs are always classified to project
(site) overhead; and General (head-office) overhead.

3.5.4.2.1. Project overhead

Project overhead is a site-related cost and includes the cost of items that cannot be directly
charged to a specific work element and it can be a fixed or time-related cost. These include the
costs of site utilities, supervisors, housing and feeding of project staff, parking facilities, offices,
workshops, stores, and first aid facilities. Also, it includes plants required to support working
crews in different activities. A detailed analysis of the particular elements of site-related costs is
required to arrive at an accurate estimate of these costs. However, companies used to develop
their forms and checklists for estimating these costs. Sit overhead costs are estimated to be
between 5% - 15% of the project's total direct cost.

3.5.4.2.2. General overhead

The costs that cannot be directly attributed to a specific project are called general overhead.
These are the costs that are used to support the overall company activities. They represent the
cost of the head-office expenses, managers, directors, design engineers, schedulers, etc.
Continuous observations of the company expenses will give a good idea of estimating reasonable
values for the general overhead expenses. Generally, the general overhead for a specific contract

111
can be estimated to be between 2% - 5% of the contract's direct cost. The amount of the general
overhead that should be allocated to a specific project equals:

It is very important to determine the actual dates when the expenditures (cost) will take place. At
that time, the expenditures will rename expenses. Figure 3.4 illustrate the difference between the
costs and the expenses. As shown in the figure, they are the same except the expenses are shifted
(delayed) then the costs).

Figure 3.4: Project cost and expense curves

Example 3.1
Consider the construction of 8-week foundation activity with an operating cost of LE8800. The
operation cost is broken down into the following elements:
 Labour LE 1600 paid weekly,
 Plant LE 4000 paid weekly after 4 weeks credit facility,
 Materials LE 800 paid weekly after 5 weeks credit facility,
 Subcontractors LE 2400 paid weekly after 3 weeks credit facility. So, determine the expenses
(cash out) of this activity.
Solution
A time-scaled plan is developed for this activity for the payments for labor, plant, material, and
subcontractors. The cost will be plotted weekly with the delay specified in Example 3.1.

112
Table 3.1 The general project cost (labor, plant, material, and subcontractors
cost)

Operation Weeks
1 2 3 4 5 6 7 8 9 10 11 12 13
Labor - 200 200 200 200 200 200 200 200
Plant - - - - 500 500 500 500 500 500 500 500
Material - - - - - 100 100 100 100 100 100 100 100
Subcontractors - - - 300 300 300 300 300 300 300 300
Total payment - 200 200 500 1000 1100 1100 1100 1100 900 900 600 100
(Expense)

3.5.5. Schedule of Labour Requirement

As we have seen in the previous chapter a typical format of labour requirement schedule is
shown in Table3.2. From the split-up of quantity, labor requirement is calculated. The
requirement of labor is estimated for each activity. The number of laborers is dependent on the
quantity involved in an activity and the productivity of laborers. While the productivity aspect
has been dealt with separately elsewhere in the book, it has been referred to for illustration here.
Thus, in our example, the quantity of earthwork involved in month 1 is 833.33 m3. If the
productivity of laborers for this work is assumed to be 3 man-hours per m3, the total man-hours
required for this activity alone would be 2,500 man-hours. If it is assumed that each day consists
of:

Table 3.2: Schedule of Labour Requirement

S.NO Description (labor category) Total man M M M M M M


Total man per month 1 2 3 4 5 6
1 Unskilled
2 Semi-skilled

113
3 Skilled

3.5.6. Labor and Labor Costs

The real challenge in pricing construction work is determining labor and equipment
expenditures. These are the categories of construction expenses that are inherently variable and
the most difficult to estimate accurately. To do an acceptable job of establishing these outlays,
the estimator must make a complete and thorough job analysis, maintain comprehensive unit
costs and production rates from past projects, and obtain advanced decisions regarding how
construction operations will be conducted. Contractors differ widely in how they estimate labor
costs. Some choose to include all elements of labor expense in a single hourly rate. Others
evaluate direct labor cost separately from indirect cost.

Some contractors compute regular and overtime labor costs separately, while others combine
scheduled overtime with straight time to arrive at an average hourly rate. Some evaluate labor
charges using production rates; others use labor unit costs. There are usually good reasons for a
contractor to evaluate its labor expense as it does, and they’re certain are no single correct
method that must be followed. The procedures described in this content are commonly used and
are reasonably representative of general practice.

The most reliable source of labor productivity information is obtained from cost accounting
reports compiled from completed projects. Labor cost information is also available from a wide
variety of published sources. While information of this type can be very useful at times, it must
be emphasized that labor productivity differs from one geographical location to another and
varies with season and many other job factors. Properly maintained labor records from recent
jobs completed in the locale of the project being estimated to reflect, to the maximum extent
possible, the effect of local and seasonal conditions.

As per different research, for assessing the labor cost for any item, engineers rely mainly on the
prevailing labor rates in the vicinity of sites and the productivity norms as published by various
research organizations. In addition, the rule of the experience of the estimator is used. To assess
the percentage of people using the above method of analysis of rates for various items of work, a

114
question is asked. The response to this question indicates that a good 54 percent of the
respondents rely on productivity norms, while 42 percent base their analysis on experience.

Generally, this component includes the cost incurred on the labor employed at the site. It consists
of not only the actual basic wage paid to the employee, but also bonus payments that may be
made from time to time, and costs towards welfare schemes such as pension, retrenchment,
insurance, health benefits, and notices pay. It may be borne in mind that local laws may provide
for mandatory contributions by the employer towards schemes providing social security and
retirement benefits for employees. At times, companies also provide housing to their employees
with or without arrangements for preparing food, and also incur a cost at the time of their
mobilization and demobilization, and all costs incurred towards such arrangements are also
counted as labor costs.

3.5.6.1. Indirect labor cost

Direct labor cost is determined from the workers’ basic wage rates, that is, the hourly rates used
for payroll purposes. Indirect labor costs are those expenses that are additions to the basic hourly
rate and that are paid by the employer. Indirect labor expense involves various forms of payroll
taxes, insurance, and a wide variety of employee fringe benefits. Employer contributions to
social security, unemployment insurance, workers’ compensation insurance, and contractor’s
public liability and property damage insurance are all based on payrolls. Employers in the
construction industry typically provide for various kinds of fringe benefits, such as pension
plans, health, and welfare funds, employee insurance, paid vacations, and apprenticeship
programs. The charge for these benefits customarily is based on direct payroll costs. Premiums
for workers’ compensation insurance and most fringe benefits differ considerably from one craft
to another. Indirect labor costs are substantial in amount, often constituting a 35 to 55 percent
addition to direct payroll expenses.

Exactly when and how indirect labor costs are added into a project estimate is important so long
as it is done. For estimating purposes, the total labor outlay can be computed in one operation by
using hourly labor rates, which include both direct and indirect costs. However, this procedure

115
may not interrelate well with labor cost accounting methods. For this reason, direct and indirect
labor charges often are computed separately when job prices are being estimated. One commonly
used scheme is to add a percentage allowance for indirect costs to the total direct labor expense,
either for the entire project or for each major work category. Because of the appreciable variation
in indirect costs from one classification of labor to another, it may be preferable to compute
indirect labor costs at the same time that direct labor expense is obtained for a given work type.

3.5.6.2. Labour Unit Costs

A labor unit cost is the direct labor expense per unit of production of a work type. To illustrate,
reference is again made to concrete placing in table 3.3. This table shows that 32 hours of crew
time are required to place 280 cubic yards of abutment concrete at a total direct labor outlay of
$9,088. Dividing $9,088 by 280 yields $32.46. This value of $32.46 is a labor unit cost; it is the
average direct labor charge of pouring1cubic yard of abutment concrete.

Table 3.3: Labor cost

Focus on Labor Cost Summary


Job: Highway Bridge Bid Item No. 6: Concrete, abutments Estimator: GAS
Cost Work type Quant Unit Calculations Labor Cost Equip Materi Total
code ity Direct Indirec ment al cost cost
cost
t
3150. Abutment 1,810 Sf Labour
20 forms, labor unit cost = $1.96 per sf
1,810 x $1.96 = $3,548
prefabrica
Material $3,54
ted Ply form:$1.05 per sf 10% $1,59
8 2053
waste 50% salvage 1,810 x 6
$1.05 x 1.10 x 50% = $1,045
lumber: $0.85 per bf 50%
salvage 1.44bf/sf 2 uses
1,810x$0.85x1.44x50%=$1,108
3157. Abutment 3,620 Sf Labour $7,31 $2,55 $2,52 $1,014
20 forms, labor unit cost=$2.02 per sf 2 9 0
3,620x$2.02=$7,312
place
Equipment

116
3days of 50-ton crane time
3x8x$105=$2,520
Materials
nails, form ties, coating$0.28per
SF 3,620x$0.28=$1,014
03159 Abutment 3,620 Sf Labour
.20 forms, labour unit cost=$0.97per SF
3,620x$0.97=$3,511 $3,51
strip $878 $840
Equipment 1
1day of 50 ton crane time
1x8x$105 = $840
0331 Concrete, 280 Cy Labour
1.20 abutment, Foreman 1x $34.00 = $34.00
place mason 1 x $32.00=$32.00
laborers 6x$22.00=$132.00
operator 1x 1x
$33.00=$33.00 oiler
$22.00=$22.00 $9,088 $3,181 $3,744 $27,048
carpenter 1x$31.00=$31.00
Crew hourly rate =
$284.00
Production rate = 8.75
cy/hr
280÷8.75x$284.00=$9,088
Equipment
50T crane
1x$105.00=$105.00
Vibrators 2x$3.00=$6.00
buckets 2x$3.00=$6.00
Equipment hourly rate =
$117.00
280÷8.75x$117.00=$3,744.0
Material
transit mix $92.00 per cy 5%
waste
280x1.05x$92.00=$27,048
0334 The 1960 Sf Labour
6.20 concrete, production rate=16.67sf/hr(1
abutment mason) 1,960 ÷ 16.67 x
, rub $32.00 = $3,76 $3,762 $1,693 $235
Material
material unit cost=$0.12per sf
1,960x$0.12=$235
0337 Concrete, 3,820 SF Labour $244 $86 $92
0.20 abutment Production rate = 500sf/hr(1

117
, curing mason) 3,820 ÷ 500 x $32.00
= $244
Materials
curing compound :coverage
= 270sf/gal 3,820 ÷
270x$6.50 = $92
Total account $27,466 $9,993 $7,104 $30,542

3.6. Resource Allocation and Aggregation

The most important resources that project managers have to plan and manage on a day-to-day
basis are people, machines, materials, and money. If these resources are available in abundance
then the project could be accelerated to achieve a shorter. On the other hand, if these resources
are severely limited, then the result will be a delay in the project completion time. In general,
from a scheduling perspective, projects can be classified as either time-constrained or resource-
constrained.

Resource allocation, also called resource loading, is concerned with assigning the required
number of resources identified for each activity in the plan. More than one type of resource may
be assigned to a specific activity. For example, fixing steel plates on a bridge deck may require
different types of resources such as welders, laborers, and a certain type of welding machine.
From a practical view, resource allocation does not have to follow a constant pattern; some
activities may initially require fewer resources but may require more of the same resources
during the later stages of the project. After each activity has been assigned its resources, the next
step is to aggregate the resources used by all activities.

Resource aggregation is simply the summation, on a period-by-period basis, of the resources


required to complete all activities based on the resource allocation carried out previously. The
results are usually shown graphically as a histogram. Such aggregation may be done on an
hourly, daily, or weekly basis, depending on the time unit used to allocate resources. When a bar
chart is used, resource aggregation is fairly straightforward.

118
For a given bar chart, a resource aggregation chart can be drawn underneath the bar chart.
However, a separate graph will be required for each resource type. An example is shown in
Figure 3.5, where, for a particular resource, the required resource units for each period are
written on the bar chart. The total number of resource units for each time can then be summed
and a resource aggregation or load chart can be produced as presented underneath the bar chart.
Thus, having a project schedule is necessary to facilitate the bar chart drawing.

Figure 3.5: Resource aggregation

Figure 3.6: Earliest and Latest times Resource aggregation

119
The non-critical activities, activities that are not on the critical path, do not have fixed starting
and finishing times but are constrained by the earliest and latest starting and finishing times. This
situation offers the planner chance for adjusting the demand for resources. Figure 3.6 illustrates
such a situation, which shows the resource aggregation when the activities are scheduled in their
early times and late times. It can be seen that the resource requirements that arise when both the
earliest and latest start times are considered are different.

The shaded area represents the resources required by the critical activities, as these activities
have a fixed position because their early times equal their late time. Figure 3.6 shows, also, the
accumulation of resources at the beginning of the project when the activities are scheduled at
their early time. On the other hand, the resources accumulate at the end of the project when the
activities are scheduled at their late times.

3.7. Resource Leveling

Resource leveling is a technique for resolving resource conflicts by delaying tasks .it is a form of
network analysis in which resource management concerns drive scheduling decisions ( start and
finish dates). The main purpose of resource leveling is the smoother distribution of resource
usage. Project managers examine the network diagrams from areas of slack or float, and identify
resource conflicts. For example, sometimes remove over allocations by delaying non-critical
tasks, which does not result in an overall schedule delay. Other times you will need to delay the
project completion date to reduce or remove overallocation.

It is known that a project is classified as time-constrained in situations where the project


completion time cannot be delayed even if additional resources are required. However, the
additional resource usage should be no more than what is necessary. Accordingly, the primary
focus, for purposes of scheduling, in time-constrained projects is to improve resource utilization.
This process is called resource leveling or smoothing.

120
It applies when it is desired to reduce the hiring and firing of resources and to smooth the
fluctuation in the daily demand of a resource, as shown in Figure 3.7 In this case, resources are
not limited and project duration is not allowed to be delayed. The objective, in this case, is to
shift non-critical activities of the original schedule, within their float times so that a better
resource profile is achieved.

Resource profile with high resource Resource profile with no fluctuation


fluctuation and exceeding limit (Ideal usage) and below resource limit

Figure 3.7: Resource leveling (smoothing)

As shown in Figure 3.6, the problem of resource fluctuation appears after the initial scheduling
of the project without considering the resources. The peaks and valleys in the resource profile
indicate high day-to-day variation in resource demand. Resource smoothing is the process that
attempts to determine a resource requirement that is "smooth" and where peaks and valleys are
eliminated.
For example, even if 7 units of a given resource are available at any one time, utilizing 5 of these
units each week is preferable to using 4 units for the first week, 7 the next, 2 the next, and so on.
Even if there is no limit to the amount of any one resource available, it is still desirable that
resource usage is as smooth as possible. Given that the resource requirements of those activities
on the critical path are fixed, some order or priority needs to be established for selecting which
activity and which particular resource associated with this activity should be given priority in the
smoothing process.

121
Resource leveling heuristics shift non-critical activities within their float times to move resources
from the peak periods (high usage) to the valley periods (low usage), without delaying the
project (i.e., the area underneath the resource profile remains constant). Usually, project
managers may prefer having a desired resource profile in which the resource usage starts with
low values, and then the resources are built up to their maximum values and start to decrease as
the project approaches its end as shown in Figure 3.8.

12

10

Resource units 8

0 Time

0 1 2 3 4 5 6 7 8 9 10 11 12 13 14 15 16 17 18 19

Figure 3.8: Preferred resource usage

In resource leveling, the constraint is the fixed project duration. That is, the project must get
completed by a fixed date. Such heuristic attempts to reduce the peak requirement of resources
and smooth out period-to-period assignments. Such problems are also referred to as ‘time-limited
resource considerations’ problems. The assessment of resources is done using a resource-loading
or resource-aggregation chart.

On the other hand, a project is resource-constrained if the level of resource availability cannot be
exceeded. In those situations where resources are inadequate, project delay is acceptable, but the
delay should be minimal. The focus of scheduling in these situations is to prioritize and allocate
resources in such a manner that there is minimal project delay. However, it is also important to
ensure that the resource limit is not exceeded and the technical relationships in the project
network are respected.

122
Example 3.2
Let us assume that there are a total of seven activities, A to G, in the example network below.
The duration of each of the activities is written below the arrow, while the resource requirement
is shown in the bracket adjacent to the activity name. For example, the duration for activity A is
three days and the resource required is two units. The early start and late start times of events or
nodes are also shown in the network, from which the float available in a particular activity can be
calculated and critical activities identified. The critical path of the network is 1-2-4-5-6 and it
consists of activities A, C, E, and G. The critical path is shown in bold in the network.

Figure 3.9: Network for resource-leveling illustration

Table 3.4: Resource load shows daily requirements of workers based on early-
start order
Tasks Early Duration Reso 1 2 3 4 5 6 7 8 9 10 11 12 13 14 15 16
start (days) urces
(1,2) 0 3 2 2 2 2
(2,4) 3 3 4 4 4 4
(2,3) 3 4 3 3 3 3 3
(2,5) 3 4 4 4 4 4 4
(4,5) 6 5 3 3 3 3 3 3
(3,5) 7 2 3 3 3
(5,6) 11 5 4 4 4 4 4 4
Total 2 2 2 11 11 11 10 6 6 3 3 4 4 4 4 4

123
3.7.1. Resource leveling steps

In general principles resources can be leveled, and be carried out through the following steps:
1. The project network is prepared based on the data provided for each activity. Event times and
activity times are computed as illustrated earlier; thus, the total float is also computed for each
of the activities.
2. The activities are ranked in order of their earliest start date (refer to Table 3.4). Activity (1, 2)
has ‘0’ as its earliest start time. Activities (2, 3), (2, 4), and (2, 5) all have their earliest start
times on day 3, while activities (4, 5), (3, 5), and (5, 6) have their earliest start times on days
6, 7 and 11, respectively. The resources required on daily basis for each of the activities are
summed up and shown in the form of a chart called resource-aggregation or resource-loading
chart. Figure 3.10 shows a resource-loading chart based on the earliest start time of all the
activities. The project takes a total of 87 man-days to complete, and the daily requirement
varies from a minimum of 2 resources on days 1 to 3, to a maximum of 11 resources on days
4, 5, and 6.
3. Now, the activities are ranked in order of their latest start date (refer to Table 3.5). It may be
noted that the latest start date of an activity is the latest time of the finish event less the
duration. Thus, the latest start times of activities (1, 2), (2, 4), (2, 3), (4, 5), (2, 5), (3, 5), and
(5, 6) are on days 0, 3, 5, 6, 7, 9 and 11, respectively, in the ascending order. The resource-
loading chart shown in Figure 3.11 is prepared based on the ascending order of the latest start
times of each activity. It can be noticed that the requirement of resources varies from a
minimum of 2 to a maximum of 10 resources.
4. The two resource-loading charts obtained from steps 2 and 3 are compared. The two charts
provide the two extreme arrangements of resource requirements. In the case where peaks and
valleys are seen in the utilization pattern for a resource, the activities are manipulated by
visual inspection, and an acceptable resource requirement is found between the two extremes.
The bottom line is to ensure continuous deployment of resources and to avoid large variations
in the utilization pattern. One such compromise solution is shown
In Figure 3.8, this figure has been obtained by allocating the resources in the following manner:
1. The early start time of non-critical activity (2, 3) has been followed.
2. The late start time of non-critical activities (2, 5), and (3, 5) have been followed.

124
3. The critical activities (1, 2), (2, 4), (4, 5), and (5, 6) have not been disturbed.

Figure 3.10: Resource-loading chart based on late start

Table 3.5: Resource-loading showing daily requirements of workers based on


late-start order

125
Tasks Early Duration Resou 1 2 3 4 5 6 7 8 9 10 11 12 13 14 15 16
start (days) rces
(1,2) 0 3 2 2 2 2
(2,4) 3 3 4 4 4 4
(2,3) 5 4 3 3 3 3 3
(2,5) 6 5 3 3 3 3 3 3
(4,5) 7 4 4 4 4 4 4
(3,5) 9 2 3 3 3
(5,6) 11 5 4 4 4 4 4 4
Total 2 2 2 4 4 7 6 10 10 10 10 4 4 4 4 4

Figure 3.11: Resource-loading chart based on late start

126
The above manipulations have resulted in reducing the peak requirement (from 11 to 10) besides
bringing a gradual change in resource deployment. One can also note that there is no change in
the project duration of 16 days. This is also known as time-constrained leveling. As can be
understood from this example, it would be extremely difficult to employ this technique of visual
examination for large problems. For simple problems the leveling exercise

Figure 3.12 Resource-leveled charts (time-constrained)

Table 3.6 Resource-loading table showing daily requirements of workers after


leveling

Tasks Early Duration Resou 1 2 3 4 5 6 7 8 9 10 11 12 13 14 15 16


start (days) rces
(1,2) 0 3 2 2 2 2
(2,4) 3 3 4 4 4 4
(2,3) 5 4 3 3 3 3 3
(2,5) 6 5 3 3 3 3 3 3
(4,5) 7 4 4 4 4 4 4
(3,5) 9 2 3 3 3
(5,6) 11 5 4 4 4 4 4 4
Total 2 2 2 7 127
7 7 6 7 7 10 10 4 4 4 4 4
3.7.2. Heuristic Procedure for Resource Smoothing

This section describes another way to smooth resource profiles using some heuristic rules. This
method can be summarized in the following steps:
 Prepare a complete activity schedule.
 Draw a bar chart of the project under study based on the ES timing of the activities.
 Critical activities are to be drawn first (as these activities will not be moved).

 Write the resource usage above each bar of the related activity.
 Draw the FF as a dashed line beside the upper side of the bar and the TF beside the
lower side.
 Aggregate (determine the resource sum) the resources in each period.
 Calculate the total usage of resources = ∑ unit period usage.
 Calculate the average resource usage = ∑ usage / utilization period.
 Shift non-critical activities within their FF first, then their TF to decrease the peaks and
raise the valleys.
 Revise activities floats.
 Aggregate resources at each time after shifting any activity.
 When shifting activities, it is preferred to start with the activities that have no
successors, as shifting these activities will not affect other activities. Also, by shifting
these activities, a float will be created for its predecessors.
 Shift activities only that will enhance the resource profile.

Example 3.3
The activities involved in the construction of a certain project are given in Table 3.7. One
resource type will be used during the contract. Determine the minimum level of resources
required to complete the project.

Table 3.7: Data for Example 3.3

Activity Duration (Weeks) Predecessors Resource (units/week)

128
A 0 - 0
B 2 1 0
C 5 1 2
D 3 1 2
E 2 2 1
F 6 2 2
G 6 3 3
H 6 4 1
I 4 4 0
J 2 5, 6 4
K 7 6, 7 2
L 3 2, 8 2
M 2 2, 8, 9 4
N 2 10, 11, 12, 13 0

Solution
The project network is shown in Figure 3.13 with the activity timings and project completion
time of 20 weeks. Table 3.7 shows the activities' timings and floats.

Figure 3.13: Precedence network of Example 3.3

129
Figure 3.14 shows the bar chart and the resource histogram of the project and the weekly usage
of the resources and the total usage of 90 resource units. As shown in the resource histogram, the
peak resource usage is 13 units and the minimum usage is 2 units. The total resource usage
equals 90 units with a utilization period of 18 weeks. Then, the average resource usage equals 5
units (=90/18=5).

Table 3.8: Activities times and floats of Example 3.3

Activity ES EF FF TF
A 0 0 0 0
B 0 2 0 3
C 0 5 0 0
D 0 3 0 6
E 2 4 4 12
F 2 8 0 3
G 5 11 0 0
H 3 9 0 6
I 3 7 2 9
J 8 10 8 8
K 11 18 0 0
L 9 12 6 6
M 9 11 7 7
N 18 20 0 0

The resource demand on weeks 9, 10, and 11 is high, while it is low in weeks 13 through 18.
Accordingly, the solution process will try to sift the resources from that peak period to the
period of low usage. The following activities will be shifted:

 Activity M has a free float of 7 weeks. Shifting activity M by 7 weeks will reduce
the peak usage of the resource on weeks 10 and 11 and increase the usage on
weeks 17 and 18. Also, shifting activity M will give chance for preceding

130
activities to be shifted.
 Activity J can be shifted by 6. However, it has 8 weeks of free float. By shifting
activity J, the free float of both activities E and F are changed.
 Shift activity L by 2 weeks to optimize resource usage. The free float of activity
will be changed to 2 weeks.
 Next, shift activity E by 10 weeks to improve resource usage.
 Shift activity H by 2 weeks.
 Finally, shift activity F by 1 week.

0 1 2 3 4 5 6 7 8 9 10 11 12 13 14 15 16 17 18 19 20

C, 2
G, 3
K, 2
N, 0

B, 0
D, 2
E,1
F, 2
H, 1
I, 0
J, 4
L, 2
M, 4

4 4 7 6 5 6 6 6 8 13 9 4 2 2 2 2 2 2 ∑ = 90

Units
14
13
12
11
10
9
8
7
6
5
4
3
2
1
0 Time

0 1 2 3 4 5 6 7 8 9 10 11 12 13 14 15 16 17 18 19 20

131
Figure 3.14: Bar chart and resource histogram before leveling

The heuristic procedure for leveling project resources is shown in Figure 3.15. In each step, the
resources are aggregated to ensure that shifting an activity improves resource utilization. The
resource histogram for the leveled project is shown in Figure 3.16.

Figure 3.15: Applying heuristic procedure for resource-leveling

4
o
u
e
s

1
Time

132
0 1 2 3 4 5 6 7 8 9 10 11 12 13 14 15 16 17 18 19 20

Figure 3.16: Resource histogram for Example 3.3 after leveling


Example 3.4

The activities involved in the construction of a small project are given in Table 3.9. The
resource usage for each activity is shown in Table 3.9. Smooth the resource so that preferred
resource usage is obtained.

Table 3.9: Data for Example 3.4

Activity Duration (Weeks) Predecessors Labors (units/week)


A 3 - 9
B 5 - 6
C 1 - 4
D 1 A 10
E 7 B 16
F 6 B 9
G 4 C 5
H 3 C 8
I 6 D, E 2
J 4 F, G 3
K 3 H 7

Solution

The precedence network of the project is shown in Figure 3.17 with the activity timings and
project completion time of 18 weeks . To achieve a resource profile with less resource load at
the beginning and build-up towards the middle of the project and decreases towards the end, the
following activities will be shifted:
 Shift activity K by 11 weeks, this activity has a free float of 11 weeks.
 Shift activity H by 11 weeks (it has 11 weeks of free float).
 Activity A will be shifted by one week, accordingly, activity D will e shifted by one week.

133
This is because activity A has no free float.
 Shift activity F by 3 weeks and accordingly, activity J will be shifted 3 weeks because
activity F has no free float.
 Finally, shift activity G by 3 weeks.

Figure 3.17: Precedence network of Example 3.4

0 1 2 3 4 5 6 7 8 9 10 11 12 13 14 15 16 17 18

B, 6
E, 1 6
I, 2

A, 9
C, 4
D, 1 0
F, 9
G, 5
H, 8
J, 3
K, 7

19 28 28 29 18 32 32 25 25 25 25 19 5 5 5 2 2 2

134
Figure 3.18: Bar chart and resource aggregation

Figure 3.18 shows the bar chart and the resource histogram of the project and the weekly
usage of the resources.

The heuristic procedure for leveling project resources is shown in Figure 3.19. In each step, the
resources are aggregated to ensure that shifting an activity improves resource utilization. The
resource histogram for the leveled project is shown in Figure 3.20.

Figure 3.19: Solution of Example 3.4

135
30

25

20

Units
Res
ourc

15
e

10

0
Time (weeks)
0 1 2 3 4 5 6 7 8 9 10 11 12 13 14 15 16 17 18

Figure 3.20: Resource histogram after leveling of Example 3.4

Example 3.5

The activities of a project along with their durations, predecessors, and resources used are given
in Table 3.10. If resource 1 is limited to 8 units and resource is limited to one unit, determine
the activities schedule start and finish times so that the weekly resource usage does not exceed
the resource limits.

Table 3.10: Data of Example 3.5

Duration Resource (units/week)


Activity (Weeks) Predecessors
R1≤8 R2 ≤1
A 4 - 3 0
B 6 - 6 1
C 2 - 4 0
D 8 A 0 1
E 4 D 4 1
F 10 B 0 1
G 16 B 4 0
H 8 F 2 0
I 6 E, H 4 1
J 6 C 5 1
K 10 G, J 2 0
Solution

136
The project network is drawn and the activities timings are calculated giving a project
completion time of 32 weeks without considering the resource limits. The solution will be
arranged in the table below (Table 3.11).

Table 3.11: Solution of example 3.5

Current Eligible Resources Finish


Duration ELS Decision
time activities R1 ≤8 R2 ≤1 time
B 6 1 6 0 Start 6
0 A 3 0 4 10 Delay -
4 0 2
C 14 Delay -
G 4 0 16 6 Start 22
F 0 1 10 8 Start 16
6 3 0 10 10
A 4 Start
4 0 14 -
C 2 Delay
G 4 0 16 - Continue 22
F 0 1 10 - Continue 16
10
C 4 0 2 14 Start 12
D 0 1 8 14 Delay -
G 4 0 16 - Continue 22
F 0 1 10 - Continue 16
12 Delay
D 0 1 8 14 -
Delay
J 5 0 6 16 -
G 4 0 16 - Continue 22
D 0 1 8 14 Start 24
16
J 5 1 6 16 Delay -
H 2 0 8 18 Start 24
D 0 1 8 - Continue 24
22 H 2 0 8 - Continue 24
J 5 1 6 16 Delay -
J 5 1 6 14 Start 30
24
E 4 1 4 22 Delay -
E 4 1 4 22 Start 34
30 K 2 0 10 22 Start 40
K 2 0 10 - Continue 40
34 6 Start
I 2 0 26 40

Then, the project completion time is 40 weeks with activities timing as given below (Figure
3.21)

137
Figure 3.21: Precedence diagram of Example 3.5

Table 3.12: Solution of example 3.5

Schedule Schedule
Activity start finish
A 6 10
B 0 6
C 10 12
D 16 24
E 30 34
F 6 16
G 6 22
H 16 24
I 34 40
J 24 30
K 30 40

3.7.3. General producer for human resource leveling

138
The consequence of employment fluctuation and equipment utilization adds to the cost of the

project. This is a problem where the only solution is to keep employment at an acceptable level.

And the use of activity float on level resource requirement is illustrated as following project

hypothesis.

Example 3.6
The activities of a project along with their durations, predecessors, and human resources used
are given in Table 3.13 level the resources.

Table 3.13: Solution of example 3.6

Activity Predecessors Duration Human resource

A - 11 10
B - 4 10
C - 7 5
D B 8 15
E - 5 15
F C 5 5
G E, F 6 10
H E, F 5 5
I E, F 14 15
J A, D, H 6 5
K C 6 10
L J, G, 4 5

Answer

139
Step-1: Draw the network diagram

Solution

K-10 men

6 Days

I - 15 men
14 days

G -10 men
6 Days

C -5 men F-5 men H-5 men J-5 men L-5 men


1 3 4 6
7 Days 5 Days 5 Days 5 6 Days 4 Days 7
2

E -15 men
5 days
B -10 men D -15 men
2
4 days 8 days
2
A -10 men
11 days

Figure 3.22: Network Diagram

Step-2: Determining the paths, and the critical path.


Solution

Paths Duration
1. C, K 7 + 6 = 13
2. C, F, I 7 + 5+ 14 = 26
3. C, F, G, L 7 + 5 + 6 + 4 = 22

140
4. C, F, H, J, L 7 + 5 + 5 + 6+ 4 = 27
5. E, I, 5 + 14 = 19
6. E, G, L 5 + 6 + 4 = 15
7. E, H, J, L 5 + 5 + 6 + 4 = 20
8. B, D, J, L 4 + 8 + 6 + 4 = 22
9. A, J, L 11 + 6 + 4 = 21

 Therefore activity, C, F, H, J, L (7 + 5 + 5 + 6+ 4 = 27) is the critical activity,


 Is the longest path duration (27 days) to complete the project
 And all the critical activity has no float days (Zero floats).

Step-3: Float Calculation

Solution

Activity Predecessors Float


A - 7-7 = 0
B - 4- 4 = 0
C - 7-7 = 0
D B 17- 12= 5
E - 12-5 = 7
F C 12-12 = 0
G E, F 23-18= 5
H E, F 17-17=0
I E, F 27-26= 1
J A, D, H 23-23= 0
K C 27-13=14
L J, G, 27- 27= 0

Step-4: Manpower Calculation

141
Solution
1. From Day 1 to 4 the manpower of activities are:
 Activity B, E, C, and A for 4 days where, Activity B - completed
 Daily manpower = 10 + 15 +5 +10= 40
 Total manpower = 40 x 4 = 160

2. On day 5 the manpower of activities are:


 Activity E, C, A, and D for 1 day where, Activity E - completed
 Daily manpower=15+5+10+15= 45and there are resource fluctuations
 Total manpower = 45 x 1 = 45

3. On days 6 to 7 the manpower of activities are:


 Activity C, A, and D for 2 days where, Activity C - completed
 Daily manpower =5 +10 +15= 30 and there is resource fluctuation
 Total manpower = 30 x 2 = 60

4. Days 8 to11 the manpower of activities are:


 Activity K, F, A, and D for 4 days where, Activity A - completed
 Daily manpower =10 +5 +10 +15= 40 and there is resource fluctuation
 Total manpower = 40 x 4 = 160

5. On day 12 the manpower of activities are:


 Activity K, F, and D for 1 day where, Activity F& D - completed
 Daily manpower =10 +5 +15= 30 and there is resource fluctuation
 Total manpower = 30 x 1 = 30

6. On day 13 the manpower of activities are:


 Activity K, I, G, and H for 1 day where, Activity K - completed
 Daily manpower =10+15 +10+5= 40 and there is resource fluctuation
 Total manpower = 40 x 1 = 40
7. On days 14 to 17 the manpower of activities are:
 Activity I, G, and H for 4 days where, Activity H - completed
 Daily manpower =15 +10+5= 30 and there is resource fluctuation
 Total manpower = 30 x4 = 120
8. On day 18 the manpower of activities are:

142
 Activity I, G, and J for 1 day where, Activity G - completed
 Daily manpower =15 +10+5= 30 and there is resource fluctuation
 Total manpower = 30 x 1= 30

9. On days 19 to 23 the manpower of activities are:


 Activity I, and J for 5 days where, Activity J - completed
 Daily manpower =15 +5= 20 and there is resource fluctuation
 Total manpower = 20 x5 = 100

10. On days 24 to 26 the manpower of activities are:


 The activity I, and L for 3 days where, Activity I - completed
 Daily manpower =15 +5= 20
 Total manpower = 20 x3 = 60
11. On day 27 the manpower of activities are:
 Activity L for 1 day where, Activity L- completed
 Daily manpower = 5 and there is resource fluctuation
 Total manpower = 5 x1 = 5
Graphically: Histogram

60

50
45
40 40 40 40
Res
ourc

30 30 30 30
30
e

20
20 20

10
5
0 Time

1 45678 11121314 17 18 19 23 24 26 27

Figure 3.23: Resource histogram before leveling

143
Step – 5 Resource (Manpower) Leveling/smoothing
 The total manpower required to complete the project is 810
 The age of the project or the longest duration (critical path) to complete the project is
27 days.
Solution
Then, to level the resource, 810 man powers divided into 27 days, that is:
810/27 =30 at least number of manpower required per day of the 27 days.

1. On days 1 to 4 the manpower of activities are:


 Activity B, E, C, and A for 4 days Where delay task “A”
 Daily manpower = 10 + 15 +5= 30 Focus the 30 targeted and
 Total manpower = 30x 4 = 120 Activity “B” completed

2. On day 5 the manpower of activities are:


 Activity E, C, and A for 1 day Where delay task “D”
 Daily manpower = 15 +5+ 10= 30 Focus the 30 targeted and
 Total manpower = 30x 1 = 30 Activity “E” completed

3. On days 6 to 7 the manpower of activities are:


 Activity C, A, and D for 2 days Focus on the 30 targeted and
 Daily manpower = 5+ 10+ 15= 30 Activity “C” completed
 Total manpower = 30x 2 = 60

4. Days 8 to12 the manpower of activities are:


 Activity A, D, and F for 5 days Where delay task “K”
 Daily manpower = 10+ 15+5= 30 Focus the 30 targeted and
 Total manpower = 30x 5 = 150 Activity “ F” completed

5. On day 13 the manpower of activities are:


 Activity A, D, and H, for 1 day Again delay task “I, K, G”

144
 Daily manpower =10+15 +5= 30 Focus the 30 targeted and
 Total manpower = 30 x 1 = 30 Activity “D” completed

6. On days 14 to 15 the manpower of activities are:


 Activity A, I, and H for 2 days Where, delay Activity “K, G”
 Daily manpower =10+15 +5=30 Focus on the 30 targeted and
 Total manpower = 30 x 2 = 60 Activity “A” completed

7. On days 16 to 17 the manpower of activities are:


 The activity I, H, and G for 2 days Where, delay Activity “K ”
 Daily manpower =15+5 +10=30 Focus on the 30 targeted and
 Total manpower = 30 x 2 = 60 Activity “ H” completed

8. On days 18 to 21 the manpower of activities are:


 Activity J, I, and G for 4 days Where delay Activity “K, ”
 Daily manpower =5+15 +5=30 Focus on the 30 targeted and
 Total manpower = 30 x 4 = 120 Activity “G” completed

9. On days 22 to 23, the manpower of activities are:


 Activity J, k, and I for 2 days Focus on the 30 targeted and
 Daily manpower =5+10 +5=30 Activity “J” completed
 Total manpower = 30 x 2 = 60

10. On days 24 to 27 the manpower of activities are:


 The activity I, K, and L for 4 days Where, delay Activity “K, G”
 Daily manpower 15 +10 +5=30 Focus on the 30 targeted and
 Total manpower = 30 x 4 = 120 Activity “I, K, L” completed

To cote, Still the project has a contract time is 27 days (the critical time of the
project) without affecting or balance of the project while moving an activity that

145
has float time.
Graphically: Histogram

60
50
ourc

30 40
30 30
e

30
20

0
1 2 3 4
Time
5 6 7 8 9 10 11 12 13 14 15 16 17 18 19 20 21 22 23 25 26 27

Figure 3.24: Resource histogram after leveling

146
Assessment

Practical

Week 10: Practice


1. From the given data in the table below:
B. DRAW the network diagram;
C. Identify the path and the critical tasks;
D. Calculate the human resource per day and the total manpower required to complete the
project, and
E. Level the humane resource.
Table A

Activity ID Name Duration Predecesso Resource per Day


r
1 A 4 - 9
2 B 2 - 3
3 C 2 - 6
4 D 2 - 4
5 E 3 B 8
6 F 2 C 7
7 G 3 D, F 2
8 H 4 E, G 1

`
Table B

Activity Duration Predecessor Resources


A 1 - 2
B 2 - 1
C 2 - 1
D 1 A 1

147
E 3 B 1
F 2 C 1
G 1 D 1
2. From the following given data in the table below:
A. Draw network diagram and precedence diagram,
B. Calculate all floats.

Activity
Activity ID Predecessors Duration
Description
1 A - 1
2 B - 2
3 C B 3
4 D A 5
5 E B 2
6 F A 4
7 G A 3
8 H D 2
9 J C 6
10 K J 3
11 L K 2
12 M F 5

148
2. Theory
Objective type of questions

1. State whether True or False:


a. Float is a useful tool for project management to use to level resources:
b. Estimates and schedules when initially prepared to assume unlimited resources:
c. During the resource leveling exercise, critical activities are given priority for resource
allocation.
d. Resource leveling exercise uses activity floats to reschedule activities, without
delaying project completion.

2. Short-answer type questions


a. How is it used for optimizing project costs? Explain with help of a diagram.
b. Explain the step-by-step process of ‘resource scheduling. What do you understand by
‘resource leveling’ and how do you achieve it?
d. What is the need for resource-leveling in any construction project?
e. Differentiate between resource smoothening and resource-leveling.
f. What do you mean by project scheduling and why is it important?
g. Discuss resource leveling and allocation

3. Which would not be considered a scheduling resource:


a. People b. Money c. Equipment d. Laydown space

149
Subjective type of questions

1. Define and elaborate resource management process, resource management technique, resource
Management tools, resource allocation, over-allocation, and resource dependency.

2. Discuss the importance of Human Resource Management, Human resources planning, and the
function of human resource management in construction projects

3. Define Construction Project Costs, direct cost and indirect cost and labor direct cost, and
labour indirect costs.

4. A Precedence network for a project has the logic given in the following table. The resource
rates for each activity are also given. It is required to:

A. Level the resource I and then draw a resource histogram before and after leveling on
the same diagram.
B. Level resource II and then draw a resource histogram before and after leveling on the
same diagram.
C. Level resources I and II simultaneously, and plot the histograms for both resources.
Compare the results with those obtained from (a) and (b).

Resource Rate
Activity Predecessors
Duration Resource I Resource II
A --- 2 3 1
B A 8 4 6
C A 6 8 6
D A 4 6 5
E A 3 4 1
F B 12 2 0
G C, D 4 7 9
H C, D, E 6 9 5
K F, G, H 3 2 0

2. The following project consisting of 10 activities is considered for multi-resource scheduling:

150
A). Develop a resource schedule assuming that the availability of resources is limited as
below.
Resource A: 7
Resource B: 7
Resource C: 6

B). If the project completion time is limited to 14 weeks. Estimate the minimum resource
level required for each resource type. Also, draw the resource histograms before and
after leveling.

Resources required
Activity (i – j) Description Duration (days)
A B C
1-2 A 3 4 4 2
1-3 B 4 3 4 1
1-5 C 5 1 3 2
2-4 D 2 1 0 0
2-6 E 3 2 1 0
3-4 F 4 2 2 1
4-7 G 3 3 1 2
5-6 H 6 4 4 4
5-7 I 4 3 2 1
6-7 J 3 1 4 5

3. Consider the following project.

Duration Resources required


Activity Predecessor
(days) Plumbers Labors
A - 4 2 3
B - 3 1 -
C - 6 1 3
D B 8 3 4
E B 7 1
F C 2 3 5
G A, 9 1 2
H DE 5 2 4
I E 4 2
J F, I 4 2 3

151
Assume eleven plumbers and nine laborers are available for the project. Both resources must
work at the same time when assigned to the same activity. Prepare an activity schedule that
satisfies the resource constraints.

152
Evaluation Sheet:

Evaluation Out of 5 Score Remark


Working Principle
Planning,
scheduling projects
with MS. Project
Result Software
Evaluatio Discussion
n
Scheduling output
using the software
and conclusion
Applications
Tool Setup
Usage and storage
Work
of the software
Evaluatio
Planning and
n
Scheduling
reliability
Time used (180
Time Min.) (1 per 10
min.)
Total /45

153
References

1. Kumar Neeraj Jha (2015), (Associate Professor Department of Civil Engineering), IIT, Delhi
Construction Project Management, Theory, and Practice. SECOND EDITION

2. Emad Elbeltagi, Ph.D., P.Eng., (2009), Professor of Construction Management Structural


Engineering Department, Faculty of Engineering, Mansoura University, Lecture notes on
construction Project management

3. Carl Chatfield &Timothy Johnson, (2013) Microsoft Project.

4. Fundamentals of Construction Management

5. Awani, Alfred O. (1983). “Project Management Techniques.” Petrocelli Books Inc.

6. Cormican, David. (1985). “Construction Management: Planning and Finance.” Construction


Press, London.

7. Gould, Frederick E. (1997). “Managing the Construction Process: Estimating, Scheduling,


and Project Control.” Prentice-Hall Inc., New Gersy.

8. Harris, Robert. (1978). “Precedence and Arrow Networking Techniques for Construction.”
John Wiley & Sons Inc., NY.

154
155
Chapter 4: Materials and Equipment Management

Lesson Plan

1. Learning objectives
At the end of this chapter students/ trainees will be able to:
 Know the Materials Management;
 Develop a better understanding of the objectives of Materials Management;
 Know the Material Procurement Process in Construction Organization;
 Improve the level of understanding of Materials Management Functions;
 Inform about the importance of Inventory Management, the function of inventory
management, and Inventory Models;
 Develop the level of understanding of Equipment Management;
 Know the Advantages of using construction equipment in construction projects;
 Identify and know the Classification of Construction Equipment;
 Identify the Factors Behind the Selection of Construction Equipment;
 Improve the level of understanding of how Equipment Costs, Life and Replacement
Analysis (Ownership cost(initial cost, Depreciation, Investment (Or Interest) Cost,
Insurance, Tax and Storage Costs), Cost of Operating Construction Equipment
(Maintenance and Repair Costs, Tire Cost, Fuel Cost, Lubricating Oil Cost, Mobilization
and Demobilization Cost, Equipment Operator Cost and Maintenance and Repair Costs),
Overhead cost and Profits);
 Develop the level of understanding of the Method of depreciation calculation and
Depreciation and taxation;
 Improve knowledge of Equipment Utilization, Equipment Life Downtime, and cost of
equipment downtime.
2. Motivation
 Outputs through Question and Answer; repetitive discussion and instruction of material

156
management, objectives of Materials Management, Material Procurement Process, Material
Management Functions, the importance of Inventory Management, the function of
inventory management, and Inventory Models, Equipment Management, Advantages of
using construction equipment’s, Classification of Construction Equipment, Factors Behind
the Selection of Construction Equipment, Equipment Costs, Life and Replacement Analysis
(Ownership cost(initial cost, Depreciation, Investment (Or Interest) Cost, Insurance, Tax and
Storage Costs), Cost of Operating Construction Equipment (Maintenance and Repair Cost,
Tire Cost, Fuel Cost, Lubricating Oil Cost, Mobilization and Demobilization Cost,
Equipment Operator Cost and Maintenance and Repair Costs), Overhead cost and Profits),
Method of depreciation calculation and Depreciation and taxation, Equipment Utilization,
Equipment Life Downtime and cost of equipment downtime.
3. Expectations or Outcomes
 Define the terms of material management, Material Procurement Process, Inventory
Management, and Inventory Models, Equipment Management, selection of Construction
Equipment, Equipment Costs, Life and Replacement Analysis (Ownership cost, Cost of
Operating Construction Equipment, Overhead cost, and Profits, Method of depreciation
calculation, Depreciation and taxation, Equipment Utilization, Equipment Life Downtime
and cost of equipment downtime.
 Able to identify objectives of Materials Management, Material Management Functions, the
importance of Inventory Management, the function of inventory management, and
Inventory Models, Equipment Management, Advantages of using construction equipment,
Classification of Construction Equipment, Factors Behind the Selection of Construction
Equipment, Equipment Costs, Life and Replacement Analysis (Ownership cost(initial cost,
Depreciation, Investment (Or Interest) Cost, Insurance, Tax and Storage Costs), Cost of
Operating Construction Equipment (Maintenance and Repair Cost, Tire Cost, Fuel Cost,
Lubricating Oil Cost, Mobilization and Demobilization Cost, Equipment Operator Cost and
Maintenance and Repair Costs), Overhead cost and Profits), Method of depreciation
calculation and Depreciation, Equipment Utilization, Equipment Life Downtime and cost of
equipment downtime.
 Can forecast and calculate equipment Costs.
4. Equipment

157
 Desktop or Laptop computer with internet connection.
 MS Project software and textbooks
5. Practice contents/Activities/Safety

6. Clean-up
 After finishing practice, all tools and equipment utilized in the practice shall be returned to
the proper storage place.
 Clean up the practice shop.

7. Independent practice/Follow-up activities


Learning through assignment

8. Review/Reflection
Review the outcome of the practice, improvement measures, and previously reflected
opinions.

158
159
Chapter 4: Materials and Equipment Management

4.1. Materials Management

4.1.1. Definition and Introduction

The term ‘materials’ is used in a general sense and refers to the whole range of goods and
services that are purchased or otherwise procured from sources outside the organization and are
used or processed to provide finished products for sale or to produce a product for constructing
construction projects. One can notice a fundamental difference in materials management in a
factory-like situation and a construction project situation. In a factory situation, products are
standardized, the manufacturing location and process are permanent, and, as such, long-term
planning is possible. Further, an increase in input costs of materials can be passed on to the
customer through an increase in the sales price of the end products. However, as has been
repeatedly pointed out in the chapter, each construction project is unique in content and nature of
execution. Project locations are transient and temporary, and the cost is predetermined at the start
of the project, which calls for utmost care in material procurement to ensure that cost provisions
are not overrun and if not, Materials account for a large fraction of the overall construction
project cost.

A typical construction project requires a variety of materials. Materials in a project are broadly
classified under capital equipment, construction types of machinery, and consumables. Some
companies classify their materials under two broad categories capital items and revenue items.
Plant and machinery, vehicles, office equipment, land, and buildings are placed under capital
items, while non-capitalized items, heavy tools and tackles, small tools, consumables, electrical
items, construction materials, special/one-time items, and spares are designated under revenue
items.

160
The importance of materials management can be gauged from the fact that in any typical
building project the share of material costs is about 55 percent; labor costs, about 25 percent; and
POL (petrol, oil, and lubricants), overheads, tax components, and profits, about 5 percent each.
Thus, materials management occupies an important place in construction management and in
recent times has received considerable attention. This is due to the following major factors:

 First, a worldwide recession has an impact on reducing sales volumes and revenues, and
this has forced management to reconsider how best to lower the levels of inventory to
maintain margins (by reducing interest and obsolescence costs).
 Secondly, the changes in manufacturing philosophy, specifically the growth in just-in-
time (JIT) applications, have reduced the need for inventory as an insurance buffer within
the overall logistics activity.
 Thirdly, many companies have realized that a greater return on investment (ROI) can be
obtained by developing the core business and that investment in working capital items,
such as inventory and debtors, returns far less in comparison.
 Fourthly, developments on the information technology (IT) front have provided a
potential tool to reduce inventory.

Materials management is the integrated functioning of the different sections of a company


dealing with the supply of materials and other related activities, so, as to obtain maximum
coordination and optimum expenditure on materials. The scope of materials management is vast
as it begins with the ‘award of contract’ and ends with material resting at its point of use.
Typically, the objectives of materials management are to:

 Minimize material cost;


 Procure and provide material of desired quality when required;
 Reduce investment tied in inventories for use in other productive purposes and develop
high inventory turnover ratios;
 Purchase, receive, transport, and store material efficiently and to reduce the related costs;
 Cut down costs through simplification, standardization, value analysis, import

161
substitution, etc.;
 Modify paper-work procedure to minimize delays in procuring material; and
 Train personnel in the field of materials management to increase operational efficiency.

To cote, materials management aims to provide the right item of the right quality, in the right
quantity, at the right price, at the right place, and at the right time for ensuring uninterrupted
execution of works. Empirical evidence strongly suggests that materials management activities
are either not being practiced or being practiced ineffectively in the construction industry. ‘Not
being able to get material’ and ‘not being able to get the right tool’ are commonly heard
complaints on the site.

Research indicates in the world (Especially in a developing country like Ethiopia) that as much
as one-third of the worker’s time is spent waiting for the right materials or tools to perform a
construction task. ‘Unavailability of materials and tools may act as a major de-motivator to
workers’ performance on a typical construction site. The result is reduced productivity and delay
in the project. It is the responsibility of the construction manager to make sure that the right
material and the right tool are available to the worker.

Muehlhausen (1987) points out some of the reasons for poor materials management practices
adopted by construction contractors:
 Senior management of construction firms does not recognize the impact that materials
management has on the cost-effectiveness of their project operations.
 Personnel performing related materials management activities have not been properly
selected and trained.
 Computerized systems related to materials management activities have not been properly
selected, designed, or used to provide needed information for management and control.

The problem of low worker productivity and unreasonable construction project cost due to
unavailable materials at the time and place of need will continue until construction managers
have learned how to manage the flow of materials to and through the construction site. It can be
appreciated that one of the areas of great concern today is the effective utilization of our planet’s

162
limited resources. This has led to a renewed understanding of the need for new and efficient
construction materials, and the re-use and recycling of materials from existing facilities.
Reconstruction and replacement of aging infrastructure will place increased emphasis on the
development of new and/or improved materials for use in pavements, bridges, water, and
electrical distribution facilities, buildings, and other structures. Therefore, management skills and
analytical tools are needed to effectively utilize the multitude of resources required for the
completion of a construction project.

4.1.2. Material Procurement Process in Construction Organization

In a large construction organization, having countrywide domestic as well as overseas


operations, the set-up for performing materials management functions can be quite big. For an
organization that has regional offices throughout the country and a head office in a major
metropolitan of the country, the materials management department is normally headed by a
person of the vice president cadre and he or she is supported by a general manager. The general
manager is overall in charge of the central materials department and regional materials
department. The responsibilities of the central materials department are to:

 Procure high-value materials centrally for different areas;


 Arrange for their transport to respective areas;
 Exercise inventory control;
 Enter into rate contract for frequently used construction materials such as cement, steel,
and plywood at the organization level;
 Develop computerized procedures;
 Procure capital equipment and arrange for their maintenance;
 Supply raw materials to the company’s manufacturing units, if any, and cater to material
needs of overseas projects;
 Import construction materials depending on the requirement of projects;
 Gather management information system reports and their analyses;
 Standardize and codify the construction materials; and

163
 Dispose of the waste and scrap materials.

Several people are involved in the materials procurement process. For example, in a large
construction organization, the project manager is largely responsible for materials management.

The planning engineer and the materials engineer are responsible for preparing the materials
schedule, sending the requisition for the materials, monitoring, and control of materials
consumption at projects, comparing the tender provision and actual cost of materials, and so on.
For the selection of sources, especially for procurement of materials of natural origin such as
aggregate and stones, the quality control engineer is responsible.

The quality engineer is responsible for sampling and testing materials received at project sites as
also for checking their conformance with the specification. The stores in charge are responsible
for the follow-up with the vendors, receiving and issuing the right materials, and so on. After the
contract is awarded to a construction organization, the planning engineer in association with the
materials engineer prepares a materials schedule and lists out the important materials and the
departments responsible for their procurement. In a large organization having countrywide and
overseas operations, materials are purchased at the head office level, regional or branch-office
level, and project-site levels.

The materials engineer in consultation with the planning engineer fills up the requisition for
desired materials giving all the details such as quantity required, schedule of procurement,
specification of materials, drawings, and list of approved manufacturers/suppliers. The
requisition in most cases needs approval from higher authorities at the project site or regional
office and is forwarded to the procurement or materials department, which may be located at a
regional office, head office, or project site itself. And at the regional- or head-office level, it is
ascertained whether the requested material pertains to:
(a) Rate contract items,
(b) Standard price-list items, or
(c) Other items. In the case of rate contract items, a purchase order is released as per the existing
rate contract, and for standard price-list items, the purchase order is placed with an agreed

164
discount. For other items, suitable vendors are identified, inquiries are sent with all the details
received from project sites, and price offers are received from suppliers. The price offers are
compared and evaluated; a structured negotiation takes place with a few suppliers, and the
purchase order is issued to the most eligible supplier. The materials supplied by the supplier are
dispatched to the project sites, which inspect, receive, and dispatch the material receipt note to
the concerned office from where the materials have been procured. Payment is released to the
vendor after the materials receipt note has been received from the project sites.

4.1.3. Materials Management Functions

The functions of materials management are:


 Materials Planning
 Procurement
 Receiving
 Materials Accounting
 Transportation
 Inventory Monitoring and Control
 Materials Codification
 Computerization
 Source Development (Vendor Development)
 Disposal

4.1.3.1. Materials Planning

The site-planning engineer is responsible for this function. Normally, the purchase of any
material is initiated by requisitioning it. To have proper control over the material purchase, it is a
good idea to have some specific engineer approve the material requisition. The site-planning
engineer makes a purchase requisition in consultation with the construction manager, following
which the material is procured by administrative people (mainly store people). Materials
schedule should be the basis for raising material requisitions. Materials planning involves

165
identifying materials; estimating quantities; defining specifications; forecasting requirements;
and locating the right sources for procurement.

4.1.3.2.Procurement

Before procurement, the site-planning engineer along with the materials engineer surveys the
local market and identifies the items that are to be procured locally and those that are to be
procured from the head office under a centralized procurement system. Local procurement
should be kept to the minimum and, as far as possible, limited to non-engineered items and
consumables. For items that are required from the head office (centralized purchasing), an
authorized purchase request is sent to the head materials manager, who will coordinate with the
site materials engineer and organize the items. The Head materials manager and site materials
engineer ensure that procurement is as per the terms and conditions of the contract. Early and
late procurement of materials can be the two approaches to material procurement.

4.1.3.3. Receiving

The main documents used are: inward register, material receipt note (MRN), delivery challan
(DC), indent, dispatch covering note, outward register, loan register, repair register, and plant.

4.1.3.4. Materials Accounting

The main purpose of materials accounting is to monitor the inflow and consumption of raw
materials. Material accounting involves materials stock accounting; materials issues and returns
accounting; monthly stocktaking of selected materials; and materials wastage analysis. Material
wastage analyses aim at finding the causes of wastage and rectifying them. Wastage during
procurement can result from various factors—buying materials of wrong specifications; buying
more than the actual requirements; unnecessary buying of items to cater for unrealistic and
unforeseen eventualities; untimely buying of short-life materials; improper and unnecessary
handling of materials; and wastage while transportation. Some other reasons for wastage of
materials are breakages and damages during handling; lack of pre-work preparation and

166
coordination; inferior quality of materials; improper accounting and poor storekeeping; negligent
attitude of the supervisor; unforeseen circumstances like accidents, fire, etc.; high rate of
deterioration due to long storage at the place of work; over-issues from the central stores and
failures to return unused surplus materials; and thefts and pilferage.

4.1.3.5. Transportation

Construction materials used at project sites undergo considerable movement right from their
point of origin to the storage point, to the actual point of consumption. In some construction
companies, materials are first received in central stores, from where they are dispatched to the
project stores located at project sites and then finally to the workplace. The construction
materials could be in the form of raw materials such as aggregate, sand, and plywood, or they
may be in a semi-finished form such as mixed mortar, mixed concrete, and dressed stones.
Generally, raw materials procurement is within the scope of the materials management team,
while the procurement of semi-finished material comes under the purview of the construction
team. Proper care should be taken while planning for the transportation of raw and semi-finished
materials to avoid any adverse effect on the characteristics or performance of materials.

4.1.3.6. Inventory Monitoring and Control

Inventory is defined as a ‘usable but idle resource’. If a resource is some physical and tangible
object such as materials, then it is generally termed as stock. Thus, stock and inventory are
synonymous terms, though inventory has wider implications. It is very important to check on the
inventory level. Scientific inventory management is an extremely important problem area in the
materials management function. Materials account for more than half the total cost of any
business, and organizations maintain a huge amount of stocks, though much of this could be
reduced by following scientific principles.

Inventory management is highly amenable to control. Across industries, there is a substantial


potential for cost reduction through inventory control. Inventory being a symptom of poor
performance, one can reduce inventories through proper design of procurement policies by the

167
reduction in the uncertainty of lead times by variety reduction, and many other ways. The
financial implications of inventory as an element of the company’s assets must be understood
properly. The following reasons could be used to justify inventory:

 Improves customer service;


 Permits purchase and transportation economies, the argument here is based on the notion
that both product procurement and transportation costs will be reduced if lot sizes are
large;
 Hedges against price changes: One can observe the tendency to hoard commodities in
anticipation of price rise just before the budget (in January/February in the Indian
context);
 Protects against demand and lead-time uncertainties: When there are uncertainties in the
customer demand patterns and the suppliers’ replenishment lead times, it is preferable to
invest in ‘safety stock’ so that services are maintained at acceptable levels to customers;
 Hedges against contingencies: While there are now fewer labor disputes in most
economies, fires, floods, and other exogenous variables can create problems. It is argued
that these will be minimized if stockholding is increased.

There are several inventory control mechanisms such as ABC analysis (based on the value of
consumption), FSN analysis (based on movement from store), and VED analysis (based on
necessity). Considering the importance of inventory monitoring and control, we discuss these in
detail later in the text.

4.1.3.7. Materials Codification

Codification is an important function in materials management, especially for construction


companies where thousands of different items are used all along the project duration. There are
different systems of codification followed by construction organizations namely, numeric,
alphanumeric, and color codification. As the term suggests, in numeric codification numbers are
used, while in the alphanumeric system both letters and numbers are used.

168
A proper system of material codification serves the following purposes:
 Proper identification of items by all departments concerned;
 Avoiding the use of long descriptions of items;
 Avoiding duplicate stocks under different descriptions;
 Material accounting and control;
 Ensuring receipt and issue documents are posted in appropriate records;

 Helps in the mechanization of records.

4.1.3.8. Computerization

Computers are being used increasingly in the application of construction materials management.
They find wide application in almost all the functions of materials management, including:
 Forecasting the prices of materials based on past data and analyzing past trends,
 Planning different materials using the construction schedule, one can work out the
materials schedule quite easily using computers.
 Developing the specification of materials: one can refer to past specifications stored in
the computer and make suitable adjustments depending on the requirement of the project
 Purchasing of materials: it is now possible to float inquiries for different materials
online as well as invite bids from different suppliers;
 Preparing a comparative statement and finalizing the supplier;
 Inventory control: computers can play an important role in decision-making as far as
economic order quantity is concerned. An inventory analysis using different methods
illustrated elsewhere can be easily performed using computers

4.1.3.9. Source Development (Vendor Development)

Managing vendors or suppliers is an important issue. It is not an easy task to manage a large
number of vendors. Thus, it is better to have only a manageable number of vendors. It is also
necessary to develop a long-term relationship with the vendors. It has become imperative and
169
strategically important to manage vendors by considering them as partners in the business.
Initiatives such as vendor-managed inventory (VMI) can play an important role in containing
and managing costs. To trace new sources of supply and develop cordial relations with them to
ensure continuous, reliable, and quality material supply at reasonable rates, has become an
important item on the agenda of professional materials managers.

4.1.3.10. Disposal

Every year, old and used items that are not economical to use have to be disposed off in a
planned way. For this, the quantity and quality of materials to be disposed off should be
assessed. The reusable items from the scrap can be retained for further use. The remaining scrap
is disposed off either by selling it to some scrap dealer or through the process of floating inquiry,
collecting quotations, and awarding it to the highest bidder. Normally, while disposing of scraps,
payment is collected in advance from the scrap buyer.

4.1.4. Inventory Management

It can be recalled that inventory is a usable but idle resource. The problem of inventory
management is of maintaining an adequate supply of something to meet an expected demand
pattern for a given financial investment. This could be raw materials, work in progress, finished
products, spares, and other indirect materials. Inventory is one of the indicators of management
effectiveness on the materials management front. The inventory turnover ratio (annual demand/
average inventory) is an index of business performance. A soundly managed organization will
have a higher inventory turnover ratio, and vice versa. Inventory management deals with the
determination of optimal policies and procedures for the procurement of commodities. Since it is
quite difficult to imagine a real work situation in which the required material will be made
available at the point of use instantaneously, maintaining inventories becomes almost necessary.

170
4.1.4.1. Inventory-Related Cost

The inventory-related cost has the following broad cost components . The three types of costs are
the most commonly incorporated costs in inventory analysis, though there may be other cost
parameters relevant in such an analysis including inflation and price discounts. These are
explained in the following sections.
A. Cost of carrying inventories (holding cost)
This is expressed as an item held in stock/unit time. This is the opportunity cost of blocking
material in the non-productive form as inventories. Some of the cost elements that comprise
carrying costs are the cost of blocking capital (interest rate); cost of insurance; storage cost; and
cost due to obsolescence, pilferage, deterioration, etc. It is generally expressed as a fraction of
the value of the goods stocked per year. For example, if the fraction of the carrying charge is 20
percent per year and a material worth ` 1,000 is kept in inventory for one year, the unit carrying
cost will be 200/year. It is obvious that for perishable items, the attributed carrying cost will be
higher.
B. Cost of incurring shortages (stock-out cost)
It is the opportunity cost of not having an item in stock when one is demanded. It may be due to
lost sales or backlogging. In the case of backlogging (or back-ordering), the order is not lost but
is backlogged, to be cleared as soon as the item is available in stock. In most sales, the order is
lost. Both cases are characterized by demand, penalty cost, emergency replenishment, loss of
goodwill, etc.
C. Cost of replenishing inventories (ordering cost)
This is the amount of money and efforts expended in the procurement or acquisition of stock. It
is generally called ordering cost. This cost is usually assumed to be independent of the quantity
ordered because the fixed-cost component is generally more significant than the variable
component.

4.1.4.2. Functions of Inventory

Inventory is a necessary evil because it aims at absorbing the uncertainties of demand and supply
by ‘decoupling’ the demand and supply sub-systems. Thus, an organization may be carrying
inventory for the following reasons:

171
 Demand and lead-time uncertainties necessitate the building of safety stock (buffer stock)
to enable various sub-systems to operate somewhat in a decoupled manner. It is obvious
that the larger the uncertainty of demand and supply, the larger will have to be the amount
of buffer stock to be carried for a prescribed service level.
 Time lag in deliveries also necessitates the building of inventories. If the replenishment
lead times are positive, then stocks are needed for system operation.
 Cycle stocks may be maintained to get the economics of scale so that total system cost due
to ordering, carrying inventory, and backlogging is minimized. Technological requirements
of batch processing also build up cycle stocks.
 Stocks may build up as pipeline inventory or work-in-progress inventory due to the
finiteness of production and transportation rates. This includes materials being worked on
or moving between work centers, or materials in transit to distribution centers and
customers.
 When the demand is seasonal, it may become economical to build inventory during periods
of low demand to ease the strain of peak period demand.
 Inventory may also be built up for other reasons such as quantity discounts being offered
by suppliers, discount sales, an anticipated increase in material price, and the possibility of
future non-availability.

Different functional managers of an organization may view inventory from different points,
leading to conflicting objectives. This calls for an integrated systems approach to the planning of
inventories so that conflicting objectives can be scrutinized to enable the system to operate at
minimum total inventory-related costs both explicit costs such as purchase price as well as
implicit costs such as carrying, shortage, transportation, and inspection costs.

4.1.4.3. Inventory Policies

In this section, we discuss (a) the Lot size reorder point policy, (b) the Fixed order interval
scheduling policy, (c) the Optional replenishment policy, and (d) the Two-bin system. In
practice, there may be other policies that may be special cases of the policies mentioned or

172
maybe a combination of these policies. Some of the factors affecting the choice of an inventory
policy are the nature of the problem; the usage value of an item; and situational parameters.
A. Lot size reorder point policy
The inventory status is continuously reviewed and as soon as the inventory level falls to a
prescribed value called ‘reorder point’, a fresh replenishment order of fixed quantity known as
EOQ is placed. This is one of the very classical types of inventory policies. The decision
variables for the design of policy are Lot size and reorder point.

Figure 4.1: Typical stock balance under Lot Size Reorder Point Policy

B. Optional replenishment policy


This policy is also known as (s, S) policy. The status of the stock is periodically reviewed and
maximum stock level (S) and minimum stock level (s) are prescribed. At the time of review, if
the stock on hand is less than or equal to s, an order of size Q is placed so that stock on hand plus
the order equals the maximum stock level S. If stock on hand at review is higher than s, no order
is placed and the situation is reviewed at the next review period.

173
Figure 4.2: Typical stock balance under (s, S) policy

C. Fixed order interval scheduling policy


The time between the consecutive replenishment orders is constant. A maximum stock level (S)
is prescribed and the inventory status is reviewed periodically with a fixed interval (T). At each
review, an order of size O is placed, which takes the stock on hand plus an order equal to the
maximum stock level. The decision variables for the design of policy are S, the maximum stock
level, and T, the review period. The order quantity could vary from period to period. In this
policy, when the level of stock on hand is high at review, a smaller-size replenishment order is
placed.

174
Figure 4.3 Typical stock balances under Fixed Order Interval Scheduling
Policy

D. Two-bin system
This system is simple to operate and easy to understand. There are two bins kept full of items.
An item from the first bin is used first. The moment the first bin is exhausted, an order is placed
for items and the second bin acts as a buffer or safety cushion. Figure 11.5 shows all the stages
of this system in a schematic manner which is self-explanatory.

175
Figure 4.4 Schematic representations of different stages in two bin system

4.1.4.4. Selective Inventory Control

A list of items used in any typical construction can be found in figure 4.5. The figure lists out the
materials related to a typical construction project, in addition to commonly used small tools and
equipment. These tools are used by craftsmen and skilled labor such as masons, carpenters, steel
fitters, painters, welders, plumbers, and electricians.

176
Figure 4.5 Lists of materials related to a typical construction project

There is a large variety of items stocked by a project site and applying scientific inventory
control for all these items is neither feasible nor desirable. Since applying inventory control
across all items may render the cost of inventory control more than its benefits, it may prove to
be counter-productive. Inventory control has to be exercised selectively.

Depending upon the value criticality and usage frequency of an item, we may have to decide on
an appropriate type of inventory policy. Selective inventory control, thus, plays a crucial role so
that we can apply our limited control efforts more judiciously to the more significant group of

177
items. In selective control, items are grouped in a few discrete categories depending upon value,
criticality, and usage frequency. Table 4.1 shows some of the ways to make such groupings. This
type of grouping may well form the starting point for scientific inventory management in an
organization.

Table 4.1 some commonly adopted inventory-control policies

Name of Expansion Basis of classification Remarks


the policy

VED Vital, essential, and It is based on the criticality The classification depends on the
desirable of the item, which is consequences of material stock-out
classified into three when demanded.
categories.

FSN Fast, slow, and It is based on the It helps control obsolescence.


normal consumption rate of the
inventory.

HML High, medium, and It is based on the unit price It is mainly used to control the
low of the material. inventory of purchased material.

XYZ Value of balance It is used for classifying Its main use is in the review of
stocks very high materials in storage. inventory.

SDE Scarce, difficult, It is based on the level of It is useful in lead-time analysis and
and easy to obtain difficulty in the procurement decisions related to the procurement
of inventory. of purchasing strategies.

GOLF Government, It is based on the inventory. It is useful for decisions related to


ordinary, local and the procurement strategy.
foreign

HML High, medium, and It is based on the prices of It is useful for delegating
low price materials. purchasing responsibilities.

A. ABC Analysis

This is based on Pareto’s Law, which says that in any large group there are significant few and
insignificant many. For example, only 20 percent of the items may be accounting for 80 percent

178
of the total material cost procured by a construction organization. Here, the 20 percent constitute
the significant few that require utmost attention.
To prepare an ABC-type curve, we may follow a simple procedure:
1. Different materials required for the project are identified and their estimated quantities are
worked out. The quantity estimate could be based on either annual consumption or the
project’s total requirement.
2. The unit rates of materials are estimated.
3. The usage values for each of the materials are obtained by multiplying the estimated quantities
and their unit rates. These values are converted into a percentage of the total annual usage cost
or total project cost.
4. The percentage usage cost for each of the materials is arranged in the descending order of their
ranking, starting with the first rank, i.e., highest to lowest usage value. The cumulative
percentage usage value is also calculated.
5. A curve as shown in Figure 4.6 is plotted, and points on the curve at which there are
perceptible sudden changes of slopes are identified. In the absence of such sharp points, cut-off
points corresponding to the top 10 percent and the next 20 percent or so are marked as a
general indicator of A, B, and C types of materials.
6. According to an empirical approach, ‘A’ class items account for about 70 percent of the usage
value, ‘B’ class items for about 20 percent of the usage value, and ‘C’ class items for about 10
percent of the usage value. In terms of numbers, ‘A’ class items constitute about 10 percent of
total items, ‘B’ class items about 20 percent of total items, and ‘C’ class items about 70 percent
of total items. These percentages are indicative only and can vary depending on several factors.

Upon classification of materials into A, B, and C types, suitable inventory policies can be
decided. Corresponding to each type of materials, the implications on inventory policy are
mentioned below:

Item type ‘A’


The salient features are:
 Accurate forecast of quantities needed;
 Involvement of senior level for purchasing;

179
 Ordering is on a requirement basis;
 Inquiries for procurement need to be sent to a large number of suppliers;
 A strict degree of control is required, preferably monitoring every week;
 Low safety stock is needed.

Figure 4.6 ABC analysis

Item type ‘B’


The salient features are:
 Approximate forecast of quantities needed
 Requires the involvement of the middle level in purchasing
 Ordering is an EOQ basis
 Inquiries for procurement need to be sent to three to five reliable suppliers
 A moderate degree of control is required, preferably monitoring every month

180
 Moderate safety stock needed

Item type ‘C’


The salient features are:
 No need for forecasting; even a rough quantity estimate is sufficient
 Junior-level staff is authorized to order purchase
 Bulk ordering is preferred
 Quotations from even two to three reliable suppliers are sufficient
 A relatively relaxed degree of control is sufficient, and monitoring can be done every
quarter
 Adequate safety stock can be maintained.

B. VED Analysis
This analysis attempts to classify items into three categories depending on the consequences of
material stock-out when demanded. As stated earlier, the cost of shortage may vary according to
the seriousness of such a situation. Thus, the items are classified into V (vital), E (essential), and
D (desirable) categories. Vital items are the most critical having an extremely high opportunity
cost of shortage and must be available in stock when demanded. Essential items are quite critical
with a substantial cost associated with shortage and should be available in stock by and large.
Desirable groups of items do not have very serious consequences if not available when
demanded, but these can be stocked items.

The percentage risk of shortage with the vital group of items has to be kept quite small and thus
calling for a high level of service. With the ‘essential’ category we can take a relatively higher
risk of shortage, and for the ‘desirable’ category, even higher. Since even a C-class item may be
vital or an A-class item may be desirable, we should carry out a two-way classification of items
grouping them into nine distinct groups as A-V, A-E, A-D, B-V, B-E, B-D, C-V, C-E, and C-D.
We can then determine the aimed service level for each of these nine categories and plan for
inventories accordingly.

181
The vital group comprises those items for the want of which the production will come to a stop.
For example, power in the factory. The essential group features those items for whose non-
availability the stock-out cost is very high. The desirable group contains items whose no
availability causes no immediate loss of production; the stock cost involved is very less and their
absence may only cause minor disruption in the production for a short time.
The steps used for classifying materials as vital, essential, and desirable are given below:

Step 1: Factors such as stock-out case, lead time, nature of items, and sources of supply are
identified and considered for VED analysis.
Step 2: Assign points or weights to the factors according to the importance they have to the
company, as shown above.
Step 3: Divide each factor into three degrees and allocate points to each degree.
Step 4: Prepare a categorization plan to provide a basis for the classification of items: for
example, items scoring between 100 and 160 can be classified under desirable items;
items between 161 and 230 can be classified under essential items, and items between
231 and 300 can be classified under vital items.
Step 5: Specify the degree and allocate weights to all the factors.
Step 6: Evaluate and find the final score for every item, and specify the type of item.

C. FSN Analysis
Not all items are required with the same frequency. Some materials are required quite regularly,
some are required very occasionally, and yet some others may have become obsolete and might
not have been demanded for years together. FSN analysis groups them as fast-moving, slow-
moving and non-moving (dead stock), respectively.

Inventory policies and models for the three categories have to be different. Most inventory
models in the literature are valid for fast-moving items exhibiting a regular movement
(consumption) pattern. Many spare parts come under the slow-moving category, which has to be
managed on a different basis. For non-moving dead stock, we have to determine optimal stock
disposal rules rather than inventory provisioning rules. Categorization of materials into these

182
three types on value and critical usage enables us to adopt the right type of inventory policy to
suit a particular situation.

‘F’ items are those items that are fast-moving: i.e., in a given time, say, a month or a year, they
have been issued several times. However, ‘fast-moving’ does not necessarily mean that these
items are consumed in large quantities.
‘S’ items are those items that are slow-moving: in the sense that in the given time they have been
issued in a very limited number.
‘N’ or non-moving items are those that are not at all issued for a considerable time.

For example, for efficient operations, it would be necessary that fast-moving items are stored as
near as possible to the point of issue, for these to be issued with a minimum of handling. Also,
such items must be stored at the floor level, avoiding higher heights. Thus, if the items are slow-
moving or issued once in a while in a given period, they can be stored in the interior of the stores
and even at greater heights because the handling of these items becomes rare. Further, the stores
in-charge must know about non-moving items for various reasons mentioned below:
1. Non-moving items mean unnecessary blockage of money which affects the rate of returns of
the company.
2. Non-moving items also occupy valuable space in the stores without any usefulness and,
therefore, it becomes necessary to identify these items and find reasons for their non-moving
status. If justified, the recommendation may be made to top management for their speedy
disposal so that company operations are performed efficiently.

4.1.4.5. Inventory Models

There are several computer-based analytical inventory models available (such as the economic
order quantity [EOQ] model), most of which can generate economic purchase orders, shipping
orders, delivery notes, and invoices. Most models claim to improve management control by
reducing inventory-holding costs without loss of customer service. The basic philosophy behind
these models is to use a trade-off analysis by comparing the cost of inventory holding versus the
cost of ordering. The most popular inventory models are in the following section.

183
A. Economic Order Quantity (EOQ) Model
The EOQ model provides answers on how much to order. Figure 11.7 shows the behavior of the
EOQ model. The reorder point R and the quantity to be ordered, Q, are shown in the figure, as is
the lead time L. The ordered quantity derived from this model is known as economic order
quantity, EOQ.

It is usually less expensive to purchase (and transport) or produce a bunch of material at once
than to order it in small quantities. If orders for large quantities are specified, there will be fewer
orders placed. For purchasing, this means that quantity discounts and transportation efficiencies
may be realized. The other side of the coin, however, is that larger lot sizes result in more
inventory, and inventory is expensive to hold. EOQ model attempts to specify a balance between
these opposing costs.

This aspect is shown graphically in Figure 4.7, where it is clear that there is a decrease in cost
associated with an increase in order quantity, while there is an increase in cost with an increase
in inventory.

Figure 4.7: Total cost curve Economic Order Quantity (EOQ) Model

184
The total cost is given by the sum of inventory-carrying cost and ordering cost.

Total cost (TC) = Ordering cost + Carrying cost

The following notations are used to develop the EOQ model:


D = Demand rate; unit/year I = Inventory-carrying charges per year
A = Ordering cost; Dollar /order H = Annual cost of carrying inventory/U. item
C = Unit cost; Dollar /unit of item Q = Order quantity; the number of units per lot

It is assumed that demand is at a uniform rate. Thus, the average inventory required would be:

(0+Q) = Q throughout the year and,


2 2

The total number of orders placed would be D/ Q per year.

Order cost per year = Number of orders placed per year x Cost per order = A x D
Q

Carrying cost per year = Order quantity x Unit cost of item x Annual cost to carry
2

Where H=CxI

Using the notations mentioned above, we can write the expression of TC as:
TC = A x D + H x Q
Q 2

For optimum Q, one needs to find the particular value of Q which will minimize total cost. This
can be done by differentiation, and one gets:

EOQ=√ 2 x Order cost x Demand / Inventory carrying cost

EOQ=√ 2 x A xD /I xC

Some of the observations that are clear from the above expressions are:
 The more the demand per year, the larger the order quantity.

185
 The higher the order cost, the larger the order quantity.
 The more expensive the item, the smaller the order quantity.
 The higher the carrying cost, the smaller the order quantity.
The derivation of EOQ is based on several assumptions such as:
 Demand is deterministic and continuous at a constant rate.
 The process continues infinitely.
 No constraints are imposed on quantities ordered, storage capacity, budget, etc.
 Replenishment is instantaneous (the entire order quantity is received all at one time as
soon as the order is released).
 All costs are time-invariant.
 There are no shortages of items.
 The quantity discounts are not available.
 There is a negligible or deterministic lead time

In case the lead time is varying, one has to keep safety stock or buffer stock. Normally, the
service level that is expected will decide the safety stock. If one keeps a very small quantity as
safety stock, there is a danger that stock-out may occur. On the other hand, large safety or buffer
stock may result in large inventory-carrying costs. Thus, safety stock will be decided based on
the service level desired. The following relations are important and should be noted.

 Reorder Point = Demand or usage per period x Lead time


 Average Inventory Carried = Order Quantity + Safety Stock
2
 Transit Inventory = Days in transit x Inventory carried per day
 The higher the lead time, the more will be the safety stock, and vice versa. In general, the
safety stock varies with the square root of lead time (assuming all other factors as constant).
For example, if the lead time for an item is reduced by a factor of 4, then the safety stock will
be reduced by a factor of 2.

186
B. Effect of Uncertainty
The following table can be used to find the area under the curve
on Demand from the central line to any Z-value up to 3. To determine the area
Generally, demand is never under the curve between 0 and 1.35, start at the row for 1.3, and read
uniform throughout the year. In along until 1.35. The value corresponding to Z = 1.35 is 0.4115.
case, the demand has a mean Dm
and standard deviation Sd, the reorder point is expressed as given below:

Reorder point=D m x Lead time+ Z x Sd x √ Lead Time

Where Z is the standard normal variety for a given service level. To find the value of Z, one can
use the normal distribution table provided in table 4.2 of the text.

Table 4.3 Standard Normal Distribution

0.00 0.01 0.02 0.03 0.04 0.05 0.06 0.07 0.08 0.09
0.0 0.0000 0.0040 0.0080 0.0120 0.0160 0.0199 0.0239 0.0279 0.0319 0.0359
0.1 0.0398 0.0438 0.0478 0.0517 0.0557 0.0596 0.0636 0.0675 0.0714 0.0753
0.2 0.0793 0.0832 0.0871 0.0910 0.0948 0.0987 0.1026 0.1064 0.1103 0.1141
0.3 0.1179 0.1217 0.1255 0.1293 0.1331 0.1368 0.1406 0.1443 0.1480 0.1517
0.4 0.1554 0.1591 0.1628 0.1664 0.1700 0.1736 0.1772 0.1808 0.1844 0.1879
0.5 0.1915 0.1950 0.1985 0.2019 0.2054 0.2088 0.2123 0.2157 0.2190 0.2224
0.6 0.2257 0.2291 0.2324 0.2357 0.2389 0.2422 0.2454 0.2486 0.2517 0.2549
0.7 0.2580 0.2611 0.2642 0.2673 0.2704 0.2734 0.2764 0.2794 0.2823 0.2852
0.8 0.2881 0.2910 0.2939 0.2967 0.2995 0.3023 0.3051 0.3078 0.3106 0.3133
0.9 0.3159 0.3186 0.3212 0.3238 0.3264 0.3289 0.3315 0.3340 0.3365 0.3389
1.0 0.3413 0.3438 0.3461 0.3485 0.3508 0.3531 0.3554 0.3577 0.3599 0.3621
1.1 0.3643 0.3665 0.3686 0.3708 0.3729 0.3749 0.3770 0.3790 0.3810 0.3830
1.2 0.3849 0.3869 0.3888 0.3907 0.3925 0.3944 0.3962 0.3980 0.3997 0.4015
1.3 0.4032 0.4049 0.4066 0.4082 0.4099 0.4115 0.4131 0.4147 0.4162 0.4177
1.4 0.4192 0.4207 0.4222 0.4236 0.4251 0.4265 0.4279 0.4292 0.4306 0.4319

187
1.5 0.4332 0.4345 0.4357 0.4370 0.4382 0.4394 0.4406 0.4418 0.4429 0.4441
1.6 0.4452 0.4463 0.4474 0.4484 0.4495 0.4505 0.4515 0.4525 0.4535 0.4545
1.7 0.4554 0.4564 0.4573 0.4582 0.4591 0.4599 0.4608 0.4616 0.4625 0.4633
1.8 0.4641 0.4649 0.4656 0.4664 0.4671 0.4678 0.4686 0.4693 0.4699 0.4706
1.9 0.4713 0.4719 0.4726 0.4732 0.4738 0.4744 0.4750 0.4756 0.4761 0.4767
2.0 0.4772 0.4778 0.4783 0.4788 0.4793 0.4798 0.4803 0.4808 0.4812 0.4817
2.1 0.4821 0.4826 0.4830 0.4834 0.4838 0.4842 0.4846 0.4850 0.4854 0.4857
2.2 0.4861 0.4864 0.4868 0.4871 0.4875 0.4878 0.4881 0.4884 0.4887 0.4890
2.3 0.4893 0.4896 0.4898 0.4901 0.4904 0.4906 0.4909 0.4911 0.4913 0.4916
2.4 0.4918 0.4918 0.4920 0.4922 0.4927 0.4929 0.4931 0.4932 0.4934 0.4936
2.5 0.4938 0.4940 0.4941 0.4943 0.4945 0.4946 0.4948 0.4949 0.4951 0.4952
2.6 0.4953 0.4955 0.4956 0.4957 0.4959 0.4960 0.4961 0.4962 0.4963 0.4964
2.7 0.4965 0.4966 0.4967 0.4968 0.4969 0.4970 0.4971 0.4972 0.4973 0.4974
2.8 0.4974 0.4975 0.4976 0.4977 0.4977 0.4978 0.4979 0.4979 0.4980 0.4981
2.9 0.4981 0.4982 0.4982 0.4983 0.4984 0.4984 0.4985 0.4985 0.4986 0.4986
3.0 0.4987 0.4987 0.4987 0.4988 0.4988 0.4989 0.4989 0.4989 0.4990 0.4990

For ready reference, the values of Z for some commonly used service levels are given in Table
4.3 the service level is the probability of having material in stock when the demand for this
material occurs in a construction project.

Table 4.3 Values of Z for different service levels

Service level 90% 92% 94% 95% 96% 98% 99%


Z 1.29 1.41 1.56 1.65 1.75 2.05 2.33

Example 4.1
A construction material trading company receives a total of 200 as annual demand for steel
reinforcement. The annual cost of carrying per unit of reinforcement is 2,000, and the cost to
place an order is $25,000. What is the economic order quantity?

Solution
The economic order quantity is computed using the expression
EOQ=√ 2 x Order cost x Demand / Inventory carrying cost

EOQ=√ 2 x 25,000 x 200/2000


Thus, economic order quantity = 70.7 t per order.

188
Example 4.2
A construction company purchases $10,000 bags of cement annually. Each bag of cement costs
$200 and the cost incurred in procuring each lot is $100. The cost of carrying is 25 percent. What
is the most economic order quantity? What is the average inventory level?

Solution
Given, Unit item cost (C) = $200
Ordering cost = $100 per order
Annual usage = 10,000 units
Carrying rate = 25% and to find, EOQ
EOQ=√ 2 x A x D/ Ix C
EOQ=√ 2 x 100 x 10,000/0.25 x 200
EOQ=√ 40,000
EOQ=200 units . Hence, the order quantity will be 200 units and on average, 200/2 = 100
units in inventory will be carried.

Example 4.3
For Example 4.2, if the lead time of procuring cement is two weeks, determine the reorder point.

Solution
The lead time given is two weeks. So, the weekly usage is 10,000 units/52 weeks 5 192 units.
The reorder point will be = 192 units x 2 weeks = 384 units. That is, place an order for cement
whenever the stock reaches the level of 384 units and therefore, the order quantity will be 200
units.

Example 4.4
A shop dealing in construction goods has seven different items in its inventory. The average
number of units of each of these items held in the store along with their unit costs is given in
Table 4.4. The shopkeeper has decided to employ the ABC inventory system. Classify the items
in A, B, and C categories.

189
Table 4.4 Data for Example 4.4

Item The average number of units Average cost per unit in inventory (in $)
1 10,000 121.50
2 10,000 100
3 24,000 14.50
4 16,000 19.75
5 60,000 3.10
6 50,000 2.45
7 30,000 0.50

Solution
From Table 4.5, it is clear that the total number of units stored in the shop is $200,000 and the
cost is equal to $ 32.025 lakh. The total cost of inventory is worked out by summing up the cost
of storing the given numbers of all seven items. The percentage share of each item and costs are
calculated as shown in Table 4.5. From these two values, cumulative percentage values are
worked out.

Table 4.5 Computation details for Example 4.4

Item Units % of total Cumulative % of Unit cost Total cost % of the Cum. % of the
the total item ($) ($) total cost total cost
1 10,000 5.00 5 121.50 12.15 37.9 37.9
2 10,000 5.00 10 100.00 10.00 31.2 69.2
3 24,000 12.00 22 14.50 3.48 10.9 80.0
4 16,000 8.00 30 19.75 3.16 9.9 89.9
5 60,000 30.00 60 3.10 1.86 5.8 95.7
6 50,000 25.00 85 2.45 1.225 3.8 99.5
7 30,000 15.00 100 0.50 0.15 0.5 100.0
Total 200,000 100.00 32.025 100.00

And a graph as in Figure 4.4 is drawn between the cumulative percentage of cost and the
cumulative percentage of numbers. From the graph, it is clear that about 70 percent of the cost is

190
consumed in 10 percent of the inventory items. These are for items 1 and 2. Thus, items 1 and 2
are Class:

Figure 4.8: Cumulative percentage of cost and numbers for Example 4.4

4.2. Equipment Management

4.2.1. Introduction

Construction equipment is one of the very important resources of the modern-day construction
industry, especially in infrastructure projects. Such projects utilize equipment for most of the
works including earthmoving operations, aggregate production, concrete production, and its
placement, etc. One cannot think of any major construction activity without the involvement of
construction equipment.

There are different types of construction equipment suitable for different activities in a
construction project. The choice of construction equipment defines the construction method,
which in a way leads to the determination of time and cost for the project. To select the right
equipment to perform a specific task at the least cost, it is essential to know the features of

191
construction equipment including its rate of production and the associated cost to operate the
equipment.

In this section, major types of equipment have briefly been introduced. For a contractor, the
decision to employ a particular type of equipment is influenced by the contracts. They enter with
the owners, the site condition, and the location of the project site. Specifications indicating the
capabilities and the different features of construction equipment are provided by manufacturers.

The factors to be considered for the selection of some important equipment have been discussed
and an effort has also been made in this section to cover topics such as financial issues involved
in the purchase of equipment, issues involved in choosing equipment, accounting for assets
generated, and considerations in the computation of depreciation of equipment, using primarily a
case-study approach. Certain aspects of the implications of the above factors on tax calculation
are also covered.

4.2.2. Advantages of using construction equipment

There are many advantages of construction equipment in the construction industry some of them:

 Easy for time management.


 Easy for budget allocation.
 Flexibility in planning.
 Less number of people deal with it.
 Well, construction method.
 Helps to defend against delay.

4.2.3. Classification of Construction Equipment

Construction equipment’s have been classified in various ways.


I. The classification could be based on the function equipment performs., equipment
classification, based on functional (material-loading function, material-transporting function)

192
are grouped into:
a. Power units,
b. Prime movers, which make it possible for the equipment to operate,
c. Tractors,
d. Material-handling equipment and
e. Material-processing equipment.
II. It could be based on the operation in which the equipment is used. Based on this
classification could have:
a. Equipment used for loosening and moving the materials found in their natural state
(equipment under this class includes compressors, pumps, excavators, earthmoving
equipment, trenchers, and conveying and hauling equipment);
b. Equipment used for processing the materials, such as aggregate, concrete, and asphalt
production equipment;
c. Equipment used for transporting the processed materials; and
d. Equipment used for placing finished materials.
III. Another basis is the purpose, whether general-purpose or special-purpose equipment.
Based on general-purpose equipment can have earthwork equipment, material-hoisting
equipment, concreting equipment, etc., while special-purpose equipment can have bridge-
construction equipment, marine equipment, coffer dams and caissons equipment, large-
diameter pipe-laying equipment, and so on. Classification based on the purpose could be
company-specific. For example, if a construction company is into tunnel construction, the
tunnel-boring machine could be the general-purpose equipment for this company, while it
may be special-purpose equipment for a company that is mostly into building construction.

In the following sections, we briefly describe some common construction equipment that can be
seen on most construction sites these days.

4.2.3.1. Earthwork Equipment

A brief description of the equipment used in earthwork is given in Table 4.6 and they are shown
in Figure 4.9. The selection of earthmoving equipment is mainly dependent on the quantities of
193
material to be moved, the available time to complete the work, the job conditions, the prevailing
soil types, the swell and compaction factors, etc. The job conditions include factors such as soil
conditions, availability of loading and dumping area, accessibility of the site, traffic flows and
weather conditions at the site and the type of supervision and the length of working day and shift
work are also important for the selection of a particular type of earthmoving equipment.
The decision to choose between track-mounted earth-moving equipment and a wheel-mounted
one depends to a large extent on the prevailing job conditions.

To plan the number of earthwork equipment needed, the planner first determines the following:
1. The suitable class of equipment for earthwork. For example, if the soil to be excavated is
loose and marshy, and bulk excavation is involved in the project, one may opt for a
dragline. The planner needs to be familiar with different classes of equipment used in
earthwork and their suitability of working in different conditions
2. The appropriate model of equipment within that class. For a given class of equipment, be it
an excavator, a scraper, or a dragline, manufacturers usually come up with different
models with different performance characteristics such as the payload of the bucket and
speeds obtainable on different soil resistances. For example, draglines come in different
capacities ranging from 0.38 m3 to 3.06 m3; scrapers in capacities ranging from 8 m3 to 50
m3; and so on
3. The number of equipment needed for the project to carry out the given quantity, once the
model with suitable payload capacity is decided, the only thing we need to determine is
the cycle time
4. The number of associated equipment required to support the main equipment

Table 4.6 Earthwork equipment

Description Purpose Application Remarks Capacity


Backhoe For excavation below theCutting of trenches,Suitable for heavyStruck bucket
ground (lower elevation) pits, etc., leveling and positive cutting capacity 0.38 m3
loading to 3.25 m3
Shovel orFor excavation above itsFor cutting and Suitable for heavy Struck bucket
front shovel track or wheel level loading positive cutting in all capacity 0.38 m3
types of dry soil to 3.25 m3
Dragline For bulk excavation in looseFor canals and pitsSuitable for loose soils, 0.38 m3 to 3.06

194
soils below its track level excavation, cutting marshy land, and areas m3
and desalting of containing water
ditches
Clamshell orFor deep confined cutting inSuch as shafts, pits,Consists of a0.38 m3 to 3.06 m3
grab pits, trenches wells hydraulically controlled
bucket suspended from
a lifting arm
Dozers For moving earth up to aClearing and grubbingCan be track-mounted orBlade capacity
distance of about 100 m, sites, excavation of wheel-mounted 1.14 m3 to 6.11
shallow excavation, and surface earth, and m3
acting as a towing tractor maintaining roads
and pusher to scraper
machines
Roller For compaction of earth orUsed for large worksComes in differentFor (1) 8-10 t (2)
compactor other materials of highways, canals, varieties such as (1) 4-17 t (3) 11-25 t
and airports smooth-wheeled roller, (4) 2.5-11.5 t
(2) vibratory roller, (3) Capacity can be
pneumatic-tired roller, increased by
and (4) sheep-foot roller ballasting
Scraper For site stripping andComes in differentBest suited for haulSizes varying from
leveling, loading, hauling, varieties such as distances varying 8 m3 to 50 m3
and discharging over long towed scrapers, two- between 150 m and 900
distances axle scrapers, and m
three-axle scrapers
Dumper For horizontal transportationSuitable for hauling onComes in differentThe load capacity
of materials on and off sites softer sub-grades. varieties with front of 1 t to about
Large-capacity tipping, side tipping, or 80 t; 20 t is
dumpers are used in elevated tipping common for a
mines and quarries arrangements small dumper
Grader For spreading fill and fine-Grading and finishingGraders usually operate
trimming the subgrade. the upper surface of in the forward
Grader performs a follow- the earthen formations direction
up operation to scraping or and embankments
bulldozing

Articulated Trucks Backhoe Loaders Compactors Hydraulic Excavators

Motor Graders Multi Terrain Loaders Off-highway Tractors Off-Highway Trucks

195
Skid Steer Loaders Track Loaders Track-type Tractors Wheel Dozers

Paving Equipment Wheel Excavators Wheel Loaders Scrapers

Figure 4.9: Some common earthwork equipment

4.2.3.2. Concreting Equipment

The Selection of concreting equipment (see Table 4.7) can be complicated. The decision will
involve many issues that have to be analyzed. Look at the following factors mentioned:
• Site characteristics (boundary conditions, provision of temporary roads, noise limitations,
etc.)
• Equipment availability (local) whether the contractor owns that equipment
• Continuity of operation
• Effect of permanent work
• Weather conditions
• Temporary works
• Time restrictions
• Concrete specifications

Table 4.7 Concreting equipment


196
Description Purpose Capacity Remarks
3
Concrete For weighing and 20 m m /h to 250 Comes in different varieties such
batching and mixing a large m3 /h as stationary and mobile batching
mixing plant number of concrete plants
constituents
Concrete For mixing small Capacity could be Comes in different varieties such
mixers quantities of up to 200 l/batch for as non-tilting drum type and
concrete small mixers, and tilting drum type
constituents between 200 and
750 l/batch for large
mixers
Concrete For transporting 3 m3 to 9 m3 Capacity also depends on the
transit mixers concrete from permissible axle load
batching plant
Concrete For horizontal and 30 m3 /h for Direct acting pumps: output up
pumps, static vertical ordinary to 60m3 /h through 220 mm
or portable transportation of construction; can be delivery pipes. Concrete can be
large volumes of more than 120 m3 /h easily pumped up to a distance of
concrete in a short for specialized 450m horizontally or 50m
duration construction vertically.
Squeeze pumps: output up to 20
m3 /h through 75mm delivery
pipes. Concrete can be easily
pumped up to a distance of 90m
horizontally or 30 m vertically

197
Selection of Concrete-mixing equipment will depend on many factors such as the maximum and
the total output required in a given time frame, the method of transporting the mixed concrete,
and the requirement of discharge height of the mixer.

Concrete-placement equipment selection depends on factors such as the capacity of the vehicle,
the output of the vehicle, the site characteristics, the weather conditions, the operator’s
efficiency, the rental costs, and the temporary haul roads. It is important that the system that
guarantees minimum costs to be selected.

4.2.3.3. Hoisting Equipment

A. Hoists
Hoists are a means of transporting materials or passengers vertically using a moving level
platform. Materials hoists come in basically two forms (static and mobile models). The static
version consists of a mast or tower with the lift platform either cantilevered from the small
section mast or centrally suspended with guides on either side within an enclosing tower. Mobile
hoists usually do not need tying to the structure unless extension pieces are fitted, in which case
they are treated as cantilever hoists.

Passenger hoists, like the materials hoist, can be driven by petrol, diesel, or electric motor and
can be of a cantilever or enclosed variety. The cantilever type consists of one or two passenger
hoist cages operating on one side or both sides of the cantilevered tower; the alternative form
consists of a passenger hoist cage operating within an enclosing tower.

B. Cranes

198
A crane is a device used for hoisting and placing materials and machinery. It is used to facilitate
the handling of materials such as formwork, reinforcement, pipes, and structural steel items. The
use of cranes has greatly increased in the construction industry due mainly to the need to raise
the large and heavy prefabricated components often used in modern structures. Cranes are
available in different ranges and the choice of a crane is dependent on factors such as the weight
of the materials and machinery to be lifted, the overall dimension, the distance to which they are
to be placed, and the prevailing site condition. Construction cranes can be broadly classified
under two major categories as tower cranes and mobile cranes
Selecting between a mobile crane and a tower crane is one of the decisions faced by the planner.
This is followed by the selection of an appropriate model of mobile or tower crane. These
decisions are influenced by several factors. The additional cost associated with a tower crane
would include the cost of the design and construction of a suitable foundation, the cost of
erection and dismantling, etc.
The additional advantage of using tower cranes is that they provide an almost noise-free working
environment, and they can also work under high wind speed. Shapiro et al. (1991) point out that
“some of the tower cranes can operate in winds up to 72.5 km per hour, while mobile cranes are
designed for winds of about 32.3 km/h in conjunction with rated loads, which does not include
an allowance for wind on the lifted load.’ The use of a tower crane also enables the site
personnel to open the job so that any area of the project can be constructed at the same time as
any other area. There are different types of cranes, some of them: Tower Crane, Top-slewing
tower cranes, Bottom-slewing tower cranes, Mobile Crane, Truck cranes, Crawler cranes,
Rough-terrain cranes, All-terrain cranes, etc.

199
Derrick Crane

Figure 4.10 Different types of Cranes

4.2.4. Factors Behind the Selection of Construction Equipment

The selection of the appropriate construction equipment is an important part of job planning. A
planner has to choose the alternative that provides the best value from a cost and schedule
perspective. The selection of equipment for construction projects generally involves two classes
of factors or considerations: hard factors and soft factors.

Examples of hard factors include technical specifications of the equipment, physical dimensions
of the site and constructed facility, and cost calculations. As can be noticed, hard factors are
tangible. On the other hand, soft factors are mostly intangible, qualitative, and informal. Some
instances of soft factors are safety considerations, company policies regarding purchase/rental,
market fluctuations, and environmental constraints. However, soft factors influence decision-
making to a large extent.

Several researchers have worked to develop a method to assist in equipment selection and,
consequently, different models have evolved over the years. For instance, there are optimization
models, graphics models, and database-centered models to select tower cranes. Different expert
systems have also been developed to assist in equipment selection. Finally, there are artificial
intelligence-based models. Harris and McCaffer (2001) have developed their equipment
selection model based on multi-attribute decision-making. Dynamic programming-based models
have also been developed for equipment selection and many computer programs have been

200
developed to assist in the equipment-selection process. The specific decision variables for a
particular class of equipment selection are:

4.2.4.1. Economic Considerations

The economic considerations such as owning costs, operating labor costs, and operating fuel
costs of equipment are most important in the selection of equipment. Besides, the resale value,
the replacement costs of existing equipment, and the salvage value associated with the equipment
are also important.

4.2.4.2. Company-specific

The selection of equipment by a company may be governed by its policy on owning or renting.
While the emphasis on ‘owning’ may result in a purchase of equipment keeping in mind the
future requirement of projects, the emphasis on ‘renting’ may lead to putting too much focus on
short-term benefits.
This may explain the situation of a construction company opting for two 30 m 3 per hour batching
plants owned by the company and currently idle, instead of the required 45 m 3 per hour batching
plant that it can get on rent. The company wants to deal with the 30 m 3 per hour batching plant
keeping future projects in mind, even though going for the 45 m 3 /h batching plant option at
present may have been economical.

Further, if the company project forecast says that there will be a considerable number of projects
involving a particular type of construction equipment, says X, for the next couple of years, then
the decision would be to buy only this type of equipment. A construction company specializing
in a particular type of construction, such as tunneling, will tend to procure only the equipment
that is used in tunnel construction. The equipment-selection decision also depends on the amount
of outsourcing the company does to execute its projects. A company opting to subcontract a
majority of its project work will tend to keep low equipment assets.

201
4.2.4.3. Site Specific

Site conditions: both ground conditions, as well as climatic conditions, may affect the equipment
selection decision. For example, the soil and profile of a site may dictate whether to go for
crawler-mounted equipment or wheel-mounted equipment. If there is a power line at or in the
vicinity of the site, one may go for a fixed-base kind of equipment rather than a mobile kind of
equipment. Similarly, climatic conditions such as the presence of strong winds, visibility level,
and noise level may affect the equipment-selection decision.
Further, the access leading to the site may also affect the decision. Heavy traffic congestion near
a site may lead to a decision to produce the concrete at the site and, hence, select a concrete
batching plant rather than relying on ready-mix concrete. Similarly, if there is only a narrow road
leading to the site, it may have a bearing on the type of transportation equipment selected for the
site.

4.2.4.4. Equipment-specific

Construction equipment comes with high price tags. While it may be tempting to go for the
equipment with a low initial price, it is preferable to opt for standard equipment. Such equipment
is manufactured in large numbers by the manufacturers, and their spare parts are easily available,
which would ensure minimum downtime. Besides, they can also fetch good salvage money at the
time of their disposal. It is a general tendency to go for such equipment that can bring in
‘uniformity’ in the type of equipment that is already available with the company. For example, a
company would like to go for a uniform type of engine for different machines such as
excavators, dumpers, and tractors.

The size of the equipment selected is also an important consideration. Although the unit
production cost may be cheaper for equipment of large size, it is also true that large equipment
requires correspondingly larger sizes of matching equipment. Thus, downtime in one primary
unit may lead to downtime in dependent equipment also. A trade-off between unit production

202
costs versus size must be obtained, and as far as possible, equipment of similar sizes should be
selected for the project. Besides the above factors, the versatility (whether it can perform more
than one function) of equipment, the adaptability for future use, the experience with the
equipment, and the interaction with other equipment are also to be noted.

4.2.4.5. Client- and Project- Specific

The owner/client in a certain project may have certain preferences that are not in line with the
construction company’s preferred policies as far as equipment procurement is concerned. The
schedule, quality, and safety requirements demanded of a particular project may in some cases
force the company to yield to the demands of the client.

4.2.4.6. Manufacturer-specific

A construction company may prefer to buy equipment from the same manufacturer again and
again and that too from a specific dealer. This may be to bring uniformity in the equipment fleet
possessed by the company or because the company is familiar with the working style of the
manufacturer and the dealer. The long association may not only result in a cheaper price, but it
also ensures prompt services by the manufacturer concerning the company.

4.2.4.7. Labour Consideration

Shortage of manpower in some situations may lead to a decision in favor of procuring highly
automated equipment. Further, the selection of equipment may be governed by the availability or
non-availability of trained manpower. The company may not be inclined to select some
sophisticated equipment if it finds that there is limited availability of manpower to operate the
same. In the following sections, we briefly describe some common construction equipment that
can be seen on most construction sites these days.

4.2.5. Plant and Equipment Acquisition

203
A construction company can acquire a construction plant and equipment through:
 Cash or outright purchase,
 Hire purchase and
 Leasing.

The decision to choose between purchasing and renting is an important one, and it is governed
by several factors. A simplified form of this type of decision-making is illustrated through an
example.

Figure 4.11 All-terrain mobile cranes


In cases where the company has surplus funds available, it can invest in cash purchase of plants
and equipment provided such investment guarantees a return more than the minimum attractive
rate of return of the company. The purchase of equipment raises the asset of the company and
this entitles the company to certain tax incentives. The ownership of the plant and equipment is
with the company in the purchasing option. In the hire purchase option, the company agrees with
the financer of the plant and equipment.
The financer receives the specified rental from the company using the equipment during the
entire agreement period. The title of the equipment lies with the financer during this agreement
period. Upon the completion of the agreement period of equipment, the title is transferred in the
name of the company against some nominal amount of money. Through the hire purchase option,
the company can avoid large capital investment though it may be paying higher interest and,
thus, higher rental.

In the leasing option, the company pays the lease rentals to the lessor (one who owns the
equipment) for the use of equipment for a specified period. The title of the equipment never gets

204
transferred to the company using the equipment. These days, several business houses are in the
business of leasing construction equipment and, consequently, different variants of leasing with
several interesting features are appearing regularly.

4.2.6. Equipment Costs, Life and Replacement Analysis

Equipment costs comprise two separate components: ownership costs and operating costs.
Except for the one-time initial capital cost of purchasing the machine, ownership costs are fixed
costs that are incurred each year, regardless of whether the equipment is operated or idle.
Operating costs are the costs incurred only when the equipment is used. Each cost has different
characteristics of its own and is calculated using different methods. None of these methods will
give exact costs of owning and operating equipment for any given set of circumstances. This is
because of the large number of variables involved, which is because of the uncertain nature of
the construction business.
One should consider these estimates as close approximations while calculating ownership and
operating costs. However, these costs are divided into:
 Ownership costs,
 Operating costs,
 Overhead costs, and
 Profits

In using or renting out equipment thus, an equipment owner should properly evaluate the above
costs so that he/she recoups an equivalent value from his/her machine.

4.2.6.1. Ownership Cost

Ownership costs are fixed costs. Almost all of these costs are annual and include:
 Initial capital cost;
 Depreciation;
 Investment (or interest) cost;

205
 Insurance cost;
 Taxes;
 Storage cost.

4.2.6.2. Initial Cost

On average, the initial cost makes up about 25% of the total cost invested during the equipment’s
useful life. This cost is incurred for getting equipment into the contractor’s yard, or construction
site, and having the equipment ready for operation. Many kinds of ownership and operating costs
are calculated using initial cost as a basis, and normally this cost can be calculated accurately.
The initial cost consists of the following items:
 Price at factory + extra equipment + sales tax
 Cost of shipping
 Cost of assembly and erection

4.2.6.3. Depreciation

Depreciation represents the decline in the market value of a piece of equipment due to age, wear,
deterioration, and obsolescence. Here there is a highlight of the concept of depreciation. Assume
company X purchases a truck T, for use in its business, and for some reason, if the company
wants to sell it even the very next day, it is unlikely that the company will get a buyer who will
buy it for the price he (X) paid. In other words, the truck starts to lose value as soon as it is
driven out of the dealership.

Continuing with the example, as soon as it is bought, the truck becomes an operational asset for
X and has a certain value in the account books of the company. Now, the annual loss in the value
of the truck (on account of depreciation), too, should be appropriately reflected in the account
books, even though no physical transaction is taking place (that reduces the value of the truck),
i.e., the book value of the truck should change every year that it continues in X’s ownership.
Finally, a stage is reached when the truck may stop running and has no value to the business and

206
may be sold, scrapped, or written off.

Depreciation is the allocation of the cost of an asset over a while for accounting and tax
purposes.’ Another definition discusses depreciation in terms of a decline in the value of an asset
due to general wear and tear, deterioration, or obsolescence. It may be noted, however, that not
all assets depreciate with time. The land is one such example; it does not wear out like vehicles
or equipment and should be handled separately.

Although the above example of X buying a truck was drawn for a single asset, in general, a
company ‘owns’ various assets in the form of buildings, office furniture, machinery, equipment,
etc., all of which are acquired at a certain value, and they depreciate over time. In the end, they
cease to be of any monetary value, and can only be scrapped and a certain value extracted
(salvaged) from the scrap.

4.2.6.4. Depreciation and Taxation

As mentioned above, depreciation can be looked upon as an expenditure on which tax benefits
are often available. The following illustrative example has been included to show the close
relationship between depreciation expenses and tax liability.

Example 4.5
Given that the gross income is $200,000, and assuming that (i) depreciation expenses are fully
deductible from gross income and (ii) the income tax is 25 percent of the taxable income,
determine the tax liability in cases when (a) there are no depreciation expenses, and (b) there is
an expense of, say, $40,000 towards depreciation.

Table 4.8 Effect of depreciation expenses on tax liability Example 4.5

Case I Case II
Gross Depreciation Net income Tax payable Depreciation Net income Tax payable

207
income(a) expense (b) (c) = (a) -(b) @25% of (c) expense (b) (c) = (a) - (b) @ 25% of (c)
200,000 - 200,000 50,000 40,000 160,000 40,000

Table 4.8 summarizes the total and taxable incomes, and the tax liability in the two cases. It can
be seen that there is a net saving of $10,000 in taxes for the conditions given. Now, having
explained the implication of depreciation expenses on the tax liability, another related issue is
how the total amount of depreciation should be distributed over the service life of the asset.
Before discussing the commonly used methods, let us look at another simple illustrative
example.

Example 4.6
Let the total depreciation involved in an asset be $50,000 over two years. Assuming a gross
income of $100,000 each year, and taxes payable at a rate of 20 percent of the net income,
compare the tax liabilities in the following cases:

 Case I: Depreciation amounts in the first and second years are equal ($ 25,000)
 Case II: Depreciation amounts in the first and second years are $40,000 and $10,000,
respectively

Table 4.9 Effect of depreciation expenses on the pattern of tax liability


Example 4.6

Case I Case II
year Gross Depreciation Net income Tax payable Depreciation Net income Tax payable
income(a) expense (b) (c) = (a) - @25%of (c) expense (b) (c) = (a) - @25% of (c)
(b) (b)
1 100,000 25,000 75,000 15,000 40,000 60,000 120,000
2 100,000 25,000 75,000 15,000 10,000 90,000 18,000

4.2.6.5. Methods of Calculating Depreciation

208
From the qualitative framework given in Figure 9.10, it is clear that to deal with the issues
related to depreciation quantitatively, any method should appropriately account for the following:
(a). The purchase price or initial cost of the asset (P),
(b). Economic life or recovery period allowed for the asset (N), and
(c) Salvage value of the asset (E) or (S).
(d). Book value (BVM)
(e). Rate of depreciation (Rm)
In the Ethiopian context some of the more commonly used methods of Calculating Depreciation
are:

1. Straight-line Method

2. Sum of Years Digit Method

3. Declining Balance Method

1. Straight-line Method
Straight-line depreciation is considered to be the most common method of calculating
depreciating assets and is based on the assumption that the asset loses an equal amount of value
each year. This annual depreciation is calculated by subtracting the salvage value of the asset
from the purchase price and then dividing this number by the estimated useful life of the asset.

The computation of annual depreciation expense here requires three numbers, the initial cost of
the asset, its estimated useful life, and the salvage value at the end. For example, a truck
purchased for $500,000 and expected to have an economic life of ten years, and not have any
salvage value, has a total depreciation expense of $500,000, which is reported over the ten years
at the rate of $50,000 every year. As far as the book value of the truck is concerned, starting from
$ 500,000, it decreases by $50,000 every year and becomes zero over ten years.
Mathematically,
 Rm (Rate of depreciation) = 1/N
 Dm (Annual depreciation) = (P- E) x Rm

209
 BVM (Book value (end of year m) = BVM-1 - Dm

Example 4.7
For an asset having an initial cost of $200,000, and a salvage value of $50,000 at the end of the
economic life of 5 years, determine the rate of depreciation, total depreciation, annual
depreciation, and book value using the Straight line method.

Solution

Taking P, N, and E to be $200,000, 5 years, and $50,000, respectively, the rate of depreciation,
the total depreciation, and the annual depreciation can be calculated as follows:
A. Rate of depreciation (Rm) = 1/N = 0.2
B. Total depreciation (P - E) = (200,000 - 50,000) = 150,000
C. Annual depreciation (Dm) = (P - E) x Rm = 150,000 x 0.2 = 30,000
Using these results, Table 4.10 summarizes the change in the book value of the asset over the
five years.

Table 4.10 Tabular way of Calculation of depreciation using straight-line


method Example 4.7

Year (m) Opening bookAnnual Depreciation (Dm) Cumulative Closing book


value (BVM-1) Depreciation value (BVM)
0 0 0 0 200,000
1 200,000 30,000 30,000 170,000
2 170,000 30,000 60,000 140,000
3 140,000 30,000 90,000 110,000
4 110,000 30,000 120,000 80,000
5 80,000 30,000 150,000 50,000

Example 4.8

210
Assume the Purchasing cost of the equipment = $200,000, salvage value of the equipment =
zero, and useful life of the equipment= 5 Years, then determine the rate of depreciation, total
depreciation, annual depreciation, and book value using the Straight line method.
Solution

RM= 1/N = 1/5 = 0.2


DM= (P - F) x RM = yearly depreciation where, D1 = D2 = D3 = D 4 = D5
= (250,000-0)0.2
= $50,000

211
 Therefore, in the:
BV1 in 1st year = p - M (D1) = 250,000-50,000 = $200,000
2nd year BV2 = BV1- D2 = $200,000 -50,000 = $150,000
3rd year BV3 = BV2- D3 = 150.000 -50,000 = $100,000
4th year BV4 = BV3- D4 = 100,000-50,000 = $50,000
5th year BV5 = BV4- D5 = 50,000-50,000 = $0,000

Table 4.11 Tabular way of Calculation of depreciation using straight-line


method Example 4.8

Year (m) Yearly Depreciation (DM) Cumulative Book value


Depreciation (BVM)
0 0 0 250,000
1 50,000 50,000 200,000
2 50,000 100,000 150,000
3 50,000 150,000 100,000
4 50,000 200,000 50,000
5 50,000 250,000 0

2. Sum of Years Digit Method


One of the commonly used methods of calculating depreciation leads to a higher rate of
depreciation initially which reduce progressively. As illustrated in the Example above, such a
strategy could be useful to obtain greater tax benefits in the early years of the economic life of
the asset. Mathematically, the steps in the method can be summarized as follows:

 For an asset having an economic life of N years, determine the sum of years (SOY)

SOY = (1+2 +3 +….. N) = N (N+ 1)


2
 Determine the annual rate of depreciation (Rm)
RM = N – M + 1)

212
SOY
 Determine the total depreciation involved (D)
D = Total depreciation = (P - E)

 Determine the annual depreciation for the year m (Dm)


DM = (P - E) x Rm

 Determine the book value of the asset for the year m (BVM)
BVM = BVM-1 - DM

Example 4.9

For an asset having an initial cost of $200,000, and a salvage value of $50,000 at the end of the
economic life of 5 years, determine the rate of depreciation, total depreciation, annual
depreciation, and book value using the sum of digits method. Although the details of the simple
computations have been left out, Table 4.12 shows the annual depreciations and the opening and
closing book values of the asset using the sum of digits method. From Table 4.12, it can be seen
that though the total depreciation is the same in the SOY and straight-line methods (i.e.,
$150,000), the former method gives higher amounts in the first and second years ($50,000 and
$40,000, respectively). This reduces the book value of the asset and also gives higher tax
benefits. Of course, the book value at the end of the useful life ($50,000) is the same in both
cases.

Table 4.12 Calculation of depreciation using the sum of year’s digit method
Example 4.9

Year Opening The annual rate Annual Cumulative Closing


(m) book value of Depreciation Depreciation Depreciation book value
(BVM-1) (Rm) (Dm) (Dm) (BVM)
0 0 0 0 0 200,000
1 200,000 5/15 50,000 50,000 150,000
2 150,000 4/15 40,000 90,000 110,000
3 110,000 3/15 30,000 120,000 80,000

213
4 80,000 2/15 20,000 140,000 60,000
5 60,000 1/15 10,000 150,000 50,000

Example 4.10

Assume the Purchasing cost of the equipment = $200,000, salvage value of the equipment =
zero, and useful life of the equipment= 5 Years, then determine the rate of depreciation, total
depreciation, annual depreciation, and book value using the Sum of a year method.

Solution

SOY = N (N+1)/2 = 5(5+1)/2 = 15 and RM = N – m + 1) so, R1 = 5-1+1 = 0.33


SOY 15
R2 = 5 – 2 +1 = 4/15 R3 = 5 – 3 +1 = 3/15 R4 = 5 – 4 +1 = 2/15 R5 = 5 – 5 +1 = 1/15
SOY SOY SOY SOY

D1 = (P -F) (N - m + 1)/SOY D2 = (P -F) (N - m + 1)/SOY


= (250,000 – 0) (5 – 1 + 1)/15 = (250,000 – 0) (5 – 2+ 1)/15
= (250,000) 5/15 = 83,333 = (250,000) 4/15 = 66,666.67

BV1 = (P – D1) BV2 = (BV1 – D2)


= (250,000 – 83,333) = (166,666.67 – 66,666.67)
= 166,666.67 = 100,000

D3 = (P -F) (N - m + 1)/SOY D4 = (P -F) (N - m + 1)/SOY


= (250,000 – 0) (5 – 3 + 1)/15 = (250,000 – 0) (5 – 4+ 1)/15
= (250,000) 3/15 = 50,000 = (250,000) 2/15 = 33,333.33

BV3 = (BV2 – D3) BV4 = (BV3 – D4)


= (100,000 – 50,000) = (50,000 – 33,333.33)
= 50,000 = 16,667.67

D5 = (P -F) (N - m + 1)/SOY BV5 = (BV4 – D5)


214
= (250,000 – 0) (5 – 5 + 1)/15 = (16,667.67 – 16,667.67)
= (250,000) (1)/15 = 16,666.67 =0

Table 4.13 Tabular way of Calculation of depreciation using the sum of a year
method Example 4.10

Year (m) Yearly Depreciation (Dm) Cumulative Book value


Depreciation (BVM)
0 0 0 250,000
1 (250,000 – 0) (5 – 1 + 1)/15 = 83,333. 83,333 166,667
2 (250,000 – 0) (5 – 2 + 1)/15 = 66,667 150,000 100,000
3 (250,000 – 0) (5 – 3 + 1)/15 = 50,000 200,000 50,000
4 (250,000 – 0) (5 – 4 + 1)/15 = 33,333 233,333 16,667
5 (250,000 – 0) (5 – 5 + 1)/15 = 16667 250,000 0

3. Declining Balance Method

In this method, the depreciation for a given year is calculated based on the un-depreciated
balance (instantaneous book value), rather than the original cost. Further, the method does not
take into account any salvage value of the asset. The rate of depreciation, R m, in this case, can be
represented by a factor M/N, where M is a constant and N is the service life of the asset.
Frequently, M is taken as 2, and the rate of depreciation is 2/N. In such a case, the method is also
referred to as the ‘double declining balance (DDB) method’. It may be noted that the rate of
depreciation, in this case, is twice that of the straight-line method, with the additional condition
that the salvage value is taken as zero. However, in the case of a given finite salvage value; this
value becomes the lower bound of the book value. Mathematically, the formulation for the
double declining balance method can be represented as:

215
RM = 2/N
DM = BVM-1 x RM
BVM = BVM-1 - DM

Example 4.11

For an asset having an initial cost of $200,000, and a salvage value of $50,000 at the end of the
economic life of 5 years, tabulate the annual rate of Depreciation, annual depreciation, and the
book values of the asset using the double declining balance method.
Solution
In this case, given that the economic life of the asset is 5 years, the rate of depreciation (Rm) can
be taken as (2/N = 2/5 = 0.4). Now, this factor is applied to determine the annual depreciation
(Dm), and the book value of the asset is allowed to reduce till the value does not go below the
salvage value (E). While details of the computations have been left out, the final table showing
the annual depreciations and the opening and closing book values of the asset using the double
declining balance method are given in Table 4.14.
The decline in the value of the asset is even steeper here compared to the sum of years method,
and the value falls to the salvage level at the end of three years itself, whereas the economic life
of the asset itself is taken as five years. It may also be noted that the book value of an asset never
becomes zero when the depreciation is determined using the double rate declining balance
method.

Table 4.14 Tabular Calculation of depreciation using declining balance


method Example 4.11

Year Opening book The annual rate of Annual Closing book value
(m) value (BVM-1) Depreciation(Rm) Depreciation (Dm) (BVM)
0 0 0 0 200,000
1 200,000 0.4 80,000 120,000

216
2 120,000 0.4 48,000 72,000
3 72,000 0.4 28,800(22,000) 43,200(50,000)
4 50,000 0.4 Zero 50,000
5 50,000 0.4 Zero 50,000

Here, the value of the asset is not allowed to fall below the salvage value, and the amounts for
depreciation are adjusted accordingly. Thus, in this case, the D m calculated may be taken to give
an upper bound of the allowable depreciation, which should be adjusted according to the book
and salvage values.
Example 4.12
Assume the Purchasing cost of the equipment = $90,000, salvage value of the equipment =
$10,000, and useful life of the equipment= 5 Years, then determine the rate of depreciation,
depreciation, and book value using the declining balance method.
Solution
RM = 2 = 2/5 = 0.4
N

D1 = P X RM = 90,000 * 0.4 = $36,000 D2 = BV1 X RM = 54,000 * 0.4 = $21,600


BV1 = P - D1 = 90,000 – 36,000 = $54,000 BV2 = BV1 - D1 = 54,000 – 21,600 =
$32,400

D3 = BV2 X RM = 32,000 * 0.4 = $12,960 D4 = BV3 X RM = 19,440 * 0.4 = $7776


BV3 = BV2 - D3 = 32,400 – 12,960 = $19,440 BV4 = BV3 - D4 = 19,440 – 7776 = $11,664
D5 = BV4 X RM = 11664,000 * 0.4 = $4,666
BV5= BV4 – D5 = 11,766 – 4,666 = $6998

Table 4.15 Tabular Calculation of depreciation using declining balance


method Example 4.12

End of Year (m) Yearly Depreciation(Dm) Cumulative Book value of the


Depreciation(Dm) equipment(BVM)
0 0 0 90,000
1 0.4 x 90,000 = 36,000 36,000 54,000
2 0.4 x 54,000 = 21,600 21,600 32,400

217
3 0.4 x 32,400 = 12,960 12,960 19,440
4 0.4 x 19,440 = 7,776 7,776 11,664
5 0.4 x 11,664 = 6,998 0.4 Zero

This value is less than the salvage value


Thus, 6998 is changed to 10,000 (salvage value) and is equal to 11,664 – 10,000 = 1,664 which
the owner takes the amount equal to the depreciation plus interest from the equipment every
year.

4.2.6.6. Investment (Or Interest) Cost

Investment (or interest) cost represents the annual cost (converted into an hourly cost) of capital
invested in a machine. If borrowed funds are utilized for purchasing a piece of equipment, the
equipment cost is simply the interest charged on these funds. However, if the equipment is
purchased with company assets, an interest rate that is equal to the rate of return on company
investment should be charged. Therefore, the investment cost is computed as the product of
interest rate multiplied by the value of the equipment, which is then converted into cost per hour
of operation.
The average annual cost of interest should be based on the average value of the equipment during
its useful life. The average value of equipment may be determined from the following equation:

P = IC (n + Where IC is the total initial cost, P is the average value, and n is the useful
1) life (years).
2

This equation assumes that a unit of equipment will have no salvage value at the end of its useful
life. If a unit of equipment has salvage value when it is disposed of, the average value during its
life can be obtained from the following equation:
P = IC (n + 1) + S (n -1)
2n
where IC is the total initial cost, P is the average value, S is the
salvage value, and n is the useful life (years).
Example 4.13

218
Consider a unit of equipment costing $50,000 with an estimated salvage value of $15,000 after 5
years.
Solution
The average value is:
P = 50,000 (5 + 1) + 15,000 (5 -1) = 300,000 + 60,000 = $36,000
2x5 10

4.2.6.7. Insurance, Tax, and Storage Costs

219
Insurance cost represents the cost incurred due to fire, theft, accident, and liability insurance for
the equipment. Tax cost represents the cost of property tax and licenses for the equipment.
Storage cost includes the cost of rent and maintenance for equipment storage yards, the wages of
guards and employees involved in moving equipment in and out of storage, and associated direct
overhead.

The cost of insurance and tax for each item of equipment may be known on an annual basis. In
this case, this cost is simply divided by the hours of operation during the year to yield the cost
per hour for these items. Storage costs are usually obtained on an annual basis for the entire
equipment fleet. Insurance and tax costs may also be known on a fleet basis. It is then necessary
to prorate these costs to each item.

This is usually done by converting the total annual cost into a percentage rate, then dividing
these costs by the total value of the equipment fleet. By doing so, the rate for insurance, tax, and
storage may simply be added to the investment cost rate for calculating the total annual cost of
investment, insurance, tax, and storage. The average rates for interest, insurance, tax, and storage
found in the literature are listed in Table 4.16 These rates will vary according to related factors
such as the type of equipment and location of the job site.

Table 4.16 Average Rates for Investment Costs

Item Average Value (%)


Interest 3–9
Tax 2–5
Insurance 1–3
Storage 0.5–1.5

The total equipment ownership cost is calculated as the sum of depreciation, investment cost,
insurance cost, tax, and storage cost. As mentioned earlier, the elements of ownership cost are
often known on an annual cost basis. However, while the individual elements of ownership cost
are calculated on an annual cost basis or an hourly basis, total ownership cost should be

220
expressed as an hourly cost. After all elements of ownership costs have been calculated, they can
be summed up to yield the total ownership cost per hour of operation. Although this cost may be
used for estimating and charging equipment costs to projects, it does not include job overhead or
profit. Therefore, if the equipment is to be rented to others, overhead and profit should be
included to obtain an hourly rental rate.

Example 4.14

Calculate the hourly ownership cost for the second year of operation of a 465 HP twin-engine
scraper. This equipment will be operated 8 h/day and 250 days per year in average conditions.
Use the sum-of-years’-digits method of depreciation as the following information:
 Initial cost: $186,000  Insurance: 1.5%
 Tire cost: $14,000  Taxes: 3%
 Estimated life: 5 years  Storage: 0.5%
 Salvage value: $22,000  Fuel price: $2.00/gal
 Interest on the investment: 8%  Operator’s wages: $24.60/h

Solution
Depreciation in the second year = 4 (186,000 - 22,000 - 14,000) = $40,000
15
= 40,000 = $20:00/h
8(250)

Investment cost, tax, insurance, and storage cost:


Cost rate = investment + tax, insurance, and storage = 8 + 3 + 1.5 + 0.5 =13%
Average investment = 186,000 ± 22,000 = $20,800
2x5
Investment, tax, insurance, and storage = 84,000(0.18)/2000 = $7.56/h
Therefore, the Total ownership cost= 16.53 + 7.56 = $24.09/h

4.2.6.8. Cost of Operating Construction Equipment

221
Operating costs of the construction equipment, which represent a significant cost category and
should not be overlooked, are the costs associated with the operation of a piece of equipment.
They are incurred only when the equipment is actually used. The operating costs of the
equipment are also called ‘‘variable’’ costs because they depend on several factors, such as the
number of operating hours, the types of equipment used, and the location and working condition
of the operation.
The operating costs vary with the amount of equipment used and job-operating conditions. The
best basis for estimating the cost of operating construction equipment is the use of historical data
from the experience of similar equipment under similar conditions. If such data is not available,
recommendations from the equipment manufacturer could be used.
A. Maintenance and Repair Cost
The cost of maintenance and repairs usually constitutes the largest amount of operating expense
for the construction equipment. Construction operations can subject equipment to considerable
wear and tear, but the amount of wear varies enormously between the different items of the
equipment used and between different job conditions. Generally, the maintenance and repair
costs get higher as the equipment gets older.

Equipment owners will agree that good maintenance, including periodic wear measurement,
timely attention to recommended service and daily cleaning when conditions warrant it, can
extend the life of the equipment and actually reduce the operating costs by minimizing the
effects of adverse conditions.

Table 4.17 Range of Typical Lifetime Repair Costs

222
Initial Cost without Tires (%) and Operating Conditions

Equipment Type Favora Average Unfavorab Equipment Type Favora Average Unfavor
ble le ble able

Crane 40-45 50-55 60-70 Motor grader 45-50 50-55 55-60

Excavator crawler 50-60 70-80 90-95 Scraper 85 90-95 105

Loader track 80-85 90 100-105 Tractor crawler 85 90 95

Loader wheel 50-55 60-65 75 Tractor wheel 50-55 60-65 75

Excavator wheel 75 80 85 Truck, off-highway 70-75 80-55 90-95

All items of plant and equipment used by construction contractors will require maintenance and
probably also require repairs during the course of their useful life. The contractor who owns the
equipment usually sets up facilities for maintenance and engages the workers qualified to
perform the necessary maintenance operations on the equipment.

The annual cost of maintenance and repairs may be expressed as a percentage of the annual cost
of depreciation or it may be expressed independently of depreciation. The hourly cost of
maintenance and repair can be obtained by dividing the annual cost by its operating hours per
year. The hourly repair cost during a particular year can be estimated by using the following
formula:

Hourly repair cost = year digit x lifetime repair cost


Sum of year’s digits hours operated

The lifetime repair cost is usually estimated as a percentage of the equipment’s initial cost
deducting the cost of tires. It is adjusted by the operating condition factor obtained from Table
4.17.

Example 4.15

223
Estimate the hourly repair cost of the scraper in Example 4.14 for the second year of operation.
The initial cost of the scraper is $186,000, tire cost $14,000, and its useful life is 5 years. Assume
average operating condition and 2000 h of operation per year.

Solution
Lifetime repair cost factor = 0.90

Lifetime repair cost = 0.90(186,000 -14,000) = $154,800

224
Hourly repair cost = 2 x (154,800) = $10.32/h
15 2000

Table 4.18 Range of Typical Lifetime Repair Costs

Initial Cost without Tires (h) and Operating Conditions


Equipment Type Favorable Average Unfavorable
Loader wheel 3200 - 4000 2100 - 3500 1300 – 2500
Motor grader 5000 3200 1900
Scraper single engine 4000 - 4600 3000 - 3300 2500
Scraper twin engine 3600 -4000 3000 2300 – 2500
Scraper elevating 3600 2700 2100–2250
Tractor wheel 3200–4000 2100–3000 1300–2500
Truck, off-highway 3500–4000 2100–3500 1100–2500

B. Tire Cost
The tire cost represents the cost of tire repair and replacement. Because the life expectancy of
rubber tires is generally far less than the life of the equipment on which they are used on, the
depreciation rate of tires will be quite different from the depreciation rate of the rest of the
vehicle. The repair and maintenance cost of tires as a percentage of their depreciation will also
be different from the percentage associated with the repair and maintenance of the vehicle. The
best source of information in estimating tire life is the historical data obtained under similar
operating conditions. Table 4.18 lists the typical ranges of tire life found in the most recent
literature on the subject for various types of equipment. Tire repair cost can add about 15% to
tire replacement cost. So, the following equation may be used to estimate tire repair and
replacement cost:
Tire repair and replacement costs = 1.15 x Cost of a set of tires ($)
Expected tire life (h)
C. Consumable Costs
Consumables are the items required for the operation of a piece of equipment that literally gets
consumed in the course of its operation. These include, but are not limited to, fuel, lubricants,

225
and other petroleum products. They also include filters, hoses, strainers, and other small parts
and items that are used during the operation of the equipment.
D. Fuel Cost
Fuel consumption is incurred when the equipment is operated. When operating under standard
conditions, a gasoline engine will consume approximately 0.06 gal of fuel per flywheel
horsepower hour, while a diesel engine will consume approximately 0.04gal/fwhp-h. A
horsepower hour is a measure of the work performed by an engine.

Table 4.19 Average Fuel Consumption Factors (gal/h/hp)

Working Conditions (gal/h/hp)


Equipment Type Favorable Average Unfavorable
Loader track 0.030 - 0.034 0.040 - 0.042 0.046 - 0.051
Loader wheel 0.020 - 0.024 0.027 - 0.036 0.031- 0.047
Motor grader 0.022 - 0.025 0.029 - 0.035 0.036 - 0.047
Scraper single engine 0.023 - 0.026 0.029 - 0.035 0.034 - 0.044
Scraper twin engine 0.026 - 0.027 0.031 - 0.035 0.037 - 0.044
Tractor wheel 0.020 - 0.028 0.026 - 0.038 0.031- 0.052
Truck, on highway 0.014 - 0.029 0.020 - 0.037 0.026 - 0.046
Tractor crawler 0.028 - 0.342 0.037 - 0.399 0.046 - 0.456
Truck, off-highway 0.017- 0.029 0.023 - 0.037 0.029 - 0.046

Example 4.16
Calculate the average hourly fuel consumption and hourly fuel cost for a twin engine scraper in
Example 4.14. It has a diesel engine rated at 465 HP and fuel cost $2.00/gal. During a cycle of
20s, the engine may be operated at full power, while filling the bowl in tough ground requires 5s.
During the balance of the cycle, the engine will use no more than 50% of its rated power. Also,
the scraper will operate about 45 min/h on average. For this condition, the approximate amount
of fuel consummated during 1 h is determined as follows:

Solution
 Rated power: 465 hp
 Engine factor: 0.5
 Filling the bowl, 5 s/20 s cycle = 0.250

226
 Rest of cycle, 15/20 x 0.5 = 0.375
 Total cycle = 0.625
 Time factor, 45 min/60 min = 0.75
 Operating factor, 0.625 x 0.75 = 0.47
 From Table 4.19: use ‘‘unfavorable’’ fuel consumption factor = 0.040
 Fuel consumed per hour: 0.47(465)(0.040) = 8.74 gal
 Hourly fuel cost: 8.74 gal/h ($2.00/gal) = $17.48/h.
E. Lubricating Oil Cost
The quantity of oil required by an engine per change will include the amount added during the
change plus the make-up oil between changes. It will vary with the engine size, the capacity of
crankcase, the condition of the piston rings, and the number of hours between oil changes. It is a
common practice to change oil every 100 to 200 h.
The quantity of oil required can be estimated by using the following formula

q = 0.006(HP) (f) + c
(7.4) t

where q is the quantity consumed (gal/h), HP the rated horsepower of engine, c the capacity of
crankcase (gal), f the operating factor, t the number of hours between changes, the consumption
rate 0.006 lbs/hp-h, and the conversion factor 7.4 lbs/gal. The consumption data or the average
cost factors for oil, lubricants, and filters for their equipment under average conditions are
available from the equipment manufacturers.

F. Mobilization and Demobilization Cost


This is the cost of moving the equipment from one job site to another. It is often overlooked
because of the assumption that the previous job would have already paid for it. Regardless of
these calculations, the costs of equipment mobilization and demobilization can be large and are
always important items in any job where substantial amounts of equipment are used. These costs
include freight charges, unloading cost, assembly or erection cost (if required), highway permits,
duties, and special freight costs (remote or emergency).

227
The hourly cost can be obtained from the total cost divided by the operating hours. Some public
agencies cap the maximum amount of mobilization that will be paid before the project is
finished. In these instances, the estimator must check the actual costs of mobilization against the
cap. If the cap is exceeded, the unrecovered amount must be allocated to other pay items to
ensure that the entire cost of mobilization is recovered.
G. Equipment Operator Cost
Operator’s wages are usually added as a separate item and added to other calculated operating
costs. They should include overtime or premium charges, workmen’s compensation insurance,
social security taxes, bonus, and fringe benefits in the hourly wage figure. Care must be taken by
the companies that operate in more than one state or that work for federal agencies, state
agencies and private owners.
H. Special Items Cost
The cost of replacing high-wear items, such as dozer, grader, and scraper blade cutting and end
bits, as well as ripper tips, shanks, and shank protectors, should be calculated as a separate item
of the operating cost. As usual, unit cost is divided by the expected life to yield cost per hour.
I. Maintenance and Repair Costs

Maintenance and repair costs are the crux of the equipment replacement decision and result from
the cost of labor and parts used to maintain and repair the given piece of equipment. This is an
incredibly dynamic system and can be affected by the following factors:
 Type of equipment.
 Age of the equipment
 Operating conditions
 Operating skill of the operator
 Daily care by the operator
 Maintenance department
 Frequency and level of preventive maintenance.

4.2.6.9. Overhead cost

228
Overhead cost Varies from organization to organization based on such factors as organizational
structure, company policies on payments (Salary, per dimes, etc.).

4.2.6.10. Profits

In the commercial context, profit is important and imperative to provide for profit as rooms for
capacity building of the organization which owns the equipment’s.

4.2.7. Equipment Utilization, Equipment Life Downtime and cost of downtime

4.2.7.1. Equipment Utilization

Organizations with large equipment fleets use data on equipment location to optimize the
utilization of these expensive and critical resources. Moreover, the data can be transmitted
almost anywhere on the face of the planet giving equipment manager real-time status of their
fleets, facilitating routine, and crisis decision-making. Sustained profitability using standard
equipment typically demands continual equipment utilization. The ability to do this is typically
market driven and influenced by the overall business strategy of the company. Simply stated,
equipment that is not operating is not making money.

This is control by improving equipment maintenance and service costs by allowing the schedule
for preventive maintenance to be developed with real-time knowledge of equipment location and
Increase equipment utilization and sharing across jobsites by identifying idle equipment on a
real-time basis as well as showing individual equipment and monitoring the factors of Equipment
production capacity. The main factors of Equipment production capacity are:
 Machine factor;
 Operator factor;
 Working condition factor and
 Management factor.

229
4.2.7.2. Equipment Life Downtime

Downtime is the time when equipment does not work due to repairs or mechanical adjustments.
Downtime tends to increase as equipment usage increases. Availability, the portion of the time
when equipment is in actual production or is available for production, is the opposite of
downtime. For example, if the equipment’s downtime is 10%, then its availability is 90%.
The downtime cost includes the ownership cost, operating cost, operator cost, and calculate the
hourly downtime cost. In the table, the direct cost of productivity loss is not computed because it
is not easily quantified as a dollar value. However, it is described as a weight factor where
maximum availability is held equal to 1.0 and proportionate loss in availability carries a
weightage less than 1.0.
Productivity is a measure of the equipment’s ability to produce at the original rate. The
productivity decrease results in the increase in production cost because the operating time of the
equipment should be extended or more equipment’s should be deployed to get the same
production rate.

4.2.7.3. Cost of Equipment downtime

Mostly the costs of equipment downtime are list as follows:


 Repair cost or cost of substitute
 Idle related resource cost (personnel and/or equipment)
 Loss of productivity costs (disruption, new learning, moral etc.)
 Contract related costs (liquidity damage, escalation due delay, etc.)
 Other costs (extended overhead costs, extended overtime, acceleration costs,
etc.)

4.2.8. Factors for Equipment’s Downtime

There are lists of Factors for Equipment’s Downtime. Some of them:


A. Site related factors B. Equipment’s related factors
230
 Poor working condition  Age of equipment
 Bad location of the site  Type and quality of equipment
 Uncertainty of operation  Complexity of equipment

C. Force majeure D. Project related factors

 Landslide  Rental facility


 Accident  Substitutions of equipment on hand
 Vandalism  Project requirements
 Location of workshop
 Availability of spare parts

 Availability of resources
E. Company’s Producers and Policies F. Site management action

 Schedule maintenance  Wait until repair


 Inventory management  Add/change resources
 Replacement decisions  Work out squander and
 Procurement system  Transfer crew transfer
 Repair-maintenance facilities  Get substitutions equipment

G. Crew level factors


 Skill level of machines
 Skill level of operator’s
 Fatigue
 Moral and
 Motivation

231
Assessment

Practical

Week 11: Practice

All work items in tables A, B, and C are parts of the same project.

BILL OF QUANTITY

Rat Amo
No. Description Unit Qty e unt
A. SUBSTRUCTURE
1. EXCAVATION AND EARTHWORK
Site clearance and topsoil excavation to an
1.1 average depth of 200mm m2 113.4
Pit excavation for isolated footings and footing
1.2 columns to a depth not exceeding 1700mm m3 27.44
Trench excavation in ordinary soil for masonry
foundation wall up to 1000mm depth starting
1.3 from natural ground level m3 23.67
Bulk excavation in ordinary soil to reduced
level for a depth of 40cm from cleared ground
1.4 level m³ 16.53
Backfill around footings and foundation
1.5 columns with selected material m3 39.5
Cart away excavated material and deposit to
1.6 an appropriate tip m3 50.82
250mm thick basaltic or equivalent hardcore
well rammed, consolidated, and blinded with
1.7 crushed stone m2 41.6
2. CONCRETE WORK
2.1 C -25 reinforced cement concrete for:
a) 5cm thick lean concrete, with concrete
quality C-5, 150kg of cement/m³ m2 36.82
b) footing pads m3 1.6
c) footing columns m3 0.54
d) grade beams m3 4.94

232
e) ground floor slab (10cm thick) m2 36.9
2.2 10cm wide marble skirting ml 73.05
Provide, cut, and fix in position wooden
2.2 formwork for:
a) footing pads m2 9.6
b) footing columns m2 9.6
c) grade beams m2 37.46
Steel reinforcement work according to
2.3 structural drawings
a) Ø 16mm kg 262.57

(Table A Activity Continued)

b) Ø 14mm kg 499.4
c) Ø 12mm kg 0
d) Ø 8mm kg 335.72
3. STONEMASONRY
Construct 50cm thick stone masonry from
basaltic or equivalent stone bedded and joined
in 1:3 cement mortars under external grade
3.1 beams. m3 16.02
Well-dressed stone masonry for trench
3.2 foundation above ground level m3 0
B. SUPERSTRUCTURE
1. CONCRETE WORK
1.1 C -25 reinforced cement concrete for:
a) Elevation columns m3 1.34
b) Beams m3 2.54
c) stair m3 1.35
d) 1st-floor slab m3 45.65
Provide, cut, and fix in position wooden
1.2 formwork for:
a) ground floor elevation columns m2 26.88
b) slab beams m2 37.64
c) stair m2 114.92
d) 1st slab floor m2 7.32
e) first-floor elevation columns m2 26.88
f) top tie beam m2 31.8
Provide, cut, tie and fix in position steel
1.3 reinforcement bars:
a) Ø 16mm kg 0
b)Ø 14mm kg 160.99
233
c) Ø 12mm kg 1307.34
d) Ø 10mm kg 604.67
e) Ø 8mm kg 111.85
f) Ø 6mm kg 63.09
2. MASONRY WORK
20cm HCB external wall bedded on cement mortar
2.1 (mix ratio 1:3), both sides left for plastering m2 101.46

(Table B Activity Continued)

15cm HCB internal walls bedded on cement


mortar (mix ratio 1:3), both sides left for
2.2 plastering. m2 53.2
15cm HCB parapet walls bedded on cement
2.3 mortar (mix ratio 1:3) m2 20.76
3. ROOFING WORK
3.1 G -28 Corrugated Iron Sheet roof cover m2 69.3
Ø10-12cm eucalyptus top and bottom chord
3.2 members ml 139
Ø8-10cm eucalyptus vertical and diagonal chord
3.3 members ml 10.26
3.4 provide and fix 5X8cm zigba purlin c/c 90cm ml 88
provide and fix 2.5x25cm wide zigba facia board
fixed to fit rafter or truss including two coats of
3.5 weather-resistant oil paint ml 11
Supply and fix 8mm chip wood ceiling, including
3.6 4x5cm zigba battens fixed at 60x60cm. m² 36.9
G -28 sheet metal gutter with a development
3.7 length of 45cm ml 11
3.8 Ø110mm PVC downpipe ml 6
4. METALWORK
Supply and fix metal doors & windows fabricated
from LTZ profile sizes 38X38 mm according to
4.1 size and shape specified
a) doors (900mm X 2800mm) Pcs 3
b) door (1000mm X 2800mm) Pcs 1
c) door (800mm X 2800mm) Pcs 3
d) door (700mm X 2800mm) Pcs 2
e) window (1000mm X 1700mm) Pcs 2
f) window (1200mm X 1700mm) Pcs 1
g) window (1500mmx1700mm) Pcs 2
4.2 Balustrades and Handrails for stair flights ml 26
6. FINISHING WORKS
Apply two coats of plaster for internal and external
6.1 wall finish with cement m2 475.42
234
6.2 Apply two coats of water paint on the whitewash m2 475.42
Apply 30mm thick cement screed on concrete
6.3 slabs under floor tiles m2 36.92
2
6.4 Terrazzo tile floor finish for verandah & balcony m 32.31
6.5 PVC tiles floor finishes m2 63.59
6.6 Ceramic tile work for toilet floor and walls m2 7.65
4mm thick glazing work for metal doors and
6.7 windows including putty m2 19.38

1. For the work activities given in the above table perform the scheduling task according to the
instructions given below using Ms. Project, which you did as in chapter 3. Specifically: Let the
project duration be 180 days (6 months),
A. Copy and paste the work activities listed in the above table to a blank ms-project
document
B. Create your calendar and Give your name to the calendar,
C. Set the currency to be “BIRR”,
D. Assign material and equipment resources to tasks,
E. Take your assumptions and locally used standards to determine the duration,
F. Give correct indentation to the activities
G. Give your project title: Concrete printing
H. Author: Your name
I. Link the activities according to their logical sequence. In the FS, SS, FF, or SF
relationship.
2. Prepare a resource sheet with a minimum of:
I) Material resources
II) work resources and
III) cost resources
3. Assign resources to tasks
Insert a cost column and show the total cost of the project (Fill in the cost that you
calculated the materials cost and equipment cost to complete the project in the
previous week).

Week 12: Practice

Repeat practice activity 11

235
Week 13 :Practice

For the work activities given in table activity 11 and in addition to you did previously
perform the scheduling task according to the instructions given below using MS Project:
I. Make the color of the holidays be Red
II. Prepare the time scale in three tiers with
a. Bottom tier weeks
b. Middle tier months and
c. Top tier years and
III. Save your work ‘on Desktop’ with your name and Id no.

Week 14 practice

Repeat practice activity 13

236
Objective type questions

1. State whether True or False:


A. Construction materials account for a large fraction of the overall project cost.
B. Materials are classified broadly into capital items and revenue items.
C. Functions involved in material management are materials planning, procurement,
custody (receiving, warehousing and issuing), materials accounting, transportation,
inventory monitoring and control, and materials codification.
D. Costs related to inventory are holding cost, stock-out cost, and ordering cost.
E. Commonly used inventory-control policies are ABC analysis, VED analysis, and FSN
analysis.
F. EOQ inventory model aims to provide an answer to how much to order i.e., it fixes
the tradeoff between ordering cost and carrying cost.
G. The choice of construction equipment defines the construction method, which in a
way leads to the determination of time and cost for the project.
H. Construction equipment can be classified based on functional aspects, operational
aspects, and purpose (general or special).
I. The two broadly important factors for the selection of construction equipment are
hard factors and soft factors.
J. Hard factors for equipment selection are intangible, whereas soft factors are tangible
and quantifiable.
K. Some of the decision variables affecting the selection of construction equipment are
economic considerations, company-specific, site-specific, equipment-specific, client-
and project-specific, manufacturer-specific and labor-specific.
L. Straight-line method, the sum of year’s digit method, and the declining balance
method are the methods to select construction equipment.
M. Replacement of an existing productive asset becomes necessary due to physical or
mechanical impairment, technological obsolescence, uneconomic increasing
maintenance cost, and functional inadequacy.
N. Sensitivity analysis is a non-probabilistic technique and it is aimed at identifying the
sensitivity of a particular variable.
O. Breakeven analysis is equivalent to cost-volume-profit analysis.
P. Price - direct cost is known as contribution per unit.

237
Q. Breakeven point = indirect cost/contribution.
R. Companies try to lower the breakeven point by decreasing sales price, increasing the
total cost of production, or decreasing the quantity of production.

238
Subjective type questions

1. Define and elaborate material and equipment management.


2. Discuss the importance and objective of material and equipment management.
3. Define Construction Equipment Costs, Life and Replacement Analysis (Ownership
cost(initial cost, Depreciation, Investment (Or Interest) Cost, Insurance, Tax and Storage
Costs), Cost of Operating Construction Equipment (Maintenance and Repair Cost, Tire
Cost, Fuel Cost, Lubricating Oil Cost, Mobilization and Demobilization Cost, Equipment
Operator Cost and Maintenance and Repair Costs), Overhead cost and Profits),
4. Discuss the Method of depreciation calculation and, Equipment Utilization, Equipment
Life Downtime, and cost of equipment downtime.
5. A building costing $2,000,000 is expected to have 25 years of life with 25 percent salvage
value. Calculate the depreciation charge for years 4, 8, 16, and 20 using the
a. Straight-line method
b. Sum of years method
c. Double declining balance method
6. Calculate the hourly ownership cost for the second year of operation of a 380 HP twin-
engine scraper. This equipment will be operated 6 h/day and 150 days/year in average
conditions. Use the sum-of-years’-digits method of depreciation as the following
information:
 Initial cost: $300,000
 Tire cost: $14,000
 Estimated life: 5 years
 Salvage value: $22,000
 Interest on the investment: 8%
 Insurance: 1.5%
 Taxes: 3%
 Storage: 0.5%
 Fuel price: $2.00/gal
 Operator’s wages: $24.60/h

239
7. Discuss the importance of inventory management in material management.
8. What are the different functions of material management?
9. What are the advantages and disadvantages of centralized and local purchasing?
10. What are the advantages and disadvantages of early and late procurement?
11. What are the benefits of proper classification of materials?
12. Discuss the role of vendor management in material management.
13. What are the inventory-related costs? What are the functions of inventories?
14. How do we classify different inventory systems?
15. Discuss different inventory-control policies.
16. Differentiate between ABC analysis, VED analysis, and FSN analysis.
17. Why are inventory models needed? Discuss the EOQ model.
18.What are the advantages and disadvantages of using machines in construction in a country
like India?
19. Enumerate various types of equipment required for (a) earthwork, (b) concrete, and (c)
material hoisting.
20. Describe various types of lifting cranes. State the various uses of tower cranes in the
construction industry.

Solve the following problems

1. A building costing $1,600,000 is expected to have 35 years of life with 25 percent salvage
value. Calculate the depreciation charge for years 4, 9, 18, and 26 using the
A. Straight-line method
B. Sum of years method
C. Double declining balance method

2. An asset having the first cost of $1,150,000 is expected to have a life of 12 years with a
salvage value of ` 100,000. In what year does the depreciation charge by the straight-line
method first exceed the depreciation charge allowed by the
A. Sum of year’s method and
B. Double declining balance method?
3. A construction company stores various items see in the Table below in the central stores.
The average annual consumption and cost per unit of items stored are given. Classify the
240
items using ABC analysis.

Name of the Average annual Average cost The average annual Ranking
item consumption (No.) per unit ($) cost of consumption
A 5,000 45.00 90,000 3
B 1,000 90.00 450,000 5
C 2,000 225.0 45,000 1
D 4,000 11.25 15,000 7
E 50 300.00 375,000 11
F 6,000 62.50 135,000 2
G 2,000 67.50 75,000 4
H 4,000 18.75 22,500 6
I 50 375.00 22,500 10
J 250 105.00 22,500 9
K 200 187.50 37,500 8
L 50 150.00 7,500 12

4. Consider a unit of equipment costing $80,000 with an estimated salvage value of $20,000
after 6 years. The average value is:

5. Calculate the hourly ownership cost for the second year of operation of a 380 HP twin-
engine scraper. This equipment will be operated 8 h/day and 200 days per year in average
conditions. Use the sum-of-years’-digits method of depreciation as the following
information.
 Initial cost: $200,000
 Tire cost: $14,000
 Estimated life: 5 years
 Salvage value: $22,000
 Interest on the investment: 8%
 Insurance: 1.5%
 Taxes: 3%
 Storage: 0.5%
 Fuel price: $2.00/gal
 Operator’s wages: $24.60/h

241
Evaluation sheet

Evaluation Out of 5 Score Remark


Working Principle
Equipment and
material
Result management
Evaluatio
Discussion
n
Conculusion

Applications
Tool Setup
Usage and storage
Work
equipment and
Evaluatio
materials
n
Equipment and
material Handling
Time used (180
Time Min.) (1 per 10
min.)
Total /45

242
References

1. Kumar Neeraj Jha (2015), (Associate Professor Department of Civil Engineering), IIT,
Delhi Construction Project Management, Theory, and Practice. SECOND EDITION

2. Emad Elbeltagi, Ph.D., P.Eng., (2009), Professor of Construction Management


Structural Engineering Department, Faculty of Engineering, Mansoura University,
Lecture notes on construction Project management

3. Gang Chen, Building Construction: Project Management, Construction Administration,


Drawings, and Specs, Detailing Tips, Schedules, Checklists, and Secrets Others Don’t
Tell You: Architectural Practice Simplified.

4. Fundamentals of Construction Management

5. Clough, Richard H. & Sears, Gelen A. (1979). “Construction Project Management.”


John Wiley & Sons Inc., NY.

6. Cormican, David. (1985). “Construction Management: Planning and Finance.”


Construction Press, London.

7. Eldosouky, Adel I. (1996). “Principles of Construction Project Management.” Mansoura


University Press, Mansoura, Egypt.

8. Gould, Frederick E. (1997). “Managing the Construction Process: Estimating,


Scheduling, and Project Control.” Prentice-Hall Inc., New Gersy.

9. Harris, Robert. (1978). “Precedence and Arrow Networking Techniques for


Construction.” John Wiley & Sons Inc., NY.

243
Chapter 5: Time Management

Lesson Plan (Time Management)

2. Learning objectives
At the end of this chapter students/ trainees will be able to:
 Know the Time Management;
 Develop a better understanding of the objectives and importance of Time Management;
 Know the Network Crashing and Cost-Time Trade-Off;
 Know the objective of the time-cost trade-off analysis is to reduce the original project
duration;
 Develop the level of understanding of the objective of the time-cost trade-off analysis;
Understand how to reduce the activity duration/ Shortening Project Duration;
 Identify the Project Time-Cost Relationship;
 Improve knowledge on project durations shortening and the effect of Project direct costs,
and indirect costs.

3. Motivation
 Outputs through Question and Answer; repetitive discussion and instruction of time
management, objectives of time management, the importance of Time Management,
Network Crashing and Cost-Time Trade-Off, the objective of the time-cost trade-off
analysis, and project durations shortening and its effect on Project direct costs, and
indirect cost.

4. Expectations or Outcomes
 Define the terms of Time management, the importance of Time Management, Network
Crashing, and Cost-Time Trade-Off, the objective of the time-cost trade-off analysis,
project duration shortening, and the effect of Project direct costs, and indirect costs.
 Able to identify objectives of time management, the importance of time Management,
Network Crashing and Cost-Time Trade-Off, the objective of the time-cost trade-off
analysis, project duration shortening and its effect on Project direct costs, and indirect

244
cost.
5. Equipment
 Desktop or Laptop computer with internet connection.
 MS Project software and textbooks

6. Practice contents/Activities/Safety

7. Clean-up
 After finishing practice, all tools and equipment utilized in the practice shall be returned
to the proper storage place.
 Clean up the practice shop.

8. Independent practice/Follow-up activities


 Learning through assignment

9. Review/Reflection
Review the outcome of the practice, improvement measures, and previously reflected opinions.

245
Chapter 5: Time Management

5.1. Introduction

Construction Project management as defined by PMBOK 2000 is the application of knowledge,


skills, tools, and techniques to a broad range of activities to meet the requirements of a
particular project. Munns and Bjeirmi (1996) define project management as the process of
controlling the achievement of project objectives. Utilizing the existing organizational structure
and resources, it seeks to manage the project by applying a collection of tools and techniques,
without adversely disturbing the routine operations of the company. The function of project
management includes defining the requirement of work, establishing the extent of work,
monitoring the progress of the work, and adjusting deviations from the plan.

Construction Project management aims to achieve the stated goals of the project leading to a
completed facility, under planning, executing, and controlling time, funds, and human and
technical resources. The planning essentially consists of setting objectives, identifying
resources, and forming a strategy. Executing consists of an allocation of resources, guiding
execution, coordinating efforts, and motivating the staff. Controlling consists of measuring
achievement goals, reporting, and resolving problems. The planning, execution, and control are
performed continuously till the goals of the project are realized.

Project management knowledge and practices are best described in terms of their component
processes. These processes can be placed into five process groups (initiating, planning,
executing, controlling, and closing) and nine knowledge areas (project integration management,
project scope management, project time management, project cost management, project quality
management, project human resource management, project communications management,
project risk management, and project procurement management).

246
Project
Goals
Human
Funds Resource

Project
Management

Technical
Time
Resource

Complete
Facility

Figure 5.1 the elements of project management

Therefore, time management is the one basic knowledge area of Project Management, and
Time management means you need to understand:

 Critical path scheduling techniques that include any associated specifications


 How to use scheduling software
 What causes delays and how they occur
 What the pros and cons are of various schedule and delay mitigation methods used by
others on the project
 The principles behind successful schedule and delay analysis methods

5.2. Importance of Time Management in Construction

There are many reasons why time management is important in construction. Some of them:
 One of these is the fact that workers are often paid by the hour, so time management will
help control salary costs.
 Another reason time management in construction is important is that having work delayed

247
or behind schedule can hinder the overall project,
 Time management in construction also is vital because, if projects are not finished on
time, or at least as quickly as promised, then this can void a contract and cause the
construction company to lose payment for breach of contract.
 Looking into the future, it also is important to have effective time management in place for
construction companies, because projects that are not finished on time can derail company
efforts to secure bids on other projects.
 In construction, one of the biggest costs besides materials is paying workers. Skilled
workers are often paid by the hour. Having effective time management in construction
projects includes giving workers reasonable goals, workable timeframes, and the resources
to do their jobs, and expecting them to complete their responsibilities on time. The more
efficiently, accurately, and effectively that workers complete their tasks, the lower the
overall cost will be.
 Provides Customers basic right to know:
 Establish how long construction should take

 Expect timely performance

 Liquidate damages for late performance

 And may be able to limit contractor preparations

 Provides to You, as the General Contractor, to have these rights:


 Reasonable access to work areas;

 Timely approvals;

 Timely delivery of owner-furnished materials and equipment;

 Prompt payment;

 Right to finish early, and Extra time and money for owner-caused delays.

5.3. Network Crashing and Cost-Time Trade-Off

In the previous chapter, we discussed the computational procedures of the critical path method.
In this section, we discuss one of the application areas of this networking technique. One of the
assumptions in the critical path method is that the duration of an activity can be reduced or

248
crashed to a certain extent by increasing the resources assigned to it. As is known, the execution
of activity involves both direct costs and indirect costs. These terminologies are explained in
detail elsewhere.
It is known that any reduction in the duration (by increasing resources) of critical path activities
can reduce the project duration and, thereby, enhance the possibility of a reduction in project
cost. However, as will be explained shortly, there is no point in attempting to crash all the
activities by increasing the resources.

An activity can be performed at its normal or most efficient pace or it can be performed at a
higher speed. The duration associated with the former is called ‘normal duration’ and the
duration associated with the latter is called ‘crash duration’.

Some activities along the critical path sometimes need to be shortened to reduce the overall
duration of the project. This leads to a decrease in the indirect expenses (due to a decrease in
duration), as shown in Figure 5.2, and an increase in the direct expenses (due to more
mobilization of resources), as shown in Figure 5.3. As is evident from Figure 5.2 and Figure 5.3,
the relationship between the cost of the job and the duration has been assumed to be linear. The
steeper the slope of the line, the higher the cost of expediting the job at an earlier date. The
expediting of an activity to an earlier time is referred to as crashing. Three cases normally arise:

Figure 5.2 Indirect cost vs time Figure 5.3 Direct cost vs time

249
Figure 5.4 Network used for crashing

The objective of the time-cost trade-off analysis is to reduce the original project duration,
determined from the critical path analysis, to meet a specific deadline, with the least cost. In
addition to that, it might be necessary to finish the project in a specific time to:

 Finish the project by a predefined deadline date.


 Recover early delays;
 Avoid liquidated damages;
 Free key resources early for other projects;
 Avoid adverse weather conditions that might affect productivity;
 Receive an early completion-bonus; and
 Improve project cash flow etc.

Reducing project duration can be done by adjusting overlaps between activities or by reducing
activities’ duration. What is the reason for an increase in direct cost as the activity duration is
reduced? A simple case arises in the use of overtime work. By scheduling weekend or evening
work, the completion time for an activity as measured in calendar days will be reduced.
However, extra wages must be paid for such overtime work, so the cost will increase. Also,
overtime work is more prone to accidents and quality problems that must be corrected, so costs
may increase.

250
The activity duration can be reduced by one of the following actions:

 Applying for multiple-shift work.


 Working extended hours (over time).
 Offering incentive payments to increase productivity.
 Working on weekends and holidays.
 Using additional resources.
 Using materials with faster installation methods.
 Using alternate construction methods or sequence

5.4. Activity Time-Cost Relationship

In general, there is a trade-off between the time and the direct cost to complete an activity; the
less expensive the resources, the larger duration they take to complete an activity. Shortening
the duration of the activity will normally increase its direct cost which comprises: the cost of
labor, equipment, and material. It should never be assumed that the number of resources
deployed and the task duration are inversely related. Thus one should never automatically
assume that the work that can be done by one man in 10 weeks can be done by 10 men in one
week.

A simple representation of the possible relationship between the duration of an activity and its
direct costs appears in Figure 5.5. Considering only this activity in isolation and without
reference to the project completion deadline, a manager would choose a duration that implies
minimum direct cost, called the normal duration. At the other extreme, a manager might choose
to complete the activity in the minimum possible time, called crashed duration, but at a
maximum cost.

251
Figure 5.5: Illustration of linear time/cost trade-off for an activity

The linear relationship shown in Figure 5.5 between these two points implies that any
intermediate duration could also be chosen. Some intermediate points may represent the ideal or
optimal trade-off between time and cost for this activity. The slope of the line connecting the
normal point (lower point) and the crash point (upper point) is called the cost slope of the
activity. The slope of this line can be calculated mathematically by knowing the coordinates of
the normal and crash points.

Cost slope = Crash Cost – Normal Cost


Normal Duration – Crash duration

Terminology:

NT = Normal Time (duration) of Activity


NC = Crash Time (duration) of Activity
CT = Cost of Activity at NT

In other words, the cost slope indicates the extra cost incurred by reducing the activity duration
by one day. The higher the value of the slope, the more the cost of crashing the activities. As
shown in the Figures below, the least direct cost required to complete an activity is called the
normal cost (minimum cost), and the corresponding duration is called the normal duration. The
shortest possible duration required for completing the activity is called the crash duration, and
the corresponding cost is called the crash cost. Normally, a planner starts his/her estimation and
scheduling process by assuming the least costly option.

252
Figure 5.6: Illustration of non-linear time/cost trade-off for an activity

Figure 5.7: Illustration of discrete time/cost trade-off for an activity

Example 5.1
A subcontractor has the task of erecting 8400 square meters of metal scaffolds. The contractor
can use several crews with various costs. It is expected that the production will vary with the
crew size as given below, and consider the following rates: Labor LE96/day; carpenter
LE128/day; foreman LE144/day and scaffolding LE60/day. Determine the direct cost of this
activity considering different crews formation.

253
Table 5.1 Quantity of metal scaffolds and human resources

Estimated daily Crew Crew formation


production size
2
166 m 5 - men 1 scaffold set, 2 labors, 2 carpenters, 1 foreman
204 m2 6 - men 2 scaffold sets, 3 labors, 2 carpenters, 1 foreman
230 m2 7 - men 2 scaffold sets, 3 labors, 3 carpenters, 1 foreman

Solution
The duration of installing the metal scaffold can be determined by dividing the total quantity by
the estimated daily production. The cost can be determined by summing up the daily cost of
each crew and then multiplying it by the duration of using that crew. The calculations are shown
in the following table.

Table 5.2 Duration and Cost Calculation

Crew size Duration (days) Crew formation


5 - men 50.6 (use 51) 51x (1 x 60 + 2 x 96 + 2 x 128 + 1 x 144) = 33,252
6 - men 41.2 (use 42) 42 x (2 x 60 + 3 x 96 + 2x128 + 1x144) = 33,936
7 - men 36.5 (use 37) 37 x (2 x 60 + 3 x 96 + 3 x 128 + 1 x 144) = 34,632

This example illustrates the options which the planner develops as he/she establishes the normal
duration for activity by choosing the least cost alternative. The time-cost relationship for this
example is shown in Figure 5.8 the cost slop for this activity can be calculated as follow:

Cost slope 1 (between points 1 and 2) = (33936 – 33252) / (51 – 42) = 76.22 LE/day
Cost slope 2 (between points 2 and 3) = (34632 – 33936) / (42 – 37) = 139.2 LE/day

254
Figure 5.8: Time-cost relationship of Example 5.1

5.5. Project Time-Cost Relationship

Total project costs include both direct costs and indirect costs of performing the activities of the
project. Direct costs for the project include the costs of materials, labor, equipment, and
subcontractors. Indirect costs, on the other hand, are the necessary costs of doing work that
cannot be related to a particular activity, and in some cases cannot be related to a specific
project.

If each activity was scheduled for the duration that resulted in the minimum direct cost in this
way, the time to complete the entire project might be too long, and substantial penalties
associated with the late project completion might be incurred. Thus, planners perform what is
called time-cost trade-off analysis to shorten the project duration. This can be done by selecting
some activities on the critical path to shorten their duration.

As the direct cost for the project equals the sum of the direct costs of its activities, then the
project direct cost will increase by decreasing its duration. On the other hand, the indirect cost
will decrease by decreasing the project duration, as the indirect cost is almost a linear function
with the project duration. Figure 5.9 illustrates the direct and indirect cost relationships with the
project duration.

255
Figure 5.9: Project time-cost relationship

The project total time-cost relationship can be determined by adding up the direct cost and
indirect cost values together as shown in Figure 5.9. The optimum project duration can be
determined as the project duration that results in the least project total cost.

5.6. Shortening Project Duration

The minimum time to complete a project is called project-crash time. This minimum completion
time can be found by applying critical path scheduling with all activity durations set to their
minimum values. This minimum completion time for the project can then be used to determine
the project-crash cost. Since there are some activities not on the critical path that can be
assigned a longer duration without delaying the project, it is advantageous to change the all-
crash schedule and thereby reduce costs.

Heuristic approaches are used to solve the time/cost tradeoff problem such as the cost slope
method used in this chapter. In particular, a simple approach is to first apply critical path
scheduling with all activity durations assumed to be at minimum cost. Next, the planner can
examine activities on the critical path and reduce the scheduled duration of activities that have
the lowest resulting increase in costs. In essence, the planner develops a list of activities on the
critical path ranked with their cost slopes. The heuristic solution proceeds by shortening
activities in the order of their lowest cost slopes.

256
As the duration of activities on the shortest path is shortened, the project duration is also
reduced. Eventually, another path becomes critical, and a new list of activities on the critical
path must be prepared. Using this way, good but not necessarily optimal schedules can be
identified.

The procedure for shortening project duration can be summarized in the following steps:
a. Draw the project network.
b. Perform CPM calculations and identify the critical path, using normal durations and costs
for all activities.
c. Compute the cost slope for each activity from the following equation:
Cost Slope = Crash Cost – Normal Cost / Normal Duration –Crash Duration
d. Start by shortening the activity duration on the critical path which has the least cost slope
and has not been shortened to its crash duration.
e. Reduce the duration of the critical activities with the least cost slope until its crash duration
is reached or until the critical path changes.
f. When multiple critical paths are involved, the activity(ies) to shorten is determined by
comparing the cost slope of the activity which lies on all critical paths (if any), with the
sum of the cost slope for a group of activities, each one of them lies on one of the critical
paths.
g. Having shortened a critical path, you should adjust activities timings, and floats.
h. The cost increase due to activity shortening is calculated as the cost slope multiplied by the
time of time units shortened.
i. Continue until no further shortening is possible, and then the crash point is reached.
j. The results may be represented graphically by plotting project completion time against
cumulative cost increase. This is the project direct-cost / time relationship. By adding the
project indirect cost to this curve to obtain the project time/cost curve. This curve gives the
optimum duration and the corresponding minimum cost.

Example 5.2
Assume the following project data given in Table 5.3. It is required to crash the project duration
from its original duration to a final duration of 110 days. Assume a daily indirect cost of LE
100.

257
Table 5.3: Data for Example 5.2

Normal Crash
Activity Preceded Duration Cost Duration Cost (LE)
(day) (LE) (day)
A - 120 12,000 100 14,000
B - 20 1,800 15 2,800
C B 40 16,000 30 22,000
D C 30 1,400 20 2,000
E D, F 50 3,600 40 4,800
F B 60 13,500 45 18,000

Solution
The cost slope of each activity is calculated. Both the crushability and the cost slope are shown
beneath each activity in the precedence diagram. The critical path is B-C-D-E and the project
duration is 140 days. Project total normal direct cost = sum of normal direct costs of all
activities = LE 48300.

Figure 5.10: Network development and Project crashing for Example 5.2

A. The activity on the critical path with the lowest cost slope is activity “D”, this activity can
be crashed within 10 days. Then adjust the timing of the activities.

258
Figure 5.11: Network development and Project crashing for Example 5.2

 A new critical path will be formed, B-F-E.


 The new Project duration is 130 days.
 The project's direct cost is increased by 10 x 60 = LE 600.
 Project direct cost = 48300 + 600 = LE 48900

B. At this step activity, “E” will be crashed, as this activity lies on both critical paths. Activity
“E” will be shortened by 10 days.

Figure 5.12: Network development and Project crashing for Example 5.2

Accordingly, all activities will be turned into critical activities.


 The new Project duration is 120 days.
 The project's direct cost is increased by 10 x 120 = LE 1200.
259
 Project direct cost = 48900 + 1200 = LE 50100

C. In this step, it is difficult to decrease one activity’s duration and achieve decreasing in the
project duration. So, either crash an activity on all critical paths (if any), otherwise,
choose several activities on different critical paths. As shown, activities “A” and “B” can
be crashed together which have the least cost slope (100 + 200). Then, crash activities
“A” and “B” by 5 days.

Figure 5.13: Network development and project crashing for Example 5.2

 The new Project duration is 115 days.


 The project direct cost is increased by 5 x (100 + 200) = LE 1500.
 Project direct cost = 50100 + 1500 = LE 51600

D. In this final step, it is required to decrease the duration of activity from each path. The
duration of activity ”A” will be crashed to 110 days, “C” to 35 days, and “F” to 55 days.
Thus, achieving decreasing project duration to 110 days. Also, increase in the project
direct cost by 5 x (100 + 600 + 300) = LE 5000

260
Figure 5.14: Network development and project crashing for Example 5.2

Table 5.4: Contract duration and The corresponding cost 5.2

Duration (day) Direct cost (LE) Indirect cost (LE) Total cost (LE)
140 48,300 14,000 62,300
130 48,900 13,000 61,900
120 50,100 12,000 62,100
115 51,600 11,500 63,100
110 56,600 11,000 67,600

Cost (LE)

Project duration (days)

Figure 5.15: Cost vs Duration relationship for Example 5.2

Example 5.3
The durations and direct costs for each activity in the network of a small construction contract
under both normal and crash conditions are given in the following table. Establish the least cost
for expediting the contract. Determine the optimum duration of the contract assuming the
indirect cost is LE 125/day.

261
Table 5.5: Data for Example 5.3

Activit Normal Crash


Preceded
y Duration (day) Cost (LE) Duration (day) Cost (LE)
A - 12 7,000 10 7,200
B A 8 5,000 6 5,300
C A 15 4,000 12 4,600
D B 23 5,000 23 5,000
E B 5 1,000 4 1,050
F C 5 3,000 4 3,300
G E, C 20 6,000 15 6,300
H F 13 2,500 11 2,580
I D, G, H 12 3,000 10 3,150

Solution
The cost slope of each activity is calculated. Both the crushability and the cost slope are shown
beneath each activity in the precedence diagram. The critical path is A-C-G-I and the contract
duration is 59 days.

Figure 5.15: Network development and project crashing for Example 5.3

A. The activity on the critical path with the lowest cost slope is “G”, this activity can be
crashed by 5 days, but if it is crashed by more than 2 days another critical path will be
generated. Therefore, activity “G” will be crashed for 2 days only. Then adjust the
timing of the activities.
 A new critical path will be formed, A-C-F-H-I.
262
 The new contract duration is 57 days and the cost increase is 2 x 60 = LE 120.

Figure 5.16: Network development and project crashing for Example 5.3

B. At this step the activities that can be crashed are listed below:
 Either “A” at cost LE 100/day
Or “C” at cost LE 200/day
Or “I” at cost LE 75/day
Or “F & G” at cost LE 360/day Or “H & G” at cost LE 100/ day

Activity “I” is chosen because it has the least cost slope, and it can be crashed within 2 days.
Because it is the last activity in the network, it does not affect other activities.

Figure 5.17: Network development and project crashing for Example 5.3

263
 The new contract duration is 55 days and the cost increase is 2 x 75 = LE 150.

 Cumulative cost increase = 120 + 150 = LE 270

C. Now, we could select “A” or both “H & G”, because they have the same cost slope.
Activity “A” is chosen to be crashed. This will change the timings for all activities, but
no new critical path will be formed

Figure 5.18: Network development and project crashing for Example 5.3

 The new contract duration is 53 days and the cost increase is 2 x 100 = LE 200.
 Accordingly, cumulative cost increase = 270 + 200 = LE 470

D. Now, activities “H & G” can be crashed by 2 days each. “A” new critical path AB-D-I
will be formed.
E.

264
Figure 5.19: Network development and project crashing for Example 5.3

 The new contract duration is 51 days and the cost increase is 2 x 100 = LE 200.
 Accordingly, cumulative cost increase = 470 + 200 = LE 670

F.At this stage, the network has three critical paths. The activities that can be crashed are
listed below: Either C & B at cost LE 350/day or F, G & B at cost LE 510/day Activities
C & B are chosen because they have the least cost slope.

Figure 5.20: Network development and project crashing for Example 5.3

 The new contract duration is 49 days and the cost increase is 2 x 350 = LE 700.
 Cumulative cost increase = 670 + 700 = LE 1370
 Now, there is no further shortening is possible. And the contract duration and the
corresponding cost are given in the table below.

Table 5.6: Contract duration and The corresponding cost for Example 5.3

Duration (day) Direct cost x 1000 LE Indirect cost x 1000 LE Total cost x 1000 LE
59 36.50 7.375 43.875
57 36.62 7.125 43.745
55 36.77 6.875 43.645
53 36.97 6.625 43.595
51 37.17 6.375 43.545
49 37.87 6.125 43.995
265
LE x 1000

Time
(days)

Figure 5.21: Corresponding Cost vs Duration relationship of Example 5.3

Example 5.4
Suppose that the indirect expenses are $6,000/day. Thus, the total project cost if all the activities
are executed at their normal pace is given by:

Table 5.7: Data for Example 5.4

Activit Normal Crash Cost slope


y Duration Cost ($) Duration Cost ($) $/day
(1, 2) 3 5,000 2 7,000 2,000
(2, 3) 4 6,000 2 10,000 2,000
(2, 4) 3 9,000 1 17,000 4,000
(2, 5) 4 5,000 3 9,000 4,000
(4, 5) 5 7,000 2 16,000 3,000
(3, 5) 2 8,000 1 9,500 1,500
(5, 6) 5 20,000 5 20,000 -

Total Project Cost

= [Normal cost of activities (1, 2), (2, 3), (2, 4), (2, 5), (4, 5), and (3, 5)] + (Indirect cost per
day) x Duration of the project)
= (5,000 + 6,000 + 9,000 + 5,000 1 7,000 + 8,000 + 20,000) + (6,000 × 16)
= (60,000) + (96,000) = $156,000.

It may be noted that the crashing exercise would have given the same result even if we had
assumed the base cost to be equal to zero, as this was a constant in all the steps. Since every
activity has to be performed to complete the project, a base cost of every activity has to be paid.
These costs may be regarded as ‘fixed’ and to shorten the project duration, we have to shorten

266
the duration of activities along the critical path (2-4-5-6). We observe that the activity (1, 2) is
on the critical path and has the least slope ($2,000/day), and hence, can be crashed first. This
activity can be crashed in one day; thus, the project duration reduces by a day. The project
duration has become 15 days now.

The project cost to complete in 15 days


= Cost to complete the project in 16 days + Cost of crashing by a day - Saving in indirect cost
=156,000 + 2,000 - 6,000
= $152,000

Thus, have obtained a reduction of $4,000 and a reduction in duration by 1 day. The next higher
cost slope ($3,000 per day) on the critical path is for activity (4, 5) and we can crash this
activity to a maximum of 3 days (from a normal duration of 5 days to a crash duration of 2
days). However, we will crash it in two steps of one day each. With one day of crashing, the
project duration will become 14 days now.

The project cost to complete in 14 days


= Cost to complete the project in 15 days + Cost of crashing by a day - Saving in indirect cost
=152,000 + 3,000 - 6,000
= $149,000

Table 5.8: Data for Example 5.4

Option Cost ($/day)


C and B 4,000 + 2,000 = 6,000
C and F 4,000 + 3,000 = 7,000
E and B 3,000 + 2,000 = 5,000
E and F 3,000 + 1,500 = 4,500

The project duration, thus, reduces by another day and the cost has also decreased by $3,000
over the previous crash cost. The activity (4, 5) crashed again by a day. The project duration
becomes 13 days now.
The project cost to complete in 13 days
= Cost to complete the project in 14 days + Cost of crashing by a day - Saving in indirect cost
=149,000 + 3,000 - 6,000
= $146,000

267
The project duration, thus, reduces by another day and the cost has also decreased by $3,000
over the previous crash cost, and there is still a scope of crashing activity (4, 5). However, note
that there are two critical paths 1-2-4-5-6 and 1-2-3-5-6, both of 13 days duration. The available
options for crashing are given in Table 5.8 It may further be noted that the cost of crashing one
day is the summation of individual cost slopes. The lowest-cost slope option is given by
activities E and F. The crash costs of these two activities combined are $4,500. Thus, we crash
activities E and F by 1 day, which makes the project duration equal to 12 days.

The project cost to complete in 12 days


= Cost to complete the project in 13 days + Cost of crashing by a day - Saving in indirect cost
=146,000 + 4,500 - 6,000 = $144,500
The project duration, thus, reduces by another day and the cost has also decreased by $1,500
over the previous crash cost. Again, we have two critical paths 1-2-4-5-6 and 1-2-3-5-6, both
with a project duration of 12 days. The available option for crashing is activities C and B
together. The crash cost would be equal to $4,000 + $2,000 = $6,000. The project duration
reduces by another day and becomes equal to 11 days.

The project cost to complete in 11 days


= Cost to complete the project in 12 days + Cost of crashing by a day - Saving in indirect cost
=144,500 + 6,000 - 6,000
= $144,500

The project duration reduces by another day but there is no increase or decrease in the cost over
the previous crash cost.

Total cost($)

Time
days)

Figure 5.22: Time vs cost for Example 5.4


268
Now, from the observation, there are 3 critical paths, 1-2-4-5-6, 1-2-3-5-6, and 1-2-5-6, each of
11 days duration. Our only available option for crashing is to crash activities B, C, and D
together for one day, and the cost of crashing the three activities together is equal to the sum of
the cost slopes of activities (2, 3), (2, 4) and (2, 5) that is, $10,000. Thus, the project duration
becomes 10 days.
The project cost to complete in 10 days
= Cost to complete the project in 11 days + Cost of crashing by a day - Saving in indirect cost
=144,500 + 10,000 - 6,000 = $148,500

The project duration reduces by another day but the cost has increased by $4,000 over the
previous crash cost. Thus, we have reached a stage where the decrease in duration is
accompanied by a significant increase in the direct cost, forcing us to stop further crushing. If
we combine all our results in a graph showing how project length affects the schedule costs, we
obtain the curve as in Figure 5.22 which shows the minimum project cost corresponding to
project durations of 11 days and 12 days.

It may be noted that the crashing exercise would have given the same result even if we had
assumed the base cost to be equal to zero, as this was a constant in all the steps. Since every
activity has to be performed to complete the project, a base cost of every activity has to be paid.
These costs may be regarded as the ‘fixed’ or ‘sunk’ costs of the project. Neglecting the base
cost of activities since these remain the same irrespective of crashing, the base cost of activities
has been assumed to be zero. This cost has to be paid for all activities and is independent of the
crashing done.
The time-cost curve shown in Figure 5.22 helps the project manager to choose a suitable
schedule for the project. The decision to select a particular duration does not depend on the cost
alone but is determined by several other criteria such as the considerations of the safety of
workers, the risks involved in a particular completion schedule, and so on. Thus, a project
manager may not always go for the schedule associated with the least cost and he may choose to
go for the moderate cost schedule after incorporating other factors.
Example 5.5
The following data given will explain the technique of Crashing with overhead expense
assuming the Normal cost of the project = Rs. 10,000 and the Cost of overheads per day = Rs.
200/day

269
Table 5.9: Data for Example 5.5

Activity Normal Crash Predecessor Cost


Duration Duration Slope
A 3 1 - 500
B 7 3 A 100
C 4 2 A 400
D 5 2 c 200

Solution
Step - 1
 OH Cost = 12 X 200 = Rs. 2400
 Total cost = Normal cost + OH Cost
= Rs. 10,000 + Rs. 2,400
= Rs. 12,400 (Before crashing)

Step - 2
Normal Time:

Finsh
Start
A 3 B 7 4
1 1 2 3
2 2 C4 D5
2 2
3

Figure 5.23: Network Diagram of the Normal Time for Example 5.5

 Since D has a lease cost slope, it will be crashed by 2 days


 Cost of crashing = 2 X 200 = 400
 Reduction of OH = 2 X 200 = 400
 Hence there will be no extra cost if the project time is reduced to 10 days

Step - 3
270
Finsh
Start
A 3 B 7 4
1 1 2 3
2 2 C4 D3
2 2
3

Figure 5.24: Network Diagram of the crash Time for Example 5.5

 Now there are two critical paths viz: A - B & A- C- D


 To curtail one day the options are:
 Crash - A - 500 Crash - B+C - 500 Crash - B+D - 300
 The last option is the least expensive. So, B & D will be curtailed by one day each
 Extra cost of crashing = 100 (B) + 200 (D) = 300
 Saving in O.H. = 1 X 200 = 200

 Hence the project cost will increase by Rs. 100/-

Finsh
Start
A 3 B 6 4
1 1 2 3
2 2 C4 D2
2 2
3

Figure 5.25: Network Diagram of the crash Time for Example 5.5

 Options for crashing are:


A - 500
B+C - 500
B+D - 300
 Since D is fully crashed last option can’t be used. Hence A is the least expensive option.
 Crash A by 2 days
Extra cost of crashing = 2 X 500 = 1000

271
Saving in O.H. = 2 X 200 = 400
Therefore, Net extra Cost = Rs. 600/-

Start Finsh
A 2 B 6 4
1 1 2 3
2 2 C4 D2
2 2
3

Figure 5.26: Network Diagram of the crash Time for Example 5.5

Step - 4

 At this stage:
A is fully crashed
D is fully crashed
B is crashed by 1 day (can be crashed by 3 days)
C not crashed (can be crashed by 2 days)
 The only option left for crashing is B + C
 Activities B & C will be curtailed by 2 days
Extra cost of crashing = 2 X 500 = 1000
Saving in O.H. = 2 X 200 = 400
Net Extra Cost = Rs. 600/-
 The project duration is now 5 days

Start Finsh

A 1 B 4 4
1 1 2 3
2 2 C2 D2
2 2
3

Figure 5.27: Network Diagram of the crash Time for Example 5.5

Table 5.10: Data for Example 5.5


272
Duratio Step Saving OH Extra Cost Net Extra Total
n of Crashing Cost cost
12 - - - - 1,240
10 Step -1 400 400 - 1,240
9 Step -2 200 300 100 1,250
7 Step -3 400 800 600 13,100
5 Step -4 400 1000 600 13,700

Example 5.6
The durations and direct costs for each activity in the network of a small construction contract
under both normal and crash conditions are given in the following table. Establish the least cost
for expediting the contract. Determine the optimum duration of the contract assuming the
Administration cost is $500/day.

Table 5.11: Data for Example 5.6

Activity Predecessor Time (Days) Cost ($)


Normal Crash Normal Crash
A - 3 2 $250 $500
B A 5 3 $200 $400
C B 5 2 $200 $800
D C 3 3
X A 5 2 $1,450 $2,500
Y X 4 2 $500 $750
Z y 3 1 $1,100 $1,750

Solution
Step – 1: Slope determination

Table 5.12: Data for Example 5.6

273
Activity Predecessor Time (Days) Cost ($) $ Saved/
Day (Slope)
Normal Crash Normal Crash
A - 3 2 $250 $500 250
B A 5 3 $200 $400 $100
C B 5 2 $200 $800 $200
D C 3 3 -
X A 5 2 $1,450 $2,500 $350
Y X 4 2 $500 $750 $125
Z Y 3 1 $1,100 $1,750 $325

Step – 2: Network development and Critical task determination

Start C5
A 3 B 5 3 2 5 D3 Finish
1 2 2 3 3
2 2 X5 7
2 Y4 Z3
4 6
2 1

Figure 5.28: Network Diagram of the Normal Time for Example 5.6

Then having the above cost slope which activity should we crash?
Paths Duration
1. A,B, C, D 3 +4+5 +3 = 16
2. A, X, Y, Z 3 + 5+ 4+5 = 15
Step – 3: Project crashing
Paths Duration B (Choose B due to its cheapest of the A, B, or C alternatives by 1 day.
1. A, B, C, D 16 15
2. A, X, Y, Z 15 15

Start C5
3 5 Finish
274
1 2
2 2
A 3 B 4 2 D3
2 3 3
X5 7
2 Y4 Z3
4 6
2 1

Figure 5.29: Network Diagram of the Crash Time for Example 5.6

 Cost 100 Which activity(s) should we


 Save 500 And then crash next? A?, (B, X), (B, Y), (B,
Z), (C, X), (C, Y), or (C, Z),
 Net 400
 Cumul 400

Paths Duration B By (Choose By due to its cheapest of the other alternatives by 1 day.
i. A, B, C, D 15 14
ii. A, X, Y, Z 15 14

Start C5
A 3 B 3 3 2 5 D 3 Finish
1 2 2 3 3
2 2 X5 7
2 Y3 Z3
4 6
2 1

Figure 5.30: Network Diagram of the Crash Time for Example 5.6

 Cost 100 225


 Save 500 500 Then Which activity(s) should we
 Net 400 275 crash next? A?, ( (C, X), (C, Y), and
(C, Z),
 Cumul 400 675

275
Paths Duration B BY A Choose A (it’s cheapest than the others) by 1 day.
1. A, B, C, D 15 14 13
2. A, X, Y, Z 15 14 13

Start C5
A 2 B 3 3 2 5 D 3 Finish
1 2 2 3 3
2 2 X5 7
2 Y3 Z3
4 6
2 1

Figure 5.31: Network Diagram of the Crash Time for Example 5.6

 Cost 100 225 250


 Save 500 500 500 Then Which activity(s) should
we crash next? (C, X), (C, Y) or
 Net 400 275 250 (C, Z)?
 Cumul 400 675 925

Paths Duration B BY A CY Choose Cy (it’s cheapest than the others) by 1 day.


1. A, B, C, D 15 14 13 12
2. A, X, Y, Z 15 14 13 12

276
Start C4
A 2 B 3 3 2 5 D3 Finish
1 2 2 3 3
2 2 X5 7
2 Y2 Z3
4 6
2 1

Figure 5.32: Network Diagram of the Crash Time for Example 5.6

 Cost 100 225 250 325


 Save 500 500 500 500 Then Which activity(s) should
 Net 400 275 250 175 we crash next? (C, Z)?

 Cumul 400 675 925 1100

Paths Duration B BY A CY CZ Choose CZ by 1 day.


1. A, B, C, D 16 15 14 13 12 11
2. A, X, Y, Z 15 15 14 13 12 11

Start C3
A 2 B 3 3 2 5 D 3 Finish
1 2 2 3 3
2 2 X5 7
2 Y2 Z2
4 6
2 1

Figure 5.33: Network Diagram of the Crash Time for Example 5.6

277
We lose $25, So do not
crash CZ or 12th day is
the most economical
day to be crash.
 Cost 100 225 250 325 525
 Save 500 500 500 500 500
 Net S. 400 275 250 175 -25
 Cumul 400 675 925 1100 1075

Therefore,

Start C4 Finish
A 2 B 3 3 2 5 D3
1 2 2 3 3
2 2 X5 7
2 Y2 Z3
4 6
2 1

Figure 5.34: Network Diagram of the Crash Time for Example 5.6

Paths Duration B BY A CY Achieved by crashing


1. A, B, C, D 15 14 13 12 The most economic Duration
2. A, X, Y, Z 15 14 13 12
 Cost 100 225 250 325 Spent $900 in increased direct costs
 Save 500 500 500 500 Avoided $2000 in administration costs
 Net 400 275 250 175
 Cumul 400 675 925 1100 Net Saving

Assessment

278
Practicae

Week 15 :Practicae

All work items in tables A, B, and C are parts of the same project.
BILL OF QUANTITY
Rat
No. Description Unit Qty e Amount
A. SUBSTRUCTURE
1. EXCAVATION AND EARTHWORK
Site clearance and topsoil excavation to an
1.1 average depth of 200mm m2 113.4
Pit excavation for isolated footings and footing
1.2 columns to a depth not exceeding 1700mm m3 27.44
Trench excavation in ordinary soil for masonry
foundation wall up to 1000mm depth starting
1.3 from natural ground level m3 23.67
Bulk excavation in ordinary soil to reduced
level for a depth of 40cm from cleared ground
1.4 level m³ 16.53
Backfill around footings and foundation
1.5 columns with selected material m3 39.5
Cart away excavated material and deposit to
1.6 an appropriate tip m3 50.82
250mm thick basaltic or equivalent hardcore
well rammed, consolidated, and blinded with
1.7 crushed stone m2 41.6
2. CONCRETE WORK
2.1 C -25 reinforced cement concrete for:
a) 5cm thick lean concrete, with concrete
quality C-5, 150kg of cement/m³ m2 36.82
b) footing pads m3 1.6
c) footing columns m3 0.54
d) grade beams m3 4.94
e) ground floor slab (10cm thick) m2 36.9

(Table A. Activity Continued)

279
2.2 10cm wide marble skirting ml 73.05
Provide, cut, and fix in position wooden
2.2 formwork for:
a) footing pads m2 9.6
b) footing columns m2 9.6
c) grade beams m2 37.46
Steel reinforcement work according to
2.3 structural drawings
a) Ø 16mm kg 262.57

b) Ø 14mm kg 499.4
c) Ø 12mm kg 0
d) Ø 8mm kg 335.72
3. STONEMASONRY
Construct 50cm thick stone masonry from
basaltic or equivalent stone bedded and joined
in 1:3 cement mortars under external grade
3.1 beams. m3 16.02
Well-dressed stone masonry for trench
3.2 foundation above ground level m3 0

B. SUPERSTRUCTURE
1. CONCRETE WORK
1.1 C -25 reinforced cement concrete for:
a) Elevation columns m3 1.34
b) Beams m3 2.54
c) stair m3 1.35
d) 1st-floor slab m3 45.65
Provide, cut, and fix in position wooden
1.2 formwork for:
a) ground floor elevation columns m2 26.88
b) slab beams m2 37.64
c) stair m2 114.92
d) 1st slab floor m2 7.32
e) first-floor elevation columns m2 26.88
f) top tie beam m2 31.8
Provide, cut, tie and fix in position steel
1.3 reinforcement bars:
a) Ø 16mm kg 0
b)Ø 14mm kg 160.99
c) Ø 12mm kg 1307.34

(Table B. Activity Continued)


280
d) Ø 10mm kg 604.67
e) Ø 8mm kg 111.85
f) Ø 6mm kg 63.09
2. MASONRY WORK
20cm HCB external wall bedded on cement mortar
2.1 (mix ratio 1:3), both sides left for plastering m2 101.46
15cm HCB internal walls bedded on cement
mortar (mix ratio 1:3), both sides left for
2.2 plastering. m2 53.2
15cm HCB parapet walls bedded on cement
2.3 mortar (mix ratio 1:3) m2 20.76
3. ROOFING WORK
3.1 G -28 Corrugated Iron Sheet roof cover m2 69.3
Ø10-12cm eucalyptus top and bottom chord
3.2 members ml 139
Ø8-10cm eucalyptus vertical and diagonal chord
3.3 members ml 10.26
3.4 provide and fix 5X8cm zigba purlin c/c 90cm ml 88
provide and fix 2.5x25cm wide zigba facia board
fixed to fit rafter or truss including two coats of
3.5 weather-resistant oil paint ml 11
Supply and fix 8mm chip wood ceiling, including
3.6 4x5cm zigba battens fixed at 60x60cm. m² 36.9
G -28 sheet metal gutter with a development
3.7 length of 45cm ml 11
3.8 Ø110mm PVC downpipe ml 6
4. METALWORK
Supply and fix metal doors & windows fabricated
from LTZ profile sizes 38X38 mm according to
4.1 size and shape specified
a) doors (900mm X 2800mm) Pcs 3
b) door (1000mm X 2800mm) Pcs 1
c) door (800mm X 2800mm) Pcs 3
d) door (700mm X 2800mm) Pcs 2
e) window (1000mm X 1700mm) Pcs 2
f) window (1200mm X 1700mm) Pcs 1
g) window (1500mmx1700mm) Pcs 2
4.2 Balustrades and Handrails for stair flights ml 26

(Table B Activity Continued)

281
6. FINISHING WORKS
Apply two coats of plaster for internal and external
6.1 wall finish with cement m2 475.42
6.2 Apply two coats of water paint on the whitewash m2 475.42
Apply 30mm thick cement screed on concrete
6.3 slabs under floor tiles m2 36.92
2
6.4 Terrazzo tile floor finish for verandah & balcony m 32.31
6.5 PVC tiles floor finishes m2 63.59
6.6 Ceramic tile work for toilet floor and walls m2 7.65
4mm thick glazing work for metal doors and
6.7 windows including putty m2 19.38

For the work activities given in the above table perform the scheduling task according to the
instructions given below using Ms. Project, which you did as in chapter 3-4. Specifically: Let the
project duration be 180 days (6 months), and finalize the following thasks bellow (1-5):
1. Do the following tasks
A. Copy and paste the work activities listed in the above table to a blank ms-project
document
B. Create your calendar and Give your name to the calendar,
C. Set the currency to be “BIRR”,
D. Assign material and equipment resources to tasks,
E. Take your assumptions and locally used standards to determine the duration,
F. Give correct indentation to the activities
G. Give your project title: Concrete printing
H. Author: Your name
I. Link the activities according to their logical sequence. In the FS, SS, FF, or SF
relationship.
2. Prepare a resource sheet with a minimum of:
I. 2 Material resources
II. 2 work resources and
III. 2 cost resources
3. Assign resources to tasks
4. Insert a cost column and show the total cost of the project (Fill in the cost that you calculated
the materials cost and equipment cost to complete the project in the previous week).
5. For the work activities given in table activity 15 and in addition to you did previously
perform the scheduling task according to the instructions given below using MS Project:
I. Make the color of the holidays be Red
II. Prepare the time scale in three tiers with
a. Bottom tier weeks

282
b. Middle tier months and
c. Top tier years and
III. Save your work ‘on Desktop’ with your name and Id no.

283
Theory
Objective type question

I. State whether True or False:

A. Time-cost trade-off analysis in CPM tries to optimize project duration to minimize the
total project cost.
B. An activity with the highest cost slope is to be considered first for crushing.
C. When there is more than one critical path in a network, crashing an activity on any one
path will reduce the project duration.
D. Normal time is the time required to complete the activity at ordinary conditions and cost.
E. Crash time is the shortest possible activity time.

II. Chose the correct answer

1. Which of the following is not the promise of the two parts in construction contract with time
management?
A. If projects are not finished in a timely manner, or at least as quickly as promised, the
construction company to lose payment for breach of contract,
B. In order not to be penalized at the completion of the job should it take longer than the
original estimate, the builder has to claim an extension of time.
C. Extra time and money for owner-caused delays and money for contractor-caused delays.
D. None
1. Expenses which change proportionally with the level of output?

284
A. Overhead cost
B. Investment cost
C. Variable costs
D. Fixed costs
3. Which one of the following several effective methods were managers have required for crashing
specific projects activities when resources are not constrained?

A. Adding resources B .Outsourcing project Work C. Crashing Project


D. Scheduling overtime E. All

271
Subjective type of questions

1. Define time management.


2. Discuss the importance and objective of time management and Identify objectives of
Time Management,
3. Discuss network Crashing and Cost-Time Trade-Off,
4.Discuss the time-cost trade-off analysis, project duration shortening, and its effect on
Project direct costs, and indirect costs.
5. Identify the Project Time-Cost Relationship.
6. Discuss the procedure of project durations shortening/ time-cost trade-off.
7. What do you understand by ‘crashing’ of an activity? How is it used for optimizing
project costs? Explain with help of a diagram.
8. What is the need for project time crashing in any construction project?
9. What is meant by ‘project crashing’?
10. Define direct cost and indirect cost.
11. Cost and schedule data for a small project are given below. Assume an indirect cost of
LE 100/day. Develop the time-cost curve for the project and determine the minimum
contract duration:

Preceded Cost (LE) Duration (days)


ctivity
by Crash Normal Crash Normal
A - 3900 3600 6 8
B A 6500 5500 3 4
C B 7200 6350 7 8
D B 4900 4700 18 20
E B 2200 2050 9 11
F C 1700 1200 6 7
G F 7200 7200 5 5
H E 1000 9450 10 12
0
I D, G, H 4700 4500 6 7

12. Short-answer type questions


a) What do you understand by ‘crashing’ of an activity? How is it used for
optimizing project costs? Explain with help of a diagram.
b) Define ‘crashing’ and explain the basic assumptions that enable crashing in CPM.

272
c) What is the need for project time crashing in any construction project?
d) What is meant by ‘project crashing’?
e) Discuss the time-cost trade-off.
f) Define direct cost and indirect cost.
13. Cost and schedule data for a small project are given below. Assume an indirect cost of
LE 200/day. Develop the time-cost curve for the project and determine the minimum
contract duration:

Preceded Cost (LE) Duration (days)


Activity
by Crash Normal Crash Normal
A - 3900 3600 6 7
B A 6500 5500 3 5
C B 7200 6350 7 9
D B 4900 4700 18 19
E B 2200 2050 9 10
F C 1700 1200 6 8
G F 7200 7200 5 5
H E 1000 9450 10 11
0
I D, G, H 4700 4500 6 7

14. The following table gives the activities involved in a pipeline contract. The duration and
cost data are also given. The indirect cost for the contract is LE120/day. Calculate the
minimum cost of the work corresponding to the contract duration of 102 days.

Normal Crash Cost slope


Activity Preceded
Time Cost (LE) Days LE
A - 10 200 - -
B A 20 200 - -
C A 40 1,800 - -
D A 28 500 8 10
E B 8 150 - -
F C 10 100 4 40
G E 30 3,000 20 180
H C, G 20 2,800 12 50
I C, G 24 1,000 10 65
J D, F, H 10 200 4 80
K I 8 520 1 80
L J, K 6 420 1 60
M J, K 6 200 3 40
N J, K 6 150 2 70
273
O L. N 4 300 - -
P M, O 2 180 - -
15. Assume the following project data given in the Table below. It is required to crash the
project duration from its original duration to a final duration of 111 days. Assume a
daily administration/indirect cost of LE 200.

Activit Normal Crash


Preceded
y Duration (day) Cost (LE) Duration (day) Cost (LE)
A - 120 12,000 100 14,000
B - 20 1,800 15 2,800
C B 40 16,000 30 22,000
D C 30 1,400 20 2,000
E D, F 50 3,600 40 4,800
F B 60 13,500 45 18,000

16. The data given below shows the normal schedule for a project. You can decrease (crash)
the durations at an additional expense. The Table given below summarizes the time-cost
information for the activities. The owner wants you to finish the project in 110 days.
Find the minimum possible cost for the project if you want to finish it in 110 days.
(Assume that for each activity there is a single linear, continuous function between the
crash duration and normal duration points).

Precedenc Duration (days) Cost


Activity
e Normal Crash Normal Crash
A - 120 100 12,000 14,000
B - 20 15 1,800 2,800
C B 40 30 16,000 22,000
D C 30 20 1,400 2,000
E D, F 50 40 3,600 4,800
F B 60 45 13,500 18,000

17. The data given below shows the normal schedule for a project. You can decrease (crash)
the durations at an additional expense. The Table given below summarizes the time-cost
information for the activities and Crashes the project by 4 days.

Activit Normal Crash Normal Crash Cost/


Precedence
y Time Time Cost Cost Day
A None 7 6 $7,000 $8,00 $1,000

274
0
B A 3 2 5,000 7,000 2,000
A 10,20
C 4 3 9,000 1,200
0
D B, C 5 4 3,000 4,500 1,500
E D 2 1 2,000 3,000 1,000
F D 4 2 4,000 7,000 1,500
G E, F 5 4 5,000 8,000 3,000

275
Evaluation sheet

Out of 5
Evaluation Score* Remark

Working Principle

Project crash and


cost management
Result
Discussion
Evaluation

Conclusion

Applications

software Setup

Usage of the
Work software tools and
Evaluation materials
Cost calculation
and project
crashing
Time used
Time
(180 Min.)

Total /45

276
References

[1]. Kumar Neeraj Jha (2015), (Associate Professor Department of Civil Engineering), IIT,
Delhi Construction Project Management, Theory, and Practice. SECOND EDITION

[2]. Emad Elbeltagi, Ph.D., P.Eng., (2009), Professor of Construction Management Structural
Engineering Department, Faculty of Engineering, Mansoura University, Lecture notes on
construction Project management

[3]. Cameron K. Andres P.E. Retired, Ronald C. Smith & W. Ronald Woods, Principles &
Practices of Commercial Construction.

[4]. Carl Chatfield &Timothy Johnson, (2013) Microsoft Project.

[5].Gang Chen, Building Construction: Project Management, Construction Administration,


Drawings, and Specs, Detailing Tips, Schedules, Checklists, and Secrets Others Don’t Tell
You: Architectural Practice Simplified.

[6]. Frederick Gould & Nancy Joyce, Construction Project Management.

[7]. Fundamentals of Construction Management

[8].Awani, Alfred O. (1983). “Project Management Techniques.” Petrocelli Books Inc.

[9]. Clough, Richard H. & Sears, Gelen A. (1979). “Construction Project Management.” John
Wiley & Sons Inc., NY.

[10]. Cormican, David. (1985). “Construction Management: Planning and Finance.”


Construction Press, London.

[11]. Eldosouky, Adel I. (1996). “Principles of Construction Project Management.” Mansoura


University Press, Mansoura, Egypt.

[13]. Gould, Frederick E. (1997). “Managing the Construction Process: Estimating,


Scheduling, and Project Control.” Prentice-Hall Inc., New Gersy.

277
[14]. Harris, Frank & McCaffer, Ronald. (1983). “Modern Construction Management.”
Granada Publishing, Great Britain.

[15]. Harris, Robert. (1978). “Precedence and Arrow Networking Techniques for
Construction.” John Wiley & Sons Inc., NY.

[16]. Hegazy, T. (2002). “Computer-Based Construction Project Management.” Prentice Hall,


Upper Saddle River, NJ, USA.

[17]. Pilcher, Roy. (1992). “Principles of Construction Management.” Mc-Graw Hill Book
Company, 3rd ed.

[18]. Elgare Allah, Mohamed Ibrahim & Nawara, Jamal Mohamed. (1984). “Edarat
Almsharee’ Alhandaseah.” John Wiley & Sons Inc., NY. (This book is available in Arabic).

278
k.

TECHNICAL VOCATIONAL AND TRAINING


INSTITUTE (TVTI)
Yeka Subcity, Woreda 9, Addis Ababa, Ethiopia
Phone: 011-646-4455, Fax: 011-646-5675/5678
E-mail: info@etu.edu.et, Website: http://www.etu.edu.et

You might also like